108
EDITORS-IN-CHIEF JACC Valentin Fuster, MD, PhD, Mount Sinai School of Medicine, New York, NY JACC: Cardiovascular Interventions David J. Moliterno, MD, University of Kentucky, Lexington, KY JACC: Cardiovascular Imaging Y. Chandrashekhar, MD, DM, University of Minnesota/VAMC, Minneapolis, MN JACC: Heart Failure Christopher M. OConnor, MD, Inova Heart and Vascular Institute, Falls Church, VA JACC: Clinical Electrophysiology David J. Wilber, MD, Loyola University Medical Center, Chicago, IL JACC: Basic to Translational Science Douglas L. Mann, MD, Washington University School of Medicine, St. Louis, MO JACC: CardioOncology Bonnie Ky, MD, MSCE, Perelman School of Medicine at the University of Pennsylvania, Philadelphia, PA ASSOCIATE EDITORS Edgar Argulian, MD, Imaging, Mount Sinai Hospital, New York, NY Ami B. Bhatt, MD, Congenital Heart Disease, Massachusetts General Hospital, Harvard Medical School, Boston, MA Sanjeev Bhattacharyya, MD, PhD, Imaging, Barts Health Trust, London, UK Nowell Fine, MD, Heart Failure, University of Calgary, Calgary, Canada David L. Fischman, MD, Interventional Cardiology, Jefferson University, Philadelphia, PA Ricardo Fontes-Carvalho, MD, PhD, Clinical Cardiology, Centro Hospitalar de Vila Nova de Gaia/Espinho, Vila Nova de Gaia CHVNG/E, Porto, Portugal Eric M. Isselbacher, MD, Imaging, Massachusetts General Hospital, Harvard Medical School, Boston, MA Alejandro Jimenez Restrepo, MD, Rhythm Disorders, Heart & Vascular Center at the University of Maryland Medical Center, Baltimore, MD Soan Johar, MD, Rhythm Disorders, Ripas Hospital, Bandar Seri Begawan, Brunei Antonio Sorgente, MD, PhD, Rhythm Disorders, Epicura Centre Hospitalier, Hornu, Belgium Rafael Vidal Pérez, MD, PhD, Clinical Cardiology, University Hospital A Coruña, A Coruña, Spain Mladen I. Vidovich, MD, Coronary, Peripheral, and Structural Interventions, UI Health, Chicago, IL CME EDITOR Nadeen N. Faza, MD, Houston Methodist, Houston, TX SOCIAL MEDIA DIRECTORS Estefania Oliveros Soles, MD, Mount Sinai Hospital, New York, NY SOCIAL MEDIA CONSULTANTS Mohammed A. Chamsi-Pasha, MD, Houston Methodist, Houston, TX Alec A. Schmaier, MD, PhD, Brigham and Womens Hospital, Boston, MA Konstantin Krychtiuk, MD, PhD, Medical University of Vienna, Vienna, Austria Ali Jazayeri, MD, University of Kansas Medical Center, Kansas City, KS Purvi J. Parwani, MD, Loma Linda University Health, Loma Linda, CA EDITOR-IN-CHIEF Julia Grapsa, MD, PhD, Barts Health NHS Trust, London, UK DEPUTY EDITORS Mary Norine Walsh, MD, Heart Failure, St. Vincent Heart Center, Indianapolis, IN Eric R. Bates, MD, Coronary, Peripheral, and Structural Interventions, University of Michigan, Ann Arbor, MI GUEST EDITORS Sarah Moharem-Elgamal, MD, PhD, Imaging, Bristol Heart Institute, University Hospitals Bristol NHS Foundation Trust, Bristol, UK Dimosthenis Pandis, MD, MSc, Surgery, Mount Sinai Hospital, New York, NY DIVISIONAL SENIOR DIRECTOR, PUBLISHING Justine Varieur Turco, MA, American College of Cardiology, Washington, DC EXECUTIVE MANAGING EDITOR Eileen Cavanagh, MPS, American College of Cardiology, Washington, DC MANAGING EDITOR Donna Epler, MFA, American College of Cardiology, Washington, DC DIRECTOR, PRODUCT MANAGEMENT, DIGITAL PUBLISHING Nandhini Kuntipuram, MCA, PMP, American College of Cardiology, Washington, DC SOCIAL MEDIA COORDINATOR Tamika Edaire, BS, American College of Cardiology, Washington, DC Case Reports

Full Issue PDF · The JACC: Case Reports Editorial Board and ACC editorial office would like to express their sincere gratitude to the guest editors below who worked tirelessly …

  • Upload
    others

  • View
    14

  • Download
    0

Embed Size (px)

Citation preview

Page 1: Full Issue PDF · The JACC: Case Reports Editorial Board and ACC editorial office would like to express their sincere gratitude to the guest editors below who worked tirelessly …

EDITOR-IN-CHIEF

Julia Grapsa, MD, PhD,Barts Health NHS Trust,London, UK

DEPUTY EDITORS

Mary Norine Walsh, MD,Heart Failure,St. Vincent Heart Center,Indianapolis, IN

Eric R. Bates, MD,Coronary, Peripheral, and Structural Interventions,University of Michigan,Ann Arbor, MI

GUEST EDITORS

Sarah Moharem-Elgamal, MD, PhD,Imaging,Bristol Heart Institute, University HospitalsBristol NHS Foundation Trust, Bristol, UK

Dimosthenis Pandis, MD, MSc,Surgery,Mount Sinai Hospital,New York, NY

DIVISIONAL SENIOR DIRECTOR,

PUBLISHING

Justine Varieur Turco, MA,American College of Cardiology,Washington, DC

EXECUTIVE MANAGING EDITOR

Eileen Cavanagh, MPS,American College of Cardiology,Washington, DC

MANAGING EDITOR

Donna Epler, MFA,American College of Cardiology,Washington, DC

DIRECTOR, PRODUCT MANAGEMENT,

DIGITAL PUBLISHING

Nandhini Kuntipuram, MCA, PMP,American College of Cardiology,Washington, DC

SOCIAL MEDIA COORDINATOR

Tamika Edaire, BS,American College of Cardiology,Washington, DC

Case Reports

SOCIAL MEDIA

DIRECTORS

Estefania Oliveros Soles, MD,Mount Sinai Hospital,

JACC: Cardiovascular

Interventions

Ami B. Bhatt, MD,Congenital Heart Disease,

University of MarylandMedical Center,Baltimore, MD

New York, NY

SOCIAL MEDIA

CME EDITOR

Nadeen N. Faza, MD,Houston Methodist,Houston, TX

CONSULTANTS

MohammedA.Chamsi-Pasha,MD,Houston Methodist,Houston, TX

Alec A. Schmaier, MD, PhD,Brigham and Women’s Hospital,Boston, MA

Konstantin Krychtiuk, MD, PhD,Medical University of Vienna,Vienna, Austria

Ali Jazayeri, MD,University of KansasMedical Center,Kansas City, KS

Purvi J. Parwani, MD,Loma Linda University Health,Loma Linda, CA

EDITORS-IN-CHIEF

JACC

Valentin Fuster, MD, PhD,Mount Sinai School of Medicine, New York, NY

David J. Moliterno, MD,University of Kentucky, Lexington, KY

JACC: Cardiovascular Imaging

Y. Chandrashekhar, MD, DM,University of Minnesota/VAMC, Minneapolis, MN

JACC: Heart Failure

Christopher M. O’Connor, MD,Inova Heart and Vascular Institute,Falls Church, VA

JACC: Clinical Electrophysiology

David J. Wilber, MD,Loyola University Medical Center, Chicago, IL

JACC: Basic to Translational

Science

Douglas L. Mann, MD,Washington University School of Medicine,St. Louis, MO

JACC: CardioOncology

Bonnie Ky, MD, MSCE,Perelman School of Medicine at the Universityof Pennsylvania, Philadelphia, PA

ASSOCIATE EDITORS

Edgar Argulian, MD,Imaging,Mount Sinai Hospital,New York, NY

Massachusetts General Hospital,Harvard Medical School,Boston, MASanjeev Bhattacharyya, MD, PhD,Imaging,Barts Health Trust,London, UK

Nowell Fine, MD,Heart Failure,University of Calgary,Calgary, Canada

David L. Fischman, MD,Interventional Cardiology,Jefferson University,Philadelphia, PA

Ricardo Fontes-Carvalho,MD, PhD,Clinical Cardiology,Centro Hospitalar de Vila Nova deGaia/Espinho, Vila Nova de GaiaCHVNG/E,Porto, Portugal

Eric M. Isselbacher, MD,Imaging,Massachusetts General Hospital,Harvard Medical School,Boston, MA

Alejandro Jimenez Restrepo, MD,Rhythm Disorders,Heart & Vascular Center at the

Sofian Johar, MD,Rhythm Disorders,Ripas Hospital,Bandar Seri Begawan, Brunei

Antonio Sorgente, MD, PhD,Rhythm Disorders,Epicura Centre Hospitalier,Hornu, Belgium

Rafael Vidal Pérez, MD, PhD,Clinical Cardiology,University Hospital A Coruña,A Coruña, Spain

Mladen I. Vidovich, MD,Coronary, Peripheral, andStructural Interventions, UI Health,Chicago, IL

Page 2: Full Issue PDF · The JACC: Case Reports Editorial Board and ACC editorial office would like to express their sincere gratitude to the guest editors below who worked tirelessly …

TRAINING DIRECTORS

Alex Auseon, DO,Chicago Medicine, Columbus, OH

Eric R. Bates, MD,University of Michigan,Ann Arbor, MI

Doreen DeFaria Yeh, MD,Massachusetts General Hospital,Harvard Medical School,Boston, MA

George Lui, MD,Stanford Children’s Health,Stanford, CA

Lisa J. Rose-Jones, MD,UNC Health Care, Chapel Hill, NC

Harsimran S. Singh, MD,Weill Cornell Medicine,New York, NY

Chittur A. Sivaram, MD,Oklahoma University Medicine,Oklahoma City, OK

Mary Norine Walsh, MD,St. Vincent Heart Center,Indianapolis, IN

SENIOR EDITORIAL

CONSULTANTS

Gilles Dreufys, MD,Cardiothoracic Surgery,Institut Montsouris,Paris, France

Alex Auseon, DO,Clinical Cardiology,University of Illinois,Columbus, OH

Steven Droogmans, MD, PhD,Clinical Cardiology,UZ Brussel,Brussels, Belgium

Sharon Mulvagh, MD,Clinical Cardiology,Dalhousie University,Halifax, Nova Scotia

Lisa Rose-Jones, MD,Clinical Cardiology,University of North CarolinaHospitals,Chapel Hill, NC

Harsimran Singh, MD,Clinical Cardiology,Weill Cornell Medical Center,New York, NY

Chittur A. Sivaram, MD,Clinical Cardiology,OU Health Sciences Center,Oklahoma City, OK

Shelby Kutty, MD, PhD, MHCM,Congenital Heart Disease,John Hopkins Hospital,Baltimore, MD

Liesl Zuhlke, MBChB, PhD,Congenital Heart Disease,University of Cape Town,Cape Town, South Africa

Anne Marie Anagnostopoulos, MD,Coronary, Peripheral, andStructural Interventions,Beth Israel Deaconess Hospital,Boston, MA

Christopher S. Baker, MD,Coronary, Peripheral, andStructural Interventions,Imperial College NHS Trust,London, UK

Ankur Kalra, MD,Coronary, Peripheral, andStructural Interventions,Heart and Vascular Institute(Miller Family), Cleveland Clinic,Cleveland, OH

Arnold Hoo Seto, MD,Coronary, Peripheral, andStructural Interventions,UC Irvine Medical Center, Orange, CA

Alec P. Vahanian, MD, PhD,Coronary, Peripheral, andStructural Interventions,Bichat-Claude Bernard Hospital,Paris, France

Jelena Celutkiene, MD, PhD,Heart Failure,Vilnius University,Kaunas, Lithuania

Ali Vazir, MD, PhD,Heart Failure,Royal Brompton Hospital,London, UK

Santhi Adigopula, MD,Imaging,Sentara Heart Hospital,Norfolk, VA

Mark Dweck, MD, PhD,Imaging,University of Edinburgh,Edinburgh, UK

Maurice Lionel Enriquez Sarano, MD,Imaging,Mayo Clinic,Rochester, Minnesota

Gina LaRocca, MD,Imaging,Mount Sinai Hospital,New York, NY

Lawrence Rudski, MD,Imaging,Jewish General Hospital,Montreal, Canada

Alessandro Salustri, MD, PhD,Imaging,Heart Hospital,Doha, Qatar

Alexander Sasse, MD,Imaging,Wellington Hospital, Wellington,New Zealand

Nikolaos Tzemos, MD, PhD,Imaging,University of Western Ontario,Toronto, Canada

Hussam Ali, MD,Rhythm Disorders,Humanitas University,Milan, Italy

Pedro Brugada, MD, PhD,Rhythm Disorders,University Hospital of Brussel(UZ Brussel-VUB),Brussels, Belgium

Jose Angel Cabrera, MD, PhD,Rhythm Disorders,Catedrático de Cardiología UniversidadEuropea de Madrid,Madrid, Spain

Timm-Michael Dickfeld, MD, PhD,Rhythm Disorders,University of MarylandMedical Center,Baltimore, MD

Scott A. Harding, MD, MB ChB,Rhythm Disorders,Wellington Hospital,Wellington, New Zealand

Francois Regoli, MD, PhD,Rhythm Disorders,Cardiocentro Ticino,Lugano, Switzerland

Magdi M. Saba, MD,Rhythm Disorders,St George's Hospital,London, UK

Massimo Tritto, MD,Rhythm Disorders,Istituto Clinico Humanitas MaterDomini, Varese, Italy

Bruce L. Wilkoff, MD,Rhythm Disorders,Cleveland Clinic,Cleveland, OH

Carlos A. Mestres, MD, PhD,Surgery,University of Zurich,Zurich, Switzerland

Christopher Salerno, MD,Surgery,St. Vincent Medical Group,Indianapolis, IN

Alain Berrebi, MD, PhD,Valvular Heart Disease,Hôpital Européen Georges,Pompidou,Paris, France

Erwan Donal, MD, PhD,Valvular Heart Disease,Centre Hospitalier Universitairede Rennes,Rennes, France

Patrizio Lancellotti, MD, PhD,Valvular Heart Disease,University of Liège,Liège, Belgium

Julien Magne, PhD,Valvular Heart Disease,Centre Hospitalier Universitairede Limoges,Limoges, France

Fernando Gallardo, MD, PhD,Vascular Medicine,Universidad de Málaga,Marbella, Spain

EDITORIAL CONSULTANTS

C a r d i o - O n c o l o g y

Markus S. Anker, MD, PhD,Charite Hospital,Berlin, Germany

Sherry Ann Brown, MD, PhD,Medical College of Wisconsin,Milwaukee, WI

Charlotte Manisty, MD, PhD,Barts Health Trust,London, UK

Brijesh Patel, MD, DO,University of AlabamaSchool of Medicine,Morgantown, WV

C a r d i o t h o r a c i c S u r g e r y

Darren Freed, MD, PhD,University of Alberta,Alberta, Canada

Laszlo Gobolos, MD, PhD,Cleveland Clinic Abu Dhabi,Abu Dhabi, UAE

William D. Kent, MD, MSc,University of Calgary,Calgary, Canada

Andres Obeso, MD, PhD,Complejo Hospitalario Universitariode Santiago,Santiago de Compostela, Spain

Dimosthenis Pandis, MD, MSc,Mount Sinai Hospital,New York, NY

Alberto Pozzoli, MD, PhD,University of Zurich,Zurich, Switzerland

Neil Roberts, MD, PhD,Barts Health Trust,London, UK

Ibrahim Sultan, MD,UPMC Heart and Vascular Institute,Oakland, PA

Kaushal Tiwari, MD,College of Medical Sciences,Kathmandu, Nepal

C l i n i c a l C a r d i o l o g y

Ahmed Abuzaid, MD,Christiana Care Health Services,Newark, DE

Hassan Alfraidi, MD,McGill University,Montreal, Canada

Mosaad El Banna, MD,Cleveland Clinic Abu Dhabi,Abu Dhabi, UAE

Charles Butcher, MD,Harefield Hospital,London, UK

Kashif Chaudhry, MD, PhD,Mercy Cardiology Clinic,Cedar Rapids, IA

Monika Czaja-Ziółkowska, MD,University of Katowice,Katowice, Poland

Ersilia M. DeFilippis, MD,Brigham and Women’s Hospital,Boston, MA

Nakeya Dewaswala, MD,Atlantis,Lake Worth, FL

Edit Gara, MD,Semmelweis University,Budapest, Hungary

Amresh Gul, MD,Cardiovascular Center,Pakistan

Maria Holicka, MD,University Hospital Brno,Brno, Czech Republic

Page 3: Full Issue PDF · The JACC: Case Reports Editorial Board and ACC editorial office would like to express their sincere gratitude to the guest editors below who worked tirelessly …

EDITORIAL CONSULTANTS CONTINUED

Yakup Kilic, MD,West Middlesex Hospital,London, UK

Vishal Mehta, MD,Barts Health Trust,London, UK

Tereza Mikusova, MD,University Hospital Brno,Brno, Czech Republic

Mohammad Mostafa AnsariRamandi, MD,Syed Mostafa Khomaini Hospital,Tehran, Iran

Kyriacos Mouyis, MD,Barts Health Trust,London, UK

Nupoor Narula, MD,Mount Sinai Hospital,New York, NY

Yash Patel, MD,Mount Sinai Hospital,New York, NY

Garima Y. Sharma, MD,John Hopkins Hospital,Baltimore, MD

Garima V. Sharma, MD,Johns Hopkins Medicine,Baltimore, MD

Ahmed Taha, MD,Texas Tech UniversityHealth Sciences Center,Amarillo, Texas

Ayman Tantawy, MD,Zagazig University,Zagazig, Egypt

Sebastian Vandermolen, MD,Barts Health Trust,London, UK

Borejda Xhyheri, MD, PhD,Department of Clinical and BiologicalSciences, University of Torino,Bologna, Italy

Meghan York, MD,Beth Israel Deaconess Medical Center,Boston, MA

Co n g e n i t a l H e a r tD i s e a s e

Tarek Alsaied, MD,Cincinnati Children’s HospitalMedical Center,Cincinnati, OH

Matthew Carazo, MD,Emory Clinic,Atlanta, GA

Biagio Castaldi, MD, PhD,University of Padua,Padua, Italy

Giovanni Di Salvo, MD, PhD,Royal Brompton Hospital,London, UK

Wayne J. Franklin, MD,Phoenix Children’s Hospital,Phoenix, AZ

Rupali Gandhi, MD,Advocate Childrens Heart Institute,Oak Lawn, IL

Lasya Gaur, MD,John Hopkins Hospital,Baltimore, MD

Alex Llanos, MD,Memorial Healthcare System,Fort Lauderdale, FL

Amy Sarma, MD,Massachusetts General Hospital,Harvard Medical School,Boston, MA

Marianna Stamatelatou, MD, PhD,Chu de martinique,Martinique, France

Rose Tompkins, MD,Cedars-Sinai Medical Center,Los Angeles, CA

Co r o n a r y,P e r i p h e r a l , a n dS t r u c t u r a l I n t e r v e n t i o n s

Alberto Alfie, MD,Universidad of Buenos Aires,Buenos Aires, Argentina

Pablo Avanzas, MD, PhD,Hospital Universitario Centralde Asturias,Pamplona, Spain

Yiannis S. Chatzizisis, MD, MSc, PhD,University of Nebraska Medical Center,Omaha, NE

Omar Chehab, MD, PhD,St Thomas Hospital,London, UK

Ali Elzieny, MD,Sharm International Hospital,Sharm El Sheikh, Egypt

Giuseppe Di Gioia, MD,Università Campus Bio-Medicodi Roma, Rome, Italy

Joel Giblett, MD,Royal Jubilee Hospital,Victoria, BC, Canada

Sachin Goel, MD,Houston Methodist Hospital,Houston, TX

Bartosz Hudzik, MD, PhD,Medical University of Silesia,Zabrze, Poland

Tom Kaier, MD, PhD,Barts Health Trust,London, UK

Asim Katbeh, MD,Cardiovascular Center Aalst,Aalst, Belgium

Sena Kilic, MD,Brown University,Providence, RI

Anthony Li, MD,St George’s Hospital,London, UK

Joshua Loh, MD,Medstar Washington Hospital Center,Washington, DC

Saverio Muscoli, MD, PhD,Polyclinic Foundation Tor Vergata,Rome, Italy

Jignesh A. Patel, MD,Maimonides Medical Center,New York, NY

Maria Gimenez Rubini, MD, PhD,Universitätsspital Basel,Basel, Switzerland

Lucy M. Safi, MD, DO,Hackensack University Medical Center,Hackensack, NJ

José Carlos Moreno Samos, MD, MSc,Complejo Hospitalario de Jaén,Malaga, Spain

Elias Sanidas, MD, PhD,National and KapodistrianUniversity of Athens,Athens, Greece

Gabor G. Toth, MD, PhD,University Heart Centre Graz,Graz, Austria

Angelos Tyrlis, MD,Barts Health Trust,London, UK

Theodoros Zografos, MD, PhD,Red Cross Hospital,Athens, Greece

H e a r t F a i l u r e

Kim Anderson, MD,Dalhousie University,Halifax, Canada

Alberto Aimo, MD, PhD,Scuola Superiore Sant’Anna,Pisa, Italy

Yevgeniy “Eugene” Brailovsky, MD,Columbia University Medical Center,New York, NY

Marat Fudim, MD,Duke University Hospital,Durham, NC

Daniel Harding, MD,Barts Health Trust,London, UK

Mahwash Kassi, MD,Houston Methodist Hospital,Houston, TX

Konstantin Krychtiuk, MD, PhD,MedUni Vienna,Vienna, Austria

Hazem Lashin, MD, PhD,Barts Health Trust, London, UK

Rui Placido, MD, PhD,Lisbon Academic Medical Centre,Hospital de Santa Maria,Lisbon, Portugal

Alastair Proudfoot, MD, PhD,Barts Health Trust,London, UK

Amina Rakisheva, MD,Scientific Research Institute ofCardiology and Internal Diseases,Almaty, Kazakhstan

Ashwin Ravichandran, MD,St. Vincent Medical Group,Indianapolis, IN

Nosheen Reza, MD,University of Pennsylvania,Philadelphia, PA

Henry Savage, MD, MBBS,Essex Cardiothoracic Center,Essex, UK

Alec A. Schmaier, MD, PhD,Brigham and Women’s Hospital,Boston, MA

Kevin S. Shah, MD,University of Utah,Salt Lake City, UT

Sachin Shah, MD, PhD,Barts Health Trust,London, UK

Harri Silvola, MD, PhD,Oulu University Hospital,Oulu, Norway

Maria Simonenko, MD, PhD,Almazov National Medical ResearchCentre, St. Petersburg, Russia

Rosalba Spiritoso, MD, PhD,Barts Health Trust,London, UK

Ada C. Stefanescu, MD,Brigham and Women’s Hospital,Boston, MA

Maria Trêpa, MD,Centro Hospitalar do Porto,Porto, Portugal

Mihai Trofenciuc, MD,Institute of Cardiovascular Diseases,Arad, Romania

Giuseppe Vergaro, MD, PhD,Division of Cardiovascular MedicineFondazione Toscana GabrieleMonasterio,Pisa, Italy

Teerapat Yingchoncharoen, MD,Mahidol University,Bangkok, Thailand

H y p e r t e n s i o n

Anastasia S. Mihailidou, BSc, PhD,Northern Sydney Local Health District &Macquarie University,Sydney, Australia

I m a g i n g

Vikram Agarwal, MD,St. Luke’s Hospital,Chesterfield, MO

Amir Ahmadi, MD,Mount Sinai Hospital,New York, NY

Mohammed Akhtar, MD,Barts Health Trust,London, UK

Shehab Anwer, MD,University of Zurich,Zurich, Switzerland

Ricardo Avendano, MD,Yale University,New Haven, CT

Lilit Baghdassarian, MD,Hammersmith Hospital,London, UK

Nicole Bhave, MD,University of Michigan,Ann Arbor, MI

Page 4: Full Issue PDF · The JACC: Case Reports Editorial Board and ACC editorial office would like to express their sincere gratitude to the guest editors below who worked tirelessly …

EDITORIAL CONSULTANTS CONTINUED

Maria Boutsikou, MD, PhD,Mediterraneo Hospital,Athens, Greece

Mohammed A. Chamsi-Pasha, MD,Houston Methodist Hospital,Houston, TX

Athina Chasapi, MD, PhD,Barts Health Trust, London, UK

Michael Chetrit, MD,Cleveland Clinic,Cleveland, OH

Esther Carbonero Cortinas, MD, PhD,University Hospital of Valladolid,Valladolid, Spain

Jason Dungu, MD, PhD,Essex Cardiothoracic Center,Essex, UK

Nadeen N. Faza, MD,Houston Methodist,Houston, TX

Emilio Fentanes, MD,Tripler Army Medical Center,Honolulu, HI

Laura Galian Gay, MD, PhD,Vall D’Hebron Hospital General,Barcelona, Spain

Jose Juan Gomez de Diego, MD, PhD,Hospital Clínico de Madrid,Madrid, Spain

Dan Halpern, MD,Mount Sinai Hospital,New York, NY

Csilla Jozsa, MD,Barts Health Trust,London, UK

Tomas Lapinskas, MD, PhD,Lithuanian University ofHealth Sciences,Kaunas, Lithuania

Savvas Loizos, MD, PhD,Errikos Ntynan Hospital,Athens, Greece

Marcelo H. Miglioranza,MD, MHSc, PhD,University Foundation of Cardiology,Porto Alegre, Brazil

Sara Moscatelli, MD, PhD,University of Genoa,Scuola di Specializzazione in Malattiedell’apparato Cardiovascolare,Genoa, Italy

Manish Motwani, MD, PhD,Manchester Heart Centre,Manchester, UK

Maria Carmo Nunes, MD, PhD,Federal University of Minas Gerais,Belo Horizonte (UFMG),Minas Gerais, Brazil

Didem Oguz, MD, PhD,Mayo Clinic,Rochester, MN

Ozge Ozden Tok, MD,Memorial Bahcelievler Hospital,Istanbul, Turkey

Costas Papadopoulos, MD, PhD,Red Cross Hospital,Athens, Greece

Ferande Peters, MD, PhD,Wits Hospital,Johannesburg,South Africa

Tomaz Podlesnikar, MD, PhD,University Medical Centre Maribor,Maribor, Slovenia

Stefania Rosmini, MD, PhD,Barts Health Trust,London, UK

Jolanda Sabatino, MD, PhD,Magna Graecia University of Catanzaro

Hoda Shehata, MD,University of Alexandria,Alexandria, Egypt

Iana Simova, MD,National Cardiology Hospital,Sofia, Bulgaria

Aiman Smer, MD,Creighton University, Omaha, NE

Carla Sousa, MD, PhD,Centro Hospitalar de São João,Lisbon, Portugal

Elena Surkova, MD, PhD,Royal Brompton Hospital,London, UK

Ritu Thamman, MD,University of PittsburghMedical School,Pittsburgh, PA

Miltiadis Triantafyllou, MD,Halmstad County Hospital,Halmstad, Sweden

Svetlin Tsonev, MD, PhD,Medical University of Sofia,University Hospital Alexandrovska,Sofia, Bulgaria

Seth Uretsky, MD,Morristown Medical Center,Morristown, NJ

I n t e n s i v e C a r e /H e a r t F a i l u r e

Stephen J. Shepherd, MD,Barts Health Trust,London, UK

Ou t c om e sR e s e a r c h a n d Q u a l i t y

Altibi Ahmed, MD,Harvard Medical University,Cambridge, MA

Adrienne H. Kovacs, MD, PhD,Knight Cardiovascular Institute atOHSU, Toronto, Canada

Shazia A. Mitha, MSN,Columbia University Medical Center,New York, NY

R h y t hm D i s o r d e r s

Richard Amara, MD,University of Baltimore,Baltimore, MD

Jason Appelbaum, MD,University of Baltimore,Baltimore, MD

Babken Asatryan, MD, PhD,Inselspital, University of Bern,Bern, Switzerland

Pawel Balsam, MD, PhD,Medical University of Warsaw,Warsaw, Poland

Hector Banchs-Viñas, MD,University of Puerto Rico,San Juan, Puerto Rico

William Bautista, MD,University of Colombia,Manizales, Colombia

Richard Soto Becerra, MD,Clinical La Luz, Lima, Perú

Jhan Carlos Altamar Castillo, MD,Clinica General de Norte,Barranquilla, Colombia

Eugene H. Chung, MD,University of Michigan,Michigan, ILIan Crozier, MD,Canterbury District Health Board,Christchurch, New Zealand

Abel Casso Dominguez, MD,Mount Sinai Hospital, New York, NY

Juan Viles Gonzalez, MD,Baptist Hospital of Miami,Kendall, FL

Muhammad Haq, MD,University of Maryland,School of Medicine,Baltimore, MD

Carina Hardy, MD,University of Sao Paulo,San Paulo, Brazil

Jeffrey Hirsch, MD,University of Maryland,School of Medicine,Baltimore, MD

Adedapo Iluyomade, MD,University of Miami Hospital,Miami, FL

Jean Jeudy, MD,University of Baltimore,Baltimore, MD

Jedrjej Kosiuk, MD, PhD,University of Leipzig,Liepzig, Germany

Valentina Kutyifa, MD, PhD,University of Rochester Medical Center,Rochester, MI

Federico J. Malavassi Corrales, MD,San Juan de Dios Hospital,San Jose, Costa Rica

Christos Maniotis, MD, PhD,Ygeia Hospital,Athens, Greece

Daniel Nguyen, MD,University of Baltimore,Baltimore, MD

Cristina Raimondo, MD, PhD,Unité médico-chirurgicale deCardiologie Congenitale et pédiatrique,Paris, France

Juan David Ramirez, MD,Universidad del Rosario,Medellin, Colombia

Diego A. Rodriguez, MD,Fundación Cardioinfantil Institutode Cardiologia,Bogota, Colombia

Luis Carlos Saenz, MD,Fundación Cardioinfantil Institutode Cardiologia,Bogota, Colombia

Paul Schurmann, MD,Houston Methodist Hospital,Houston, TXAfzal Sohaib, MD, PhD,Barts Health Trust,London, UK

Martin K. Stiles, MD, PhD,Waikato Hospital,Hamilton, New Zealand

David R. Tomlinson, MD, PhD,University Hospitals PlymouthNHS Trust,Plymouth, UK

Mauro Toniolo, MD, PhD,University Hospital of S. Maria DellaMisericordia,Udine, Italy

Pasquale Vergara, MD, PhD,Ospedale S. Raffaele,Milan, Italy

Syed Zawahir Hassan, MD,TIRR Memorial Hermann,Houston, TX

V a l v u l a r H e a r t D i s e a s e

Jean Deschamps, MD,University of Alberta,Edmonton, Canada

Akihisa Kataoka, MD, PhD,Teikyo University,Tokyo, Japan

Dae-Hee Kim, MD, PhD,Asan Medical Center,Seoul, Korea

Francois Tournoux, MD,Centre Hospitalier Universitaire deMontréal CHUM Montreal,Montreal, Canada

Page 5: Full Issue PDF · The JACC: Case Reports Editorial Board and ACC editorial office would like to express their sincere gratitude to the guest editors below who worked tirelessly …

Case Reports

CORRESPONDENCE FOR

AMERICAN COLLEGE OF

CARDIOLOGY

All correspondence for the College,

other than that related to

JACC: Case Reports should be

sent to Resource Center,

American College of Cardiology,

2400 N Street,

NW, Washington, DC 20037

Dipti Itchhaporia, MD, FACC,Vice President

Howard “Bo” T. Walpole, Jr., MD, MBA, FACC, Treasurer

2020-2021 OFFICERS

Athena Poppas, MD, FACC, President

Daniel M. Philbin, Jr., MD, FACC, Secretary and Board of Governors Chair

Cathleen C. Gates,Acting Chief Executive Officer

2020-2021 PUBLICATIONS AND EDITORIAL COORDINATION COMMITTEE

Viviany R. Taqueti, MD, MPH, FACC,Chair

Rhonda M. Cooper-DeHoff, MD, FACC

John U. Doherty, MD, FACC

Islam Y. Elgendy, MD, FACC

Prasad C. Gunasekaran, MD

Fadi G. Hage, MD, FACC

Fred M. Kusumoto, MD, FACC

Renato D. Lopes, MD, PhD, FACC

Sandra M. Oliver-McNeil, DNP, ACNP-BC

Syed Tanveer Rab, MBBS, MACC

Janice Sibley, MS, MA,ACC Executive Vice President, Education and Publishing Divisions

Justine Varieur Turco, MA,ACC Divisional Senior Director, Publishing

Page 6: Full Issue PDF · The JACC: Case Reports Editorial Board and ACC editorial office would like to express their sincere gratitude to the guest editors below who worked tirelessly …

The JACC: Case Reports Editorial Board and ACC editorial office would like to express their sincere gratitude to the guesteditors below who worked tirelessly for the completion of this ACC.20 dedicated issue.

Babken Asatryan (Bern, Switzerland)

Athina Chasapi (London, United Kingdom)

Yiannis Chatzizisis (Omaha, Nebraska)

John Hirshfeld (Philadelphia, Pennsylvania)

Brijesh Patel (Morgantown Virginia)

Nicolo Piazza (Montreal, Canada)

Ashwin Ravichandran (Indianapolis, Indiana)

Sachin Shah (London, United Kingdom)

Mohammad-Ali Jazayeri (Kansas City, Kansas)

Tomas Lapinskas (Kaunas, Lithuania)

Jose Carlos Moreno Samos (Malaga, Spain)

Ritu Thamman (Pittsburgh, Pennsylvania)

Nikolaos Tzemos (Ontario, Canada)

Page 7: Full Issue PDF · The JACC: Case Reports Editorial Board and ACC editorial office would like to express their sincere gratitude to the guest editors below who worked tirelessly …

J A C C : C A S E R E P O R T S VO L . 2 , N O . 7 , 2 0 2 0

ª 2 0 2 0 T H E A U T H O R S . P U B L I S H E D B Y E L S E V I E R O N B E H A L F O F T H E A M E R I C A N

C O L L E G E O F C A R D I O L O G Y F OU N D A T I O N . T H I S I S A N O P E N A C C E S S A R T I C L E U N D E R

T H E C C B Y - N C - N D L I C E N S E ( h t t p : / / c r e a t i v e c o mm o n s . o r g / l i c e n s e s / b y - n c - n d / 4 . 0 / ) .

CASE REPORT

HEART CARE TEAM/MULTIDISCIPLINARY TEAM LIVE

Successful Triplet Pregnancy Post-Allogeneic Stem Cell Transplant ina Patient With Doxorubicin-InducedCardiomyopathy

Vanthana Bharathi, MBBS,a Mishita Goel, MD,a,b Xavier F. Pombar, DO,c Sunita Nathan, MD,d

Tochukwu M. Okwuosa, DOa

ABSTRACT

ISS

Fro

Illi

Ru

Se

tha

Th

ins

vis

Ma

We report the unique case of a patient who recovered cardiac function despite a history of doxorubicin-induced car-

diomyopathy, chest radiation therapy, high dose chemotherapy post-allogeneic stem cell transplant, and triplet preg-

nancy. Data are sparse on doxorubicin-induced cardiomyopathy in pregnant patients, calling for further studies to help

formulate management or surveillance recommendations. (Level of Difficulty: Advanced.) (J Am Coll Cardiol Case Rep

2020;2:987–90) © 2020 The Authors. Published by Elsevier on behalf of the American College of Cardiology Foundation.

This is an open access article under the CC BY-NC-ND license (http://creativecommons.org/licenses/by-nc-nd/4.0/).

A 27-year-old Hispanic female with a past med-ical history of natural killer (NK)/T-cell lym-phoma, nasal type, presented in 2014 after

12 years of achieving complete remission, with com-plaints of fever, cough, shortness of breath, and a20-lb weight loss over 2 to 3 months. For her lym-phoma, she had received treatment with 8 cycles ofcyclophosphamide, doxorubicin, vincristine, andprednisone (CHOP) and radiation therapy (XRT); totaldoxorubicin dose w320 mg/m2. She had otherwisebeen active and in relatively good health before thesesymptoms developed. She denied complaints of chestdiscomfort, dizziness, syncope, palpitations, lowerextremity edema, or claudication symptoms. She hadan unremarkable physical examination with no

N 2666-0849

m the aDepartment of Internal Medicine, Division of Cardiovascular D

nois; bDepartment of Internal Medicine, Wayne State University, Detroit,

sh University Medical Center, Chicago, Illinois; and the dDepartment of In

ction of Bone Marrow Transplant and Cell therapy, Rush University Medica

t they have no relationships relevant to the contents of this paper to dis

e authors attest they are in compliance with human studies committe

titutions and Food and Drug Administration guidelines, including patien

it the JACC: Case Reports author instructions page.

nuscript received January 29, 2020; revised manuscript received April 3,

jugular venous distention, normal apical impulse, noS3 or S4; she had clear lungs bilaterally, and no lowerextremity edema. Her vital signs were stable withblood pressure of 108/57 mm Hg, pulse rate of 86beats/min, respiratory rate of 20 cycles/min, and oxy-gen saturation of 100%. Initial laboratory values werewithin normal limits.

Upon evaluation, she was found to havemoderate bilateral pleural effusions for which sheunderwent thoracentesis with flow cytometryrevealing NK lymphocytosis and confirming diseaserelapse. Further workup included a pre-treatmentechocardiogram that demonstrated a left ventricularejection fraction (LVEF) of 37% (Videos 1A and 1B),confirmed by cardiac magnetic resonance imaging.

https://doi.org/10.1016/j.jaccas.2020.04.020

iseases, Rush University Medical Center, Chicago,

Michigan; cDepartment of Obstetrics & Gynecology,

ternal Medicine, Division of Hematology/Oncology,

l Center, Chicago, Illinois. The authors have reported

close.

es and animal welfare regulations of the authors’

t consent where appropriate. For more information,

2020, accepted April 9, 2020.

Page 8: Full Issue PDF · The JACC: Case Reports Editorial Board and ACC editorial office would like to express their sincere gratitude to the guest editors below who worked tirelessly …

ABBR EV I A T I ON S

AND ACRONYMS

ASCT = allogeneic stem cell

transplant

CHF = congestive heart failure

GDMT = guideline-directed

medical therapy

LVEF = left ventricular ejection

fraction

RIC = reduced intensity

conditioning

SCT = stem cell transplant

XRT = radiation therapy

Bharathi et al. J A C C : C A S E R E P O R T S , V O L . 2 , N O . 7 , 2 0 2 0

Triplet Pregnancy Post-ASCT and Doxorubicin-Induced Cardiomyopathy J U N E 1 7 , 2 0 2 0 : 9 8 7 – 9 0

988

QUESTION 1: WHAT IS THE

DIFFERENTIAL DIAGNOSIS IN

OUR PATIENT?

ANSWER 1. The results of our initial work updid not support a diagnosis of myocarditis,infiltrative cardiomyopathy, or ischemia/infarction. Further investigation into thisnew-onset cardiomyopathy yielded normalcardiac enzyme, B-type natriuretic peptide,and thyroid-stimulating hormone levels, andnormal iron saturation; the electrocardio-gram revealed no ischemic changes.

This 27-year-old patient with normal car-

diac biomarker levels and no cardiac risk factors,presenting with overt symptoms of heart failure 12years after receiving combination chemotherapy hascardiomyopathy likely secondary to the combinedeffects of her prior doxorubicin-based chemotherapyand mediastinal radiation therapy.

Doxorubicin cardiotoxicity is cumulative, and apooled analysis of 630 patients who underwentconventional doxorubicin treatment in 3 controlledtrials reported a congestive heart failure (CHF)incidence of 5% at a cumulative dose of 400 mg/m2,rising to 16% at a dose of 500 mg/m2, 26% at a doseof 550 mg/m2, and 48% at a dose of 700 mg/m2 (1).However, a study of 141 patients who receivedconventional doxorubicin for lymphoma by Hequetet al. (2), showed that CHF occurred in 25% of pa-tients who received a cumulative dose of 300mg/m2. Our patient received 320 mg/m2 of doxoru-bicin, which is above the 250 to 300 mg/m2

threshold where the lifetime risk of cardiomyopathystarts to increase (1).

XRT has been associated with significant cardio-vascular toxicity, particularly ischemic heart disease.Cardiovascular disease incidence was assessed in1,474 survivors of Hodgkin’s lymphoma by Alemanet al. (3) in 2006 with a median follow-up of 18.7years. Multivariable Cox regression and competingrisk analyses were used to quantify treatment effectson cardiovascular disease risk. The risks of CHFsignificantly increased when mediastinal radio-therapy was combined with anthracycline-containingchemotherapy (hazard ratio: 2.81; 95% confidenceinterval: 1.44 to 5.49) (3).

QUESTION 2: HOW DID WE MANAGE

THIS PATIENT?

ANSWER 2. The patient was subsequently initiatedon guideline-directed medical therapy (GDMT) for

heart failure with carvedilol 3.125 mg and lisinopril2.5 mg daily. For her recurrent lymphoma, shereceived 4 cycles of steroid (dexamethasone), meth-otrexate (with leucovorin), ifosfamide (with mesna),L-asparaginase, and etoposide (SMILE) chemo-therapy. She was followed closely by cardio-oncologythroughout her treatment, and there were no heartfailure exacerbations through that time. She was thenreferred for evaluation for stem cell transplant (SCT)as a potentially curative option. A 10/10 match wasfound in her older brother, and she successfully un-derwent fludarabine/melphalan conditioning fol-lowed by a matched related donor allogeneic stemcell transplant (ASCT).

One year after ASCT with continued GDMT forheart failure, she remained clinically stable and wasnoted to have a near-normal LVEF of 55% on follow-up echocardiogram (Video 2).

The patient subsequently presented with a positivepregnancy test, and her pregnancy was confirmed bygynecologic ultrasound to be triplets (spontaneoustrichorionic triamniotic gestation). The patient hadbeen counseled against pregnancy for the first 2 yearspost-transplant but got pregnant while on immuno-suppressive therapy post-SCT and GDMT for heartfailure.

QUESTION 3: WHAT ARE THE CHALLENGES

TO PREGNANCY IN A CANCER PATIENT

POST-ASCT AND WITH A HISTORY OF

HEART FAILURE?

ANSWER 3. The patient conceived while she wasreceiving immunosuppressive therapy and GDMT forheart failure, which included lisinopril, a knownteratogenic. Additionally, continuation of pregnancyin our patient warranted discontinuation of thesedrugs. She also had a history of aggressive lymphoma,relapse, and was post-ASCT. She was thereforeconsidered a patient at high risk for both lymphomarelapse and worsened heart failure during herpregnancy.

Pregnancy is high risk in women with LVEFbelow 45% (World Health Organization functionalclass III) and is contraindicated if LVEF is below30% (World Health Organization functional class IV)(4). Pregnancy should also be avoided during thefirst 2 years post-transplant due to the risk ofrelapse and management of other post-transplantcomplications (5). Therefore, effective contracep-tion is crucial for those patients in whom pregnancyis contraindicated.

Page 9: Full Issue PDF · The JACC: Case Reports Editorial Board and ACC editorial office would like to express their sincere gratitude to the guest editors below who worked tirelessly …

J A C C : C A S E R E P O R T S , V O L . 2 , N O . 7 , 2 0 2 0 Bharathi et al.J U N E 1 7 , 2 0 2 0 : 9 8 7 – 9 0 Triplet Pregnancy Post-ASCT and Doxorubicin-Induced Cardiomyopathy

989

Furthermore, women with multiple gestation areknown to have an increased risk of maternal andperinatal morbidity and mortality. Maternal risks ofmultifetal pregnancies include hypertension, pre-eclampsia, gestational diabetes, postpartum hemor-rhage, and peripartum cardiomyopathy. Perinatalcomplications include preterm delivery, small forgestational age, intrauterine fetal death. The risks ofperinatal morbidity and mortality increase with thepresence of each additional fetus (6). For example,the risk of spontaneous loss of the entire pregnancy is25% for quadruplets, 15% for triplets, and 8% fortwins.

Our patient received intense counseling as a high-risk obstetrics patient. She was offered the choice ofabortion and also multifetal pregnancy reduction. Shewas also educated regarding her risk of lymphomarelapse and heart failure. However, the patient optedto keep her pregnancy.

QUESTION 4: HOW WAS THE PATIENT

MANAGED DURING HER PREGNANCY?

ANSWER 4. The management of pregnancy in theconcurrence of heart failure and cancer presents anenormous challenge to health care providers. Amultidisciplinary team, which included cardio-oncology, SCT hematology, and high-risk obstet-rics/maternal fetal medicine, were involved inproviding care during the pregnancy. Many of hermedications were discontinued, including herimmunosuppressive therapy and lisinopril. Sheremained on carvedilol and prenatal vitamins, andwas followed very closely by the involved teams,with initial monthly visits and monthlyechocardiograms.

During the third trimester, the patient beganbimonthly visits with the teams. Her LVEFremained stable until about 32 weeks of gestationwhen she began to have worsening dyspnea. Herechocardiogram showed decreasing LVEF of w45%(Video 3), moderate tricuspid regurgitation, andborderline pulmonary hypertension (right ventricu-lar systolic pressure w35 mm Hg). She had baselinetachycardia (heart rate 120 to 130 beats/min), andwas therefore followed and managed weekly. Hercarvedilol was switched to metoprolol succinate toallow improvement in her relative hypotensionduring pregnancy.

At 36 weeks of gestation, the patient deliveredvia cesarean section. Her C-section was compli-cated by bleeding from an atonic uterus, whichrequired blood transfusion support and eventually

a total hysterectomy. She was discharged onpost-op day 4. The 3 neonates did very well(APGAR [appearance, pulse, grimace, activity, andrespiration] scores of 9 each) and were dischargedwith their mother. The patient opted out ofbreastfeeding; lisinopril was reinitiated, and sheremained on GDMT for her heart failure. She hadinitial close follow-up and remained clinically sta-ble in the postpartum period. Two years later, shecontinues to follow with our SCT team, withoutevidence of graft versus host disease or recurrenceof her lymphoma. Her LVEF has recovered to 45%to 50% and has remained stable throughout herfollow-up visits/testing.

QUESTION 5: WHAT ARE THE LEARNING

POINTS FROM THIS CASE?

ANSWER 5. This is an unprecedented case of a patientwho recovered well despite a history of doxorubicin-induced cardiomyopathy, chest radiation therapy,followed by ASCT, and a triplet pregnancy.

Long-term cancer survivors are increasing innumber with increased success of chemotherapeuticregimens and improved transplant-related outcomes.However, there is still a high burden of late morbidityand mortality. One of the most distressing conse-quences of SCT is the high incidence of infertility,particularly with myeloablative conditioning regi-mens (7) with 2 or more alkylating agents. In recentyears, a considerable number of young patients havereceived SCT with less toxic, more tolerable reducedintensity conditioning (RIC) regimens. Although re-covery of ovarian function and preservation offertility have been studied in patients receiving RICregimens (8), there are only a few case reports ofsuccessful pregnancies in this cohort. Our case reportis one of the few to report a successful pregnancyusing a RIC regimen consisting of fludarabine/melphalan.

Although doxorubicin is a well-known cause ofcardiotoxicity, the normal physiological changes thatoccur during a normal pregnancy, peaking in thesecond trimester, are stressful to the cardiovascularsystem. During pregnancy, blood volume increases by45% to 50%, and cardiac output rises 30% to 50%above baseline. These changes are tolerated well bynormal healthy adults. However, patients withanthracycline exposure such as our patient, are atincreased risk of developing heart failure exacerba-tion during pregnancy.

Doxorubicin cardiotoxicity in the peripartumperiod has been described in very few case reports.

Page 10: Full Issue PDF · The JACC: Case Reports Editorial Board and ACC editorial office would like to express their sincere gratitude to the guest editors below who worked tirelessly …

PERSPECTIVES

A UNIQUE CASE: This is an unprecedented report of

a patient who recovered well despite a history of

doxorubicin-induced cardiomyopathy, chest radiation

therapy, followed by ASCT, and a triplet pregnancy.

COMPETENCY IN PATIENT CARE: Since manage-

ment of pregnancy in cancer patients is based on

limited evidence, a collaborative effort of a multidis-

ciplinary team is indispensable. Each patient might

present with a different challenge, so an individual-

ized approach is needed to manage the various stages

of pregnancy and perinatal care.

COMPETENCY IN INTERPERSONAL AND

COMMUNICATION SKILLS: Specialists might be

the only source of health information for these pa-

tients. So, all patients should be counseled regarding

contraception and educated about the risks and

challenges in pregnancy and symptoms of heart fail-

ure to ensure early intervention or refer them to

appropriate specialists.

Bharathi et al. J A C C : C A S E R E P O R T S , V O L . 2 , N O . 7 , 2 0 2 0

Triplet Pregnancy Post-ASCT and Doxorubicin-Induced Cardiomyopathy J U N E 1 7 , 2 0 2 0 : 9 8 7 – 9 0

990

Overall, the clinical manifestations and response toGDMT for heart failure and the outcomes of latedoxorubicin-induced cardiomyopathy are variable(9). One of the studies by MD Anderson Cancer Centerevaluated cardiac outcomes of childhood cancer sur-vivors who had pregnancies and were previouslyexposed to anthracyclines and/or chest XRT (10). Thestudy compared 58 patients who had pregnancies to acontrol group of 80 women without pregnancies fromthe same population with mean anthracycline dose of272 mg/m2 and median follow-up time of 20 years.Pregnancy was associated with 2.35-fold increase inrisk of cardiotoxicity in the overall study population(95% confidence interval: 1.02 to 5.41; p ¼ 0.045).From this study, younger age at the time of cancerdiagnosis, longer time from cancer treatment to firstpregnancy, and higher total anthracycline dose weresome of the high-risk factors identified.

A multidisciplinary approach was paramount to thesuccessful management of this patient’s cardiovas-cular health during pregnancy. With collaborativeefforts and early intervention, a successful outcomeis possible as documented in our case. There aresparse data on doxorubicin-induced cardiomyopathyin pregnant patients; and although managementguidelines for peripartum cardiomyopathy are welldefined regardless of etiology, there are only a fewreports and no standard guidelines for special con-siderations in pregnant women perichemotherapy ortransplant. Further studies are paramount to helpformulate management or surveillance recommen-dations in this patient population.

ADDRESS FOR CORRESPONDENCE: Prof. TochukwuM. Okwuosa, Department of Internal Medicine, Divi-sion of Cardiovascular Diseases, Rush UniversityMedical Center, 1717 West Congress Parkway, KelloggBuilding, Suite 328, Chicago, Illinois 60612. E-mail:[email protected].

RE F E RENCE S

1. Swain SM, Whaley FS, Ewer MS. Congestiveheart failure in patients treated with doxorubicin.Cancer 2003;97:2869–79.

2. Hequet O, Le QH, Moullet I, et al. Subclinicallate cardiomyopathy after doxorubicin therapy forlymphoma in adults. J Clin Oncol 2004;22:1864–71.

3. Aleman BM, van den Belt-Dusebout AW,DeBruin ML, et al. Late cardiotoxicity after treat-ment for Hodgkin lymphoma. Blood 2007;109:1878–86.

4. Regitz-Zagrosek V, Blomstrom-Lundqvist C,Borghi C, et al. ESC guidelines on the managementof cardiovascular diseases during pregnancy: theTask Force on the Management of CardiovascularDiseases During Pregnancy of the European Soci-ety of Cardiology (ESC). Eur Heart J 2011;32:3147–97.

5. Majhail NS, Rizzo JD, Lee SJ, et al. Recom-mended screening and preventive practices for

long-term survivors after hematopoietic celltransplantation. Bone Marrow Transplant 2012;47:337–41.

6. Evans MI, Britt DW. Multifetal pregnancyreduction: evolution of the ethical arguments.Semin Reprod Med 2010;28:295–302.

7. Sklar CA, Mertens AC, Mitby P, et al. Prematuremenopause in survivors of childhood cancer: areport from the childhood cancer survivor study.J Natl Cancer Inst 2006;98:890–6.

8. Papageorgiou SG, Ahmed O, Narvekar N,Davies M, Kottaridis PD. Preservation of fertilityin women undergoing reduced-intensity condi-tioning allogeneic transplantation with afludarabine-based regime. Transplantation 2012;24:29–30.

9. Hines MR, Mulrooney DA, Hudson MM, et al.Pregnancy-associated cardiomyopathy in survivorsof childhood cancer. J Cancer Surviv 2016;10:113–21.

10. Thompson KA. Pregnancy and cardiomyopathyafter anthracyclines in childhood. Front CardiovascMed 2018;5:14.

KEY WORDS cancer, cardiomyopathy,doxorubicin, stem cell transplant, tripletpregnancy

APPENDIX For supplemental videos,please see the online version of thispaper.

Go to http://www.acc.org/jacc-journals-cme to takethe CME/MOC/ECME quizfor this article.

Page 11: Full Issue PDF · The JACC: Case Reports Editorial Board and ACC editorial office would like to express their sincere gratitude to the guest editors below who worked tirelessly …

J A C C : C A S E R E P O R T S VO L . 2 , N O . 7 , 2 0 2 0

ª 2 0 2 0 T H E A U T H O R S . P U B L I S H E D B Y E L S E V I E R O N B E H A L F O F T H E A M E R I C A N

C O L L E G E O F C A R D I O L O G Y F OU N D A T I O N . T H I S I S A N O P E N A C C E S S A R T I C L E U N D E R

T H E C C B Y - N C - N D L I C E N S E ( h t t p : / / c r e a t i v e c o mm o n s . o r g / l i c e n s e s / b y - n c - n d / 4 . 0 / ) .

CASE REPORT

CLINICAL CASE

Percutaneous Coil Embolization ofConfluent Bilateral CoronaryArtery Fistulas

Shuji Kubota, MD, Hisao Hara, MD, PHD, Masaya Yamamoto, MD, Yukio Hiroi, MD, PHD

ABSTRACT

L

ISS

Fro

tha

Ed

Th

ins

vis

Ma

A 52-year-old woman presented with a continuous heart murmur at an annual health examination. Coronary computed

tomography angiography showed confluent coronary artery fistulas meeting to form a large aneurysm fistulizing to the

pulmonary artery. Percutaneous coil embolization was performed, resulting in the disappearance of the coronary artery

fistulas and aneurysms. (Level of Difficulty: Intermediate.) (J Am Coll Cardiol Case Rep 2020;2:991–5)

© 2020 The Authors. Published by Elsevier on behalf of the American College of Cardiology Foundation. This is an

open access article under the CC BY-NC-ND license (http://creativecommons.org/licenses/by-nc-nd/4.0/).

HISTORY OF PRESENTATION

A 52-year-old woman presented with a continuousheart murmur from childhood observed at her annualhealth examination. Chest pain and shortness of

EARNING OBJECTIVES

Coronary fistulas are relatively rare coronaryartery abnormalities, but myocardialischemia and congestive heart failure mayoccur if there is a large amount of shunt.When an aneurysm is also present, there is arisk of rupture. Treatment of coronary arteryfistulas should be considered.Surgical ligation and transcatheter emboli-zation can be used to treat coronary fistulas.Because the latter may be safer and lessinvasive, it should be used when the risk ofadverse events is high.

N 2666-0849

m the Department of Cardiology, National Center for Global Health and

t they have no relationships relevant to the contents of this paper to disclo

itor for this paper.

e authors attest they are in compliance with human studies committe

titutions and Food and Drug Administration guidelines, including patien

it the JACC: Case Reports author instructions page.

nuscript received February 13, 2020; revised manuscript received April 3

breath were absent. Her vital signs were withinnormal limits.

She had dyslipidemia and migraine, both stable bymedication, and no family history of cardiac disease.

DIFFERENTIAL DIAGNOSIS

Continuous murmurs are hallmarks of left-to-rightshunting, as seen in patent ductus arteriosus andruptured aneurysm of sinus of Valsalva.

INVESTIGATIONS

The only significant blood abnormality was aslightly elevated brain natriuretic peptide concen-tration (45.6 pg/ml). Twelve-lead electrocardiog-raphy (ECG) showed normal sinus rhythm, whereasa double Master stress test with ECG showedno ischemic changes. Transthoracic echocardiogra-phy showed normal left ventricular function

https://doi.org/10.1016/j.jaccas.2020.04.034

Medicine, Tokyo, Japan. The authors have reported

se. Yiannis Chatzizisis, MD, PhD, served as the Guest

es and animal welfare regulations of the authors’

t consent where appropriate. For more information,

, 2020, accepted April 13, 2020.

Page 12: Full Issue PDF · The JACC: Case Reports Editorial Board and ACC editorial office would like to express their sincere gratitude to the guest editors below who worked tirelessly …

FIGURE 1 Coronary Computed Tomography AngiographyABBR EV I A T I ON S

AND ACRONYMS

CAF = coronary artery fistula

CAG = coronary angiography

CTA = computed tomography

angiography

ECG = electrocardiography

PA = pulmonary artery

Qp/Qs = ratio of pulmonary

blood flow to systemic blood

flow

RCA = right coronary artery

RHC = right-sided heart

catheterization

RV = right ventricle

Kubota et al. J A C C : C A S E R E P O R T S , V O L . 2 , N O . 7 , 2 0 2 0

Coronary Artery Fistulas with Aneurysm J U N E 1 7 , 2 0 2 0 : 9 9 1 – 5

992

with normal wall motion, valvular function,and dimension of the right ventricle (RV).

Coronary computed tomography angiog-raphy (CTA) revealed confluent coronaryartery fistulas (CAFs) from the right coro-nary artery (RCA) and the left anteriordescending artery meeting to form a12.5-mm aneurysm fistulizing to the pulmo-nary artery (PA) (Figure 1). These findingswere confirmed with invasive coronaryangiography (CAG) (Figures 2A to 2C). Right-sided heart catheterization (RHC) showedsignificant oxygen step-up between the RVand the PA. Oxygen saturation was 70.3%and 80.2%, whereas the mean PA pressure(16 mm Hg) and mean pulmonary capillary

wedge pressure (11 mm Hg) were within normallimits. The ratio of pulmonary blood flow to systemicblood flow (Qp/Qs) was 1.47 on the basis of RHC.

The abnormal tortuous vessels originated proximal to the right

coronary artery and the middle left anterior descending artery

(arrows) with a common giant aneurysm. CRA ¼ cranial;

LAO ¼ left anterior oblique.

MANAGEMENT

Although there were no signs of myocardialischemia or congestive heart failure, transcatheterembolization for CAFs was planned because of thehigh Qp/Qs and the large aneurysm. The initialintervention was for the CAF of the left anteriordescending artery. A 7-F guide catheter (JL-4.0,Heartrail II, Terumo Corp., Tokyo, Japan) wasengaged in the left coronary artery through theright femoral artery. Unfractionated heparin main-tained an activated whole blood clotting time of>250 s. A guidewire (Runthrough NS floppy, Ter-umo Corp.) and a 2.9-F microcatheter (Excelsiorangle, Stryker, Kalamazoo, Michigan) were insertedinto the tortuous vessel. The abnormal vessel wasembolized with 2 detachable coils (Target XL soft,Stryker) up to 9 mm in diameter and 30 cm inlength. Next, embolization of the vessel proximal tothe RCA was performed. Using an AL-2.0 guidecatheter (Heartrail II) and the same microcatheter,the fistula was embolized with 6 detachable coils(Target XL soft 360) up to 7 mm in diameter andup to 80 cm in length (Figures 3A and 3B). Post-embolization CAG confirmed the occlusion of allabnormal vessels (Figures 4A and 4B). Post-embolization RHC revealed Qp/Qs improvement to0.7 and disappearance of the oxygen saturationstep-up (RV, 70.1%; PA, 71.7%).

DISCUSSION

Recently, more CAFs have been incidentally diag-nosed because of developments in CAG and coro-nary CTA quality (1). Most CAFs are congenital,developing in the early embryonic period (2). CAFsare reportedly present in 0.002% of the generalpopulation and represent 0.4% of all cardiac mal-formations (2,3). CAFs are incidentally found in0.13% to 0.22% of adults undergoing CAG (4). Asingle fistula is the most common presentation(70% to 90% of cases); multiple fistulas involvingboth coronary arteries account for 10% to 20% ofcases (1–3). Although some studies have reportedthat RCA-originating CAFs are the most commonpresentation (2,3), Said (1) reported that CAFsoriginating from the left coronary artery were themost common in the decade of the 2000s. CAFsdraining into right-sided heart structures (e.g., PA,right atrium, and RV) are common, whereas casesinvolving left-sided heart structures (e.g., leftventricle, coronary sinus, and superior vena cava)

Page 13: Full Issue PDF · The JACC: Case Reports Editorial Board and ACC editorial office would like to express their sincere gratitude to the guest editors below who worked tirelessly …

FIGURE 3 Coil Embolization

(A) The abnormal vessel originating proximal to the right coronary artery was embolized with 2 detachable coils (arrows), and (B) the vessel

originating at the middle left anterior descending artery was embolized with 6 detachable coils. Abbreviations as in Figure 1.

FIGURE 2 Coronary Angiography

The abnormal tortuous vessels (short arrows) and the (A) right coronary artery (RCA) and (B and C) left anterior descending artery (LAD)

fistula (long arrow). A small window shows the relationship between the right coronary artery and the abnormal vessels. Feeding vessels are

indicated with arrows. Arrowheads indicate the common aneurysm. CAU ¼ caudal; RAO ¼ right anterior oblique; other abbreviations as in

Figure 1.

J A C C : C A S E R E P O R T S , V O L . 2 , N O . 7 , 2 0 2 0 Kubota et al.J U N E 1 7 , 2 0 2 0 : 9 9 1 – 5 Coronary Artery Fistulas with Aneurysm

993

Page 14: Full Issue PDF · The JACC: Case Reports Editorial Board and ACC editorial office would like to express their sincere gratitude to the guest editors below who worked tirelessly …

FIGURE 4 Post-Operative Coronary Angiography

Post-operative coronary angiography confirms the disappearance of all abnormal vessels,

the aneurysm, and fistulas. (A) The vessel originating proximal to the right coronary

artery. (B) The vessel originating at the middle left anterior descending artery (LAD). The

arrow shows coils for the left anterior descending artery branch, and the arrowheads

show coils for the right coronary artery branch. Abbreviations as in Figure 1.

Kubota et al. J A C C : C A S E R E P O R T S , V O L . 2 , N O . 7 , 2 0 2 0

Coronary Artery Fistulas with Aneurysm J U N E 1 7 , 2 0 2 0 : 9 9 1 – 5

994

are rare (1,2). Feeding arteries are primarily ectaticand tortuous. For large fistulas, the most commonsymptoms are dyspnea on exertion caused bycongestion and PA hypertension caused by a shuntto right-sided heart structures. Chest pain from the

CENTRAL ILLUSTRATION Pre- and Post-Coro

Kubota, S. et al. J Am Coll Cardiol Case Rep. 2020;2(7):991–5.

Post-treatment (Post) coronary computed tomography angiography rev

CRA ¼ cranial; LAO ¼ left anterior oblique; Pre ¼ pre-treatment.

blood steal phenomenon from the myocardium isanother common symptom. Several reports haveaddressed infectious endocarditis (1,3). Coronaryaneurysms involving vessels that are 1.5-fold moredilated than normal vessels are reported in 14% ofcases (1).

The appropriate therapeutic indication hasremained controversial. According to currentguidelines, CAFs causing myocardial ischemia andarrhythmia should be treated if they are small ormoderate in size. The size is specified as follows:small, <1� distal reference; medium, 1 to 2� distalreference; and large, >2� distal reference (5).Several studies reported that indications for closureare high Qp/Qs (1.3 to 1.5), the presence of largefistulas, ECG-indicated myocardial ischemia, PAhypertension progression, congestive heart failure,and/or coronary aneurysm with a high risk ofrupture (6).

Transcatheter embolization or closure and sur-gical obliteration effectively treat CAFs. Trans-catheter embolization involving coils, coveredstents, and umbrella devices has increasedrecently because it is less invasive and less riskythan surgical treatment (7,8). Post-treatmentrecurrence is lower following transcatheter embo-lization than following surgical ligation (9% to14% vs. 25%) (9).

nary Computed Tomography Angiography

eals disappearance of the abnormal vessels, fistulas, and aneurysm.

Page 15: Full Issue PDF · The JACC: Case Reports Editorial Board and ACC editorial office would like to express their sincere gratitude to the guest editors below who worked tirelessly …

J A C C : C A S E R E P O R T S , V O L . 2 , N O . 7 , 2 0 2 0 Kubota et al.J U N E 1 7 , 2 0 2 0 : 9 9 1 – 5 Coronary Artery Fistulas with Aneurysm

995

Although our patient had no symptoms suggestiveof myocardial ischemia, we performed transcatheterembolization because of the high Qp/Qs and the giantaneurysm. There are no definite indications fortreating coronary artery aneurysms, but some reportsdescribe a risk of rupture for aneurysms 3-fold moredilated than normal vessels (10).

Transcatheter embolization is indicated when theCAF is not severely tortuous and its distal site isaccessible and able to be embolized by a closuredevice, as in this case. The patient avoided allsurgery-related complications, including surgicalstress, infections, wound healing problems,bleeding, general anesthesia adverse events, andthe higher possibility of recurrence.

To our knowledge, there are no available dataprognosis for untreated CAFs or aneurysms. Howev-er, transcatheter embolization, which is safe and hasa low risk, may improve prognosis.

FOLLOW-UP

A 3-month follow-up coronary CTA confirmed disap-pearance of the abnormal arteries and fistulas (CentralIllustration). The patient has had no post-treatmentsymptoms or complications.

CONCLUSIONS

Coronary fistulas are relatively rare and may rupturein patients with aneurysm. Transcatheter treatmentshould be considered because it is safer and lessinvasive than other methods.

ADDRESS FOR CORRESPONDENCE: Dr. Hisao Hara,Department of Cardiology, National Center forGlobal Health and Medicine, 1-21-1 Toyama,Shinjyuku-ku, Tokyo 162-8655, Japan. E-mail:[email protected].

RE F E RENCE S

1. Said SA. Current characteristics of congen-ital coronary artery fistulas in adults: Adecade of global experience. World J Cardiol2011;3:267-7.

2. Mangukia CV. Coronary artery fistula. AnnThorac Surg 2012;93:2084–92.

3. Challoumas D, Pericleous A, Dimitrakaki IA,Danelatos C, Dimitrakakis G. Coronary arteriove-nous fistulae: a review. Int J Angiol 2014;23:1–10.

4. Latson LA. Coronary artery fistulas: how tomanage them. Catheter Cardiovasc Interv 2007;70:110–6.

5. Warnes CA, Williams RG, Bashore TM, et al.ACC/AHA 2008 guidelines for the management ofadults with congenital heart disease: a report ofthe American College of Cardiology/American

Heart Association Task Force on Practice Guide-lines (Writing Committee to Develop Guidelines onthe Management of Adults With Congenital HeartDisease). Developed in Collaboration With theAmerican Society of Echocardiography, HeartRhythm Society, International Society for AdultCongenital Heart Disease, Society for Cardiovas-cular Angiography and Interventions, and Societyof Thoracic Surgeons. J Am Coll Cardiol 2008;52:e143–263.

6. Tanabe Y, Tejima T, Sakurada H. Coronary arteryfistula with a giant aneurysm treated by percuta-neous transradial embolization. Catheter Car-diovasc Interv 2013;81. 732–7.

7. Sakai M, Mori K, Hoshiai S, et al. Successfultransarterial embolization of coronary artery fis-tula with ruptured aneurysm: a case report. RadiolCase Rep 2018;14:126–8.

8. Armsby LR, Keane JF, Sherwood MC,Forbess JM, Perry SB, Lock JE. Management ofcoronary artery fistulae. Patient selection and re-sults of transcatheter closure. J Am Coll Cardiol2002;39:1026–32.

9. Said SA, Nijhuis RL, Op den Akker JW, et al.Diagnostic and therapeutic approach of congenitalsolitary coronary artery fistulas in adults: Dutchcase series and review of literature. Neth Heart J2011;19:183–91.

10. Akashi H, Tayama E, Tayama K, et al. Ruptureof an aneurysm resulting from a coronary arteryfistula: a case report. Circ J 2003;67:551–3.

KEY WORDS coronary vessel anomaly,imaging, percutaneous coronaryintervention, treatment

Page 16: Full Issue PDF · The JACC: Case Reports Editorial Board and ACC editorial office would like to express their sincere gratitude to the guest editors below who worked tirelessly …

J A C C : C A S E R E P O R T S V O L . 2 , N O . 7 , 2 0 2 0

ª 2 0 2 0 T H E A U T H O R S . P U B L I S H E D B Y E L S E V I E R O N B E H A L F O F T H E AM E R I C A N

C O L L E G E O F C A R D I O L O G Y F O U N DA T I O N . T H I S I S A N O P E N A C C E S S A R T I C L E U N D E R

T H E C C B Y - N C - N D L I C E N S E ( h t t p : / / c r e a t i v e c o mm o n s . o r g / l i c e n s e s / b y - n c - n d / 4 . 0 / ) .

CASE REPORT

CLINICAL CASE

Massive Myocardial Calcium Deposition

Hardened Heart

Mark N. Belkin, MD,a,* Mark Dela Cruz, MD,a,* Urooba Nadeem, MD,b Amit R. Patel, MD,a Gene Kim, MD,a

Jonathan Grinstein, MDa

ABSTRACT

L

ISS

Fro

Ch

tha

Th

sti

the

Ma

A 25-year-old African-American woman with end-stage renal disease presented with new-onset heart failure.

Transthoracic echocardiography indicated a significantly hyperechoic myocardium, and computed tomography noted

a circumferential hyperattenuated myocardium. Endomyocardial biopsy revealed focal interstitial and

intramyocyte calcium deposition in the heart, confirming a rare diagnosis of massive myocardial calcium

deposition. (Level of Difficulty: Beginner.) (J Am Coll Cardiol Case Rep 2020;2:996–1003) © 2020 The Authors.

Published by Elsevier on behalf of the American College of Cardiology Foundation. This is an open access article

under the CC BY-NC-ND license (http://creativecommons.org/licenses/by-nc-nd/4.0/).

HISTORY OF PRESENT ILLNESS

A 25-year-old African-American woman presentedwith chest pain during hemodialysis. She was diag-nosed with new-onset heart failure (HF) complicatedby respiratory failure. Her physical examination wasnotable for respiratory distress, fine bibasilarcrackles, and jugular vein distension. She was alsofound to have scattered atypical, full-thickness,painful dermal lesions.

EARNING OBJECTIVES

HF secondary to massive myocardial calcifi-cation is a rare but significant etiology.To expand the differential diagnosis of new-onset HF in patients with chronic kidneydisease.To highlight the role of calcium-phosphorusmetabolism in the development of acardiomyopathy.

N 2666-0849

m the aSection of Cardiology, University of Chicago, Chicago, Illinois; and

icago, Illinois. *Drs. Belkin and Dela Cruz contributed equally to this work

t they have no relationships relevant to the contents of this paper to dis

e authors attest they are in compliance with human studies committees

tutions and Food and Drug Administration guidelines, including patient co

JACC: Case Reports author instructions page.

nuscript received January 31, 2020; revised manuscript received Februar

MEDICAL HISTORY

The patient had a medical history of systemic lupuserythematosus on long-term hydroxychloroquine andend-stage renal disease (ESRD) on hemodialysis forapproximately 4 years.

DIFFERENTIAL DIAGNOSIS

The patient presented with new-onset HF of unclearetiology. In addition to more typical etiologies of HFin a young patient (e.g., viral, familial, and idiopathiccardiomyopathies), a unique differential diagnosisincluding systemic lupus erythematosus myocarditisand hydroxychloroquine toxicity was includedbecause of the patient’s reported history.

INVESTIGATIONS

Electrocardiography revealed sinus tachycardia,right-axis deviation, and poor R-wave progression(Figure 1). Transthoracic echocardiogram indicated a

https://doi.org/10.1016/j.jaccas.2020.03.034

the bDepartment of Pathology, University of Chicago,

and are joint first authors. The authors have reported

close.

and animal welfare regulations of the authors’ in-

nsent where appropriate. For more information, visit

y 27, 2020, accepted March 1, 2020.

Page 17: Full Issue PDF · The JACC: Case Reports Editorial Board and ACC editorial office would like to express their sincere gratitude to the guest editors below who worked tirelessly …

AB BR E V I A T I O N S

AND ACRONYM S

CT = computed tomographic

EMB = endomyocardial biopsy

ESRD = end-stage renal

disease

HF = heart failure

J A C C : C A S E R E P O R T S , V O L . 2 , N O . 7 , 2 0 2 0 Belkin et al.J U N E 1 7 , 2 0 2 0 : 9 9 6 – 1 0 0 3 Massive Myocardial Calcium Deposition

997

significantly hyperechoic myocardium with mild,concentric left ventricular hypertrophy, and an ejec-tion fraction of 25% (Videos 1A, 1B, and 1C). Pulsed-wave Doppler through the mitral valve revealedfusion of the E and A waves. Although this E- and A-wave fusion limited accurate calculation of theE-wave deceleration time, this metric appearedqualitatively shortened, consistent with restrictivephysiology (Figure 2A). However, tissue Doppler ofboth the medial and lateral mitral valve annulirevealed elevated e0 values, which would not beconsistent with restriction, as it is normally associ-ated with reduced e0 velocities (Figures 2B and 2C).Computed tomographic (CT) imaging of the chestrevealed circumferential hyperattenuated myocar-dium in the left ventricle, discrete areas of the rightventricular free wall, and both atria concerning forcalcium deposition (Figures 3A to 3C).

Right heart catheterization exhibited elevatedbiventricular pressures, equalization of diastolicpressures, and preserved cardiac output consistentwith a restrictive physiology (Figure 4). Coronaryangiography did not show any significant coronaryartery disease. Serum was notable for calcium at9.0 mg/dl (normal range: 8.4 to 10.2 mg/dl), phos-phorus at 6.6 mg/dl (normal range: 2.5 to 4.4 mg/dl),and a parathyroid hormone level of 3,043 pg/ml(normal range: 15 to 75 pg/ml). A native heart endo-myocardial biopsy (EMB) showed focal interstitial andintracellular calcium deposition with mild myocytehypertrophy (Figure 5). Additionally, Von Kossa

FIGURE 1 Electrocardiogram

staining was used to highlight the calciumdeposits (Figure 6). Finally, there was nohistological evidence of vacuolated myocyteson hematoxylin and eosin staining or pres-ence of cytoplasmic deposits, called “myeloidbodies,” within the cardiac myocytes onelectron microscopy, both of which are spe-cific to hydroxychloroquine toxicity (1). Bi-

opsy of the dermal lesions noted superficial to deepperivascular and periadnexal lymphoplasmacyticinfiltrate, with a fibrotic papillary dermis. The biopsywas not consistent with calciphylaxis.

Given the findings of intracellular calcium deposi-tion found on CT imaging and EMB, the patient wasdiagnosed with restrictive cardiomyopathy due tomassive myocardial calcium deposition.

MANAGEMENT

Although parathyroidectomy was indicated, the pa-tient was not hemodynamically stable for surgery.Cinacalcet 30 mg/day was started to lower circulatingphosphate level. Three doses of sodium thiosulfate12.5 mg, a calcium chelating agent increasingly usedin the treatment of calciphylaxis to promote calciumexcretion, were given following hemodialysis ses-sions while dermal biopsy results were pending andcalciphylaxis was ruled out, after which it wasstopped (2). Throughout the hospital course, the pa-tient had a pulmonary arterial catheter for invasivehemodynamic monitoring. Her hemodynamic status

Page 18: Full Issue PDF · The JACC: Case Reports Editorial Board and ACC editorial office would like to express their sincere gratitude to the guest editors below who worked tirelessly …

FIGURE 2 Echocardiogram

(A) Pulsed-wave Doppler through the mitral valve. (B) Tissue Doppler of the medial mitral valve annulus. (C) Tissue Doppler of the lateral mitral valve annulus.

DT ¼ E-wave deceleration time; E ¼ E wave; e0 ¼ e0 mitral velocity.

Belkin et al. J A C C : C A S E R E P O R T S , V O L . 2 , N O . 7 , 2 0 2 0

Massive Myocardial Calcium Deposition J U N E 1 7 , 2 0 2 0 : 9 9 6 – 1 0 0 3

998

was difficult to manage because of the restrictivecardiac physiology and poor biventricular contrac-tility secondary to the myocardial calcification, aswell as low systemic vascular resistance due to sepsis.Norepinephrine was the predominant vasoactivemedication used. Volume status was managed bycontinuous hemodialysis.

DISCUSSION

There have been only a few case reports of myocardialcalcification, which has been associated with HF, ar-rhythmias, and sudden cardiac death (3–6).

Myocardial calcification can be either dystrophic,occurring in infarcted, nonviable tissue, or metasta-tic, due to abnormalities in calcium-phosphorusmetabolism, as well as elevated parathyroid hor-mone (3,7). This patient had severe derangements inher calcium-phosphorus metabolism, consistent withsecondary hyperparathyroidism.

Secondary hyperparathyroidism results from de-rangements in calcium-phosphorus metabolism inthe setting of chronic kidney disease. In chronic kid-ney disease, reduced excretion of phosphate leads toelevated serum phosphate levels, resulting inreduced renal activation of vitamin D and subsequent

Page 19: Full Issue PDF · The JACC: Case Reports Editorial Board and ACC editorial office would like to express their sincere gratitude to the guest editors below who worked tirelessly …

FIGURE 3 Computed Tomography Imaging of the Chest

(A) Computed tomographic (CT) imaging of the chest. Arrows denote calcified myocardium. (B) Contrast-enhanced CT imaging of the chest with multiplanar refor-

matting in the 4-chamber view. (C) Contrast-enhanced CT imaging of the chest with multiplanar reformatting in the short-axis view. A ¼ calcified myocardium;

B ¼ noncalcified endocardium; LA ¼ left atrium; LV ¼ left ventricle; RA ¼ right atrium; RV ¼ right ventricle.

J A C C : C A S E R E P O R T S , V O L . 2 , N O . 7 , 2 0 2 0 Belkin et al.J U N E 1 7 , 2 0 2 0 : 9 9 6 – 1 0 0 3 Massive Myocardial Calcium Deposition

999

gastrointestinal malabsorption of calcium. Reducedcalcium levels trigger increased parathyroid hormonerelease, leading to increased calcium release from thebones, renal calcium reabsorption, and intestinalmucosal cell calcium absorption (Figure 7) (8).

The exact mechanisms for massive myocardialcalcification due to secondary hyperparathyroidismare incompletely understood. Calcium deposition incardiac soft tissues in the form of mitral annular oraortic valve calcification has long been described inpatients with ESRD. This type of soft tissue

deposition has been attributed to derangements incalcium and phosphate metabolism as well as thecoalescing of focal calcific deposits in regions ofmicroinjury and lipoprotein accumulation over time(9). However, the myocardium is presumably notsubject to the same degree of microinjury as valvularsoft tissues. Therefore, we suspect that myocardialcalcium deposition likely results from a uniquemechanism.

The cycling and storage of calcium are essential tomyocardial excitation and contraction, in addition to

Page 20: Full Issue PDF · The JACC: Case Reports Editorial Board and ACC editorial office would like to express their sincere gratitude to the guest editors below who worked tirelessly …

FIGURE 4 Right Heart Catheterization Tracings

PA ¼ pulmonary artery pressure; PCWP ¼ pulmonary capillary wedge pressure; RA ¼ right atrial pressure; RV ¼ right ventricular pressure.

Belkin et al. J A C C : C A S E R E P O R T S , V O L . 2 , N O . 7 , 2 0 2 0

Massive Myocardial Calcium Deposition J U N E 1 7 , 2 0 2 0 : 9 9 6 – 1 0 0 3

1000

playing essential roles in electrophysiologicalsignaling, mitochondrial function, cell death, andtranscription. Calcium is stored primarily in thesarcoplasmic reticulum, bound to the storage proteincalsequestrin. During depolarization, calcium entersthe cytosol through calcium channels, triggering therelease of calcium from the sarcoplasmic reticulum.Increased cytosol levels of calcium result in calciumbinding to troponin C, which initiates myocytecontraction. During diastole, the majority of cytosoliccalcium is rapidly removed by an adenosinetriphosphate–regulated calcium transporter and againstored within the sarcoplasmic reticulum. A smallerpercentage of this calcium is also excreted from thecell via sodium and adenosine triphosphate–regulated exchangers. The resulting decrease inintracellular calcium allows dissociation of calciumfrom troponin C, causing myocardial relaxation (10).

It is possible that massive myocardial calcificationresults from derangements not only in systemiccalcium-phosphorus metabolism but in the calciumcycling and storage system itself. Although mutationsthat result in reduced calcium release or in increased

calcium uptake have previously been described, theyhave not been associated with myocardial calciumdeposition. Perhaps such mutations, in the setting ofESRD and secondary hyperparathyroidism, result inan altered milieu which allows massive myocardialcalcium deposition. Such a mechanism could helpexplain how massive myocardial calcium depositionremains a rare entity despite the wide prevalence ofsecondary hyperparathyroidism among patients withESRD (8).

In this case, diagnosis of massive myocardial cal-cium deposition relied heavily on noninvasive imag-ing. Transthoracic echocardiography revealed anunusually hyperechoic myocardium and a depressedejection fraction. CT evaluation confirmed sub-endocardial, circumferential hyperattenuation,which is consistent with calcium deposition in thesetting of severe secondary hyperparathyroidism.

This degree of myocardial calcification in a youngpatient after only 4 years of hemodialysis has notpreviously been reported. Only 3 prior case reportshave reported pathology confirming myocardialcalcification, but 2 were completed postmortem,

Page 21: Full Issue PDF · The JACC: Case Reports Editorial Board and ACC editorial office would like to express their sincere gratitude to the guest editors below who worked tirelessly …

FIGURE 5 Endomyocardial Biopsy With Routine Stain

Arrows denote intracellular calcium deposits.

J A C C : C A S E R E P O R T S , V O L . 2 , N O . 7 , 2 0 2 0 Belkin et al.J U N E 1 7 , 2 0 2 0 : 9 9 6 – 1 0 0 3 Massive Myocardial Calcium Deposition

1001

and 1 was completed on an explanted heartfollowing orthotopic heart transplantation (3,5,6).This is the first case reported of EMB completed in aliving patient to confirm the diagnosis of massivemyocardial calcification. It is worth noting that theEMB showed minimal calcification likely as a resultof the endocardial sparing noted on imaging(Figures 3B and 3C). The calcification on EMB wasfound both in the interstitium and within the car-diac myocytes. Unfortunately, examination withelectron microscopy was unable to characterize thelocalization of the calcium deposits further. Furtherexamination was limited because the familydeclined autopsy.

Routine diagnostic evaluation for myocardial cal-cium deposition is not currently indicated in theESRD population, but these cases have reported di-agnoses on the basis of chest radiography, chest CTimaging, fluoroscopy, and echocardiography (4,6). In

all reported cases, patients were diagnosed post-mortem, were too critically ill to receive treatment,or required orthotopic heart transplantation foradvanced-stage cardiomyopathy (3–6). Therefore, nospecific treatments have been published for massivemyocardial calcification.

FOLLOW-UP

The patient continued to decompensate over the3 weeks following diagnosis from a combination ofcardiogenic and septic shock, in the setting of fun-gemia, due to Candida auris and Candida glabrata,as well as extended spectrum beta-lactamaseEscherichia coli bacteremia. The patient was not acandidate for advanced cardiac therapies because ofher active, resistant infections and multiorgan sys-tem failure, from which she died during thehospitalization.

Page 22: Full Issue PDF · The JACC: Case Reports Editorial Board and ACC editorial office would like to express their sincere gratitude to the guest editors below who worked tirelessly …

FIGURE 7 Pathophysiology of Secondary Hyperparathyroidism

CKD ¼ chronic kidney disease; GI ¼ gastrointestinal; PTH ¼ parathyroid hormone.

FIGURE 6 Endomyocardial Biopsy With Von Kossa Stain

Arrows denote intracellular calcium deposits.

Belkin et al. J A C C : C A S E R E P O R T S , V O L . 2 , N O . 7 , 2 0 2 0

Massive Myocardial Calcium Deposition J U N E 1 7 , 2 0 2 0 : 9 9 6 – 1 0 0 3

1002

Page 23: Full Issue PDF · The JACC: Case Reports Editorial Board and ACC editorial office would like to express their sincere gratitude to the guest editors below who worked tirelessly …

J A C C : C A S E R E P O R T S , V O L . 2 , N O . 7 , 2 0 2 0 Belkin et al.J U N E 1 7 , 2 0 2 0 : 9 9 6 – 1 0 0 3 Massive Myocardial Calcium Deposition

1003

CONCLUSIONS

Massive myocardial calcification is a rare, seriouscomplication of abnormal calcium-phosphorus meta-bolism in patients with ESRD.

ADDRESS FOR CORRESPONDENCE: Dr. JonathanGrinstein, University of Chicago, 5841 South Mary-land Avenue, A621, Chicago, Illinois 60637. E-mail:[email protected].

RE F E RENCE S

1. Joyce E, Fabre A, Mahon N. Hydroxychloroquinecardiotoxicity presenting as a rapidly evolvingbiventricular cardiomyopathy: key diagnostic fea-tures and literature review. Eur Heart J AcuteCardiovasc Care 2013;2:77–83.

2. Yu Z, Gu L, Pang H, Fang Y, Yan H, Fang W.Sodium thiosulfate: an emerging treatment forcalciphylaxis in dialysis patients. Case Rep NephrolDial 2015;5:77–82.

3. Na JY. A heart of stone: an autopsy case ofmassive myocardial calcification. Forensic Sci MedPathol 2018;14:102–5.

4. Matsui M, Okayama S, Takitsume A, et al. Heartfailure associated with metastatic myocardialcalcification in a hemodialysis patient with pro-gressive calcification of the hand. Cardiorenal Med2012;2:251–5.

5. Isotalo PA, Halil A, Green M, Tang A, Lach B,Veinot JP. Metastatic calcification of thecardiac conduction system with heart block: anunder-reported entity in chronic renalfailure patients. J Forensic Sci 2000;45:1335–8.

6. Shackley BS, Nguyen TP, Shivkumar K,Finn PJ, Fishbein MC. Idiopathic massivemyocardial calcification: a case report and re-view of the literature. Cardiovasc Pathol 2011;20:e79–83.

7. Thompson AR, Fallon J, Nussbaum S. Evaluationof metastatic cardiac calcification in a model ofchronic primary hyperparathyroidism. Surgery1990;108:1047–51.

8. Cunningham J, Locatelli F, Rodriguez M. Sec-ondary hyperparathyroidism: pathogenesis,

disease progression, and therapeutic options. ClinJ Am Soc Nephrol 2011;6:913–21.

9. Massera D, Kizer JR, Dweck MR. Mechanisms ofmitral annular calcification. Trends CardiovascMed 2020. In press.

10. Bers DM. Calcium cycling and signaling incardiac myocytes. Annu Rev Physiol 2008;70:23–49.

KEY WORDS cardiomyopathy, disorders ofcalcium metabolism, restrictive

APPENDIX For supplemental videos,please see the online version of this paper.

Page 24: Full Issue PDF · The JACC: Case Reports Editorial Board and ACC editorial office would like to express their sincere gratitude to the guest editors below who worked tirelessly …

J A C C : C A S E R E P O R T S V O L . 2 , N O . 7 , 2 0 2 0

ª 2 0 2 0 T H E A U T H O R S . P U B L I S H E D B Y E L S E V I E R O N B E H A L F O F T H E AM E R I C A N

C O L L E G E O F C A R D I O L O G Y F O U N DA T I O N . T H I S I S A N O P E N A C C E S S A R T I C L E U N D E R

T H E C C B Y - N C - N D L I C E N S E ( h t t p : / / c r e a t i v e c o mm o n s . o r g / l i c e n s e s / b y - n c - n d / 4 . 0 / ) .

CASE REPORT

CLINICAL CASE

Left Bundle Branch Pacing inTransthyretin Cardiac Amyloidosis andAlternating Bundle Branch Block

Taha Ahmed, MD, Samra Haroon Lodhi, MBBS, Hassan Lak, MD, Jae Lee, MD, Mazen Hanna, MD, Roy Chung, MD

ABSTRACT

L

ISS

Fro

Oh

co

Th

ins

vis

Ma

Progressive conduction system disease affects patients with transthyretin cardiac amyloidosis, often requiring permanent

pacing as the His-Purkinje system is affected. We present a case of left bundle branch pacing in a patient with trans-

thyretin cardiac amyloidosis and infra-Hisian conduction disease with a favorable improvement in cardiac function and

stable pacing thresholds. (Level of Difficulty: Intermediate.) (J Am Coll Cardiol Case Rep 2020;2:1004–8) © 2020 The

Authors. Published by Elsevier on behalf of the American College of Cardiology Foundation. This is an open access article

under the CC BY-NC-ND license (http://creativecommons.org/licenses/by-nc-nd/4.0/).

P acing options in patients with transthyretincardiac amyloidosis (ATTR-CA) affecting theconduction system are limited, as criteria for

biventricular pacing are often not met, and right-ventricular (RV) pacing-induced dyssynchronyfurther reduces the cardiac output. Left bundle

EARNING OBJECTIVES

To appreciate that left bundle branch pacingis a physiological pacing option to minimizepacing-induced dyssynchrony and worseningsymptoms in patients with heart failure sec-ondary to infiltrative cardiomyopathiescompared with traditional pacing modalities.To recognize that, in transthyretin cardiacamyloidosis affecting the cardiac conductionsystem, no well-defined pacing option hasbeen validated. LBBP appears to be prom-ising in patients with ATTR-CA andconduction-system disease.

N 2666-0849

m the Section of Cardiac Electrophysiology and Pacing, Department of C

io. Dr. Chung serves as consultant to Medtronic. All other authors have rep

ntents of this paper to disclose.

e authors attest they are in compliance with human studies committe

titutions and Food and Drug Administration guidelines, including patien

it the JACC: Case Reports author instructions page.

nuscript received February 18, 2020; revised manuscript received April 3

branch pacing (LBBP) can be achieved by recruitingthe left fascicular system, thus reducing the risk ofventricular dyssynchrony. This is performed byfixating a pacing lead through the ventricular septum,which has demonstrated a lower pacing threshold,larger R-wave amplitude and shorter paced QRS dura-tion (QRSD) compared with RV pacing and His bundlepacing (1). This report presents the application ofLBBP in a patient with ATTR-CA, New York Heart As-sociation (NYHA) functional class III heart failuresymptoms, and infra-Hisian conduction system dis-ease. At 3- and 6-month follow-up visit, the patientdemonstrated significant improvement in his heartfailure symptoms and functional capacity.

HISTORY OF PRESENTATION

A 75-year old man presented to our office forevaluation of pacemaker implantation in the settingof chronotropic incompetence on recent cardiopul-monary exercise testing. The patient complained of

https://doi.org/10.1016/j.jaccas.2020.04.040

ardiovascular Medicine, Cleveland Clinic, Cleveland,

orted that they have no relationships relevant to the

es and animal welfare regulations of the authors’

t consent where appropriate. For more information,

, 2020, accepted April 13, 2020.

Page 25: Full Issue PDF · The JACC: Case Reports Editorial Board and ACC editorial office would like to express their sincere gratitude to the guest editors below who worked tirelessly …

AB BR E V I A T I O N S

AND ACRONYMS

AF = atrial fibrillation

ATTR-CA = transthyretin cardiac

amyloidosis

GLS = global longitudinal strain

H/CL = heart/contralateral lung

IVS = interventricular septum

LBBB = left bundle branch block

LBBP = left bundle branch pacing

LV = left ventricle

NYHA = New York Heart Association

QRSD = QRS duration

RBBB = right bundle branch block

TTE = transthoracic echocardiogram

J A C C : C A S E R E P O R T S , V O L . 2 , N O . 7 , 2 0 2 0 Ahmed et al.J U N E 1 7 , 2 0 2 0 : 1 0 0 4 – 8 Left Bundle Branch Pacing in Cardiac Amyloidosis

1005

progressively worsening fatigue and dyspnea for 1year, with lower-extremity edema. Physical exami-nation demonstrated a jugular venous pressure of16 cm of H2O, irregularly irregular rhythm, andperipheral edema.

PAST MEDICAL HISTORY

The patient’s medical history including for bilateralcarpal tunnel syndrome and persistent atrial fibrilla-tion (AF). The patient’s previous electrocardiogram(ECG) 1 year ago showed chronic AF with a left bundlebranch block (LBBB) pattern and a QRSD of 120 ms(Figure 1B).

Previous transthoracic echocardiogram (TTE)showed biatrial dilation with severe concentric left

FIGURE 1 Patient’s Intrinsic Rhythm

(A) Patient’s normal intrinsic rhythm 10 years ago. (B) Electrocardiograph

1 year ago. (C) Electrocardiographic tracing on current visit with right b

(D) Left bundle paced rhythm with a QRS duration of 110 ms.

ventricle (LV) hypertrophy, a small LVcavity with an ejection fraction (LVEF) of55%, and a LV mass index of 138 g/m2.

DIFFERENTIAL DIAGNOSIS

Differential diagnosis included amyloidcardiomyopathy, sarcoid cardiomyopa-thy, restrictive cardiomyopathy, andidiopathic cardiomyopathy.

INVESTIGATIONS

The ECG was significant for AF withcomplete right bundle branch block(RBBB) with a QRSD of 140 ms

ic tracing with left bundle branch block and QRS duration of 120 ms

undle branch block morphology with a QRS duration of 140 ms.

Page 26: Full Issue PDF · The JACC: Case Reports Editorial Board and ACC editorial office would like to express their sincere gratitude to the guest editors below who worked tirelessly …

FIGURE 2 Off-Axis Apical 4-Chamber View Demonstrating Pacing Lead Slightly Past the Tricuspid Annulus Implanted Through the

Ventricular Septum

Red arrow indicates site of implantation.

Ahmed et al. J A C C : C A S E R E P O R T S , V O L . 2 , N O . 7 , 2 0 2 0

Left Bundle Branch Pacing in Cardiac Amyloidosis J U N E 1 7 , 2 0 2 0 : 1 0 0 4 – 8

1006

(Figure 1C). B-type natriuretic peptide (NT-proBNP)concentration was 5,150 pg/ml. Repeat TTE showedconcentric LV hypertrophy, with an interventricularseptum (IVS) thickness of 1.5 cm and global longi-tudinal strain (GLS) of –7.1%, with abnormal LVmidcavitary and basal strain pattern with apicalsparing, suggesting an underlying infiltrativedisease.

Technetium pyrophosphate scan showed diffuseGrade 3 uptake in the LV with a heart/contralaterallung (H/CL) ratio of 1.6. Serum-free light chains,immunofixation, and ATTR genetic testing resultswere negative, consistent with the diagnosis of wild-type nonhereditary form of ATTR-CA.

MANAGEMENT

The patient was initiated on tafamidis. Given thealternating bundle branch block and infiltrativenature of this disease, the patient met an indicationfor a permanent pacemaker. A SelectSecure(model 3830, Medtronic Inc., Minneapolis, Minne-sota) lead was actively fixated to the ventricularseptum, with recruitment of the left fascicular sys-tem via a trans-septal approach (Figure 2). The stim-R time (in V1) was 90 ms and the final paced QRSDwas 105 ms (Figures 1D and 3). Chest radiography

confirms a nonapical septal lead placement(Figure 4).

DISCUSSION

The concept of LBBP has developed in the last fewyears and is proposed to benefit patients with His-Purkinje conduction disease or LBBB (2,3). It is anexcellent option, as difficulties with higher pacingthresholds are encountered with traditional Hisbundle pacing (4). LBBP can bypass the pathologicalregion in the cardiac conduction system to produce anear physiological or true conduction system pacingfor patients in need for ventricular pacing for heartfailure (5).

There are limited data investigating various pac-ing options in patients with ATTR-CA. Donnellanet al. (6) studied the outcomes of biventricularpacing in patients with ATTR-CA and found thatdyssynchrony from RV pacing in an infiltrated smallventricle would have deleterious impact on the LVfunction, NYHA functional class, and mortality.Biventricular pacing has been proposed as an optionin ATTR-CA when indicated, but most of these pa-tients do not meet standard criteria for biventricularpacing. This pacing modality is essentially non-physiological; thus, the paced QRSD remains

Page 27: Full Issue PDF · The JACC: Case Reports Editorial Board and ACC editorial office would like to express their sincere gratitude to the guest editors below who worked tirelessly …

FIGURE 3 Procedural Electrocardiogram With LVAT of 90 ms, With Preceding QRS Demonstrating Left Bundle Branch Block

LVAT ¼ left ventricular activation time.

J A C C : C A S E R E P O R T S , V O L . 2 , N O . 7 , 2 0 2 0 Ahmed et al.J U N E 1 7 , 2 0 2 0 : 1 0 0 4 – 8 Left Bundle Branch Pacing in Cardiac Amyloidosis

1007

Page 28: Full Issue PDF · The JACC: Case Reports Editorial Board and ACC editorial office would like to express their sincere gratitude to the guest editors below who worked tirelessly …

FIGURE 4 Chest Radiograph Demonstrating Septal Lead Placement

Ahmed et al. J A C C : C A S E R E P O R T S , V O L . 2 , N O . 7 , 2 0 2 0

Left Bundle Branch Pacing in Cardiac Amyloidosis J U N E 1 7 , 2 0 2 0 : 1 0 0 4 – 8

1008

prolonged in patients with underlying LBBB. LBBPappears to be a promising pacing modality in thiscohort, but further studies on a larger scale areneeded to determine it.

OUTCOME AND FOLLOW-UP

At 3-month follow-up, the patient’s NYHA func-tional class improved from III to II and NT-proBNPlevel decreasing from 5,150 to 3,890 pg/ml. TheECG showed a paced QRSD of 105 ms, and anechocardiogram showed improved GLS of –9.0% andthe pacing lead in the IVS. The pacing thresholds

remained stable at 0.5 V on 3- and 6-month follow-up visits.

CONCLUSIONS

This case demonstrates that LBBP is a promisingphysiological pacing option for ATTR-CA in thesetting of infra-Hisian disease, with improvement ofcardiac synchronization and heart failure symptoms.

ADDRESS FOR CORRESPONDENCE: Dr. Roy Chung,Heart and Vascular Institute, Cleveland Clinic, 9500Euclid Avenue, Cleveland, Ohio 44195. E-mail:[email protected].

RE F E RENCE S

1. Li Y, Chen K, Dai Y, et al. Left bundle branchpacing for symptomatic bradycardia: implant suc-cess rate, safety, and pacing characteristics. HeartRhythm 2019;16:1758–65.

2. Huang W, Su L, Wu S, et al. A novel pacingstrategy with low and stable output: pacing theleft bundle branch immediately beyond the con-duction block. Can J Cardiol 2017;33:1736.e1–3.3. Vijayaraman P, Huang W. Atrioventricular blockat the distal His bundle: electrophysiological

insights from left bundle branch pacing. HeartRhythm Case Rep 2019;5:233–6.

4. Huang W, Chen X, Su L, Wu S, Xia X,Vijayaraman P. A beginner’s guide to permanentleft bundle branch pacing. Heart Rhythm 2019;16:1791–6.

5. Zhang S, Zhou X, Gold MR. Left bundlebranch pacing. J Am Coll Cardiol 2019;74:3039–49.

6. Donnellan E, Wazni OM, Saliba WI, et al.Cardiac devices in patients with transthyretinamyloidosis: impact on functional class, leftventricular function, mitral regurgitation andmortality. J Cardiovasc Electrophysiol 2019;30:2427–32.

KEY WORDS cardiac pacemaker,cardiomyopathy

Page 29: Full Issue PDF · The JACC: Case Reports Editorial Board and ACC editorial office would like to express their sincere gratitude to the guest editors below who worked tirelessly …

J A C C : C A S E R E P O R T S VO L . 2 , N O . 7 , 2 0 2 0

ª 2 0 2 0 T H E A U T H O R S . P U B L I S H E D B Y E L S E V I E R O N B E H A L F O F T H E A M E R I C A N

C O L L E G E O F C A R D I O L O G Y F OU N D A T I O N . T H I S I S A N O P E N A C C E S S A R T I C L E U N D E R

T H E C C B Y - N C - N D L I C E N S E ( h t t p : / / c r e a t i v e c o mm o n s . o r g / l i c e n s e s / b y - n c - n d / 4 . 0 / ) .

CASE REPORT

CLINICAL CASE

An Overlap Presentation of PericardialDecompression Syndrome andStress Cardiomyopathy FollowingTherapeutic Pericardiocentesis

Roberto C. Cerrud-Rodriguez, MD, Syed Muhammad Ibrahim Rashid, MD, Hussein Shaqra, MD, Ahmad Alkhalil, MD,Mohammed Algodi, MD, Jay J. Chudow, MD, Mario J. Garcia, MD, James M. Tauras, MD, Giora Weisz, MD

ABSTRACT

L

ISS

Fro

Bro

Th

Th

sti

the

Ma

Pericardial decompression syndrome, defined as paradoxical hypotension and pulmonary edema after pericardiocentesis,

is a rare complication of pericardiocentesis. Stress cardiomyopathy, caused by excess catecholamine response resulting in

left ventricular dysfunction and elevated cardiac enzymes, can overlap with pericardial decompression syndrome, and

both might belong to the same spectrum of disease. (Level of Difficulty: Intermediate.) (J Am Coll Cardiol Case Rep

2020;2:1009–13) © 2020 The Authors. Published by Elsevier on behalf of the American College of Cardiology Foundation.

This is an open access article under the CC BY-NC-ND license (http://creativecommons.org/licenses/by-nc-nd/4.0/).

HISTORY OF PRESENTATION

A 70-year-old man presented with 2 weeks ofworsening dyspnea on exertion. On physical ex-amination, the patient was not in any acute

EARNING OBJECTIVES

To recognize paradoxical hemodynamicinstability after pericardiocentesis as a po-tential complication of the procedure, aswell as its differential diagnosis.To distinguish the possible mechanisms ofparadoxical hemodynamic instability afterpericardiocentesis, including the infrequentPDS.To investigate the potential overlap be-tween PDS and SCM given potentially similarcausative mechanisms.

N 2666-0849

m the Montefiore-Einstein Center for Heart and Vascular Care, Montefiore

nx, New York. The authors have reported that they have no relationships r

anman, MD, served as Guest Editor for this paper.

e authors attest they are in compliance with human studies committees

tutions and Food and Drug Administration guidelines, including patient co

JACC: Case Reports author instructions page.

nuscript received March 31, 2020; accepted April 3, 2020.

distress. He had regular tachycardia and decreasedheart sounds. Jugular venous distention and bilat-eral 2þ lower extremity edema were present.Electrocardiography revealed sinus tachycardia (104beats/min) with low voltage and no electric alter-nans or ischemic changes. Chest computed tomog-raphy without contrast demonstrated largepericardial effusion. Cardiac enzymes (CEs) werenegative. Transthoracic echocardiography (TTE)confirmed the large circumferential pericardialeffusion with diastolic right heart collapse (Figure 1,Video 1). No wall motion abnormalities were noted,and left ventricular ejection fraction (LVEF) waspreserved. On therapeutic pericardiocentesis,2,060 ml of sanguineous fluid was removed. A fewminutes after the procedure, the patient reportedsudden-onset chest pressure and nausea, becamehypotensive (blood pressure 88/60 mm Hg), and

https://doi.org/10.1016/j.jaccas.2020.04.029

Medical Center, Albert Einstein College of Medicine,

elevant to the contents of this paper to disclose. Ritu

and animal welfare regulations of the authors’ in-

nsent where appropriate. For more information, visit

Page 30: Full Issue PDF · The JACC: Case Reports Editorial Board and ACC editorial office would like to express their sincere gratitude to the guest editors below who worked tirelessly …

FIGURE 1 Transthoracic Echo

Subcostal view showing large p

ABBR EV I A T I ON S

AND ACRONYMS

LV = left ventricular

LVEF = left ventricular ejection

fraction

PDS = pericardial

decompression syndrome

SCM = stress cardiomyopathy

TTE = transthoracic

echocardiography

Cerrud-Rodriguez et al. J A C C : C A S E R E P O R T S , V O L . 2 , N O . 7 , 2 0 2 0

Overlapping PDS and SCM Post-Pericardiocentesis J U N E 1 7 , 2 0 2 0 : 1 0 0 9 – 1 3

1010

developed worsening tachycardia (heart rate120 to 130 beats/min) and hypoxia (oxygensaturation 80% to 85%).

MEDICAL HISTORY

The patient’s medical history included hy-pertension and obesity.

DIFFERENTIAL DIAGNOSIS

The differential diagnosis included cardiac

perforation, coronary laceration, vasovagal response,pericardial decompression syndrome (PDS), andstress cardiomyopathy (SCM).

INVESTIGATIONS

Emergent TTE was performed a few minutes after thepatient’s change in clinical status and showed anewly reduced LVEF of 30%, with apical, septal, andanterolateral akinesis. All other walls were hypo-kinetic except for the basal left ventricular (LV) seg-ments, which had preserved contractility (Videos 2and 3). Troponin T and creatine kinase-MB werenewly elevated to 1.04 ng/l and 238 U/l, respectively.Electrocardiography showed interval development ofQ waves in the inferior leads. CEs peaked thefollowing day (troponin T 2.0 ng/l, creatine kinase-MB 435 U/l), and repeat TTE showed persistentbiventricular dysfunction, despite complete

cardiography

ericardial effusion causing diastolic right heart collapse.

resolution of the pericardial effusion. LVEF was 25%with new apical thrombus. Pericardial drain outputdecreased to a minimum, and the drain was removed.Coronary angiography revealed mild nonobstructivecoronary artery disease. Cardiac magnetic resonanceimaging showed mildly reduced LVEF of 49% andapical hypokinesis extending into the mid anteriorwall, with no evidence of myocardial fibrosis, infarc-tion, or myocarditis (Figure 2). Repeat TTE prior todischarge (13 days after initial TTE) showed improvedLVEF of 50% and moderate-sized apical akinesis(Video 4). Pericardial fluid analysis demonstrated anexudative effusion by Light’s criteria (pericardialfluid lactate dehydrogenase 557 U/l, serum lactatedehydrogenase 281 U/l). Histopathologic analysisshowed grossly bloody fluid without malignant cellsand reactive inflammatory cells. Other infectious,oncological, and rheumatologic work-up wasnegative.

MANAGEMENT

The initial hypotensive episode after pericardiocent-esis was transient and improved after a small fluidbolus. The patient was started on guideline-directedmedical therapy for heart failure including a beta-blocker and an angiotensin-converting enzyme in-hibitor. He was started on apixaban for treatment ofLV thrombus.

DISCUSSION

Pericardiocentesis is a lifesaving therapeutic pro-cedure for patients presenting with cardiac tampo-nade. It is a relatively safe procedure, butphysicians must be aware of post-procedural com-plications. In particular, hypotension after peri-cardiocentesis should raise suspicion for 2under-recognized and under-reported diagnoses:PDS and SCM.

PDS has been defined as paradoxical hypotension,often with pulmonary edema and ventriculardysfunction, after pericardial drainage. Onset canvary from immediately after drainage to up to 48 hlater, and mortality can be as high as 29% (1). In onecase series, 8 of 10 patients showed elevations of CEs(1), but in other series, a majority of patients with PDSshowed no increases in CEs (2,3). Full recovery of LVfunction is expected in most patients. The true inci-dence of PDS is unknown: in a large retrospectiveanalysis of 1,164 consecutive pericardiocentesis pro-cedures, PDS was reported in only 1 patient, whereasin other series it has been reported in up to 4.8% (4).As is evident, this syndrome has no uniform clinical

Page 31: Full Issue PDF · The JACC: Case Reports Editorial Board and ACC editorial office would like to express their sincere gratitude to the guest editors below who worked tirelessly …

FIGURE 2 Cardiac Magnetic Resonance Imaging

Mildly reduced left ventricular ejection fraction of 49%, apical hypokinesis extending into the mid anterior wall, and no evidence of

myocardial fibrosis, infarction, or myocarditis.

J A C C : C A S E R E P O R T S , V O L . 2 , N O . 7 , 2 0 2 0 Cerrud-Rodriguez et al.J U N E 1 7 , 2 0 2 0 : 1 0 0 9 – 1 3 Overlapping PDS and SCM Post-Pericardiocentesis

1011

presentation and may be associated with both surgi-cal pericardiostomy and pericardiocentesis, whereasthe cause of pericardial effusions and clinical sce-narios varies widely (5,6).

Various hypotheses have been proposed to explainthe mechanism of PDS: hemodynamic shifts(increased venous return and right ventricularexpansion after pericardial fluid removal causes LVcompression and pulmonary edema) (2), ischemicchanges (diminished coronary perfusion caused bycompression of epicardial vessels by the pericardialeffusion that results in transiently impaired

myocardial function and inability to handle the sud-den shifts in volume that occur when the pericardialeffusion is drained too quickly) (7), or autonomicimbalances (increased sympathetic tone caused bythe tamponade results in an inotropic effect that in-creases LV contractility and heart rate and, oncesuddenly removed by pericardiocentesis, leads to theunmasking of underlying LV dysfunction) (8).

In SCM, it has been hypothesized that perhapscardiac tamponade can act as the inciting catechol-amine surge (7). A review of 25 patients with heartfailure after pericardial drainage showed 12 patients

Page 32: Full Issue PDF · The JACC: Case Reports Editorial Board and ACC editorial office would like to express their sincere gratitude to the guest editors below who worked tirelessly …

FIGURE 3 Possible Mechanism Behind Paradoxical Hypotension and Pulmonary Edema Following Pericardiocentesis

Diagram depicting the possible mechanism behind paradoxical hypotension and pulmonary edema following pericardiocentesis in some patients with large pericardial

effusions with tamponade physiology: the tamponade physiology itself causes sympathetic stimulation to compensate for the decrease in cardiac output caused by a

decrease in venous return. That same sympathetic overdrive can cause myocardial damage, which is masked by the sympathetic-predominant state. Once the pericardial

effusion is removed, the sympathetic stimulation is lost, and the myocardial damage is unmasked. This damage has myriad clinical presentations (systolic dysfunction,

shock, pulmonary edema, cardiac enzymes (CEs) normal or elevated) and might be the explanation behind the overlap seen in pericardial decompression syndrome

and stress cardiomyopathy.

Cerrud-Rodriguez et al. J A C C : C A S E R E P O R T S , V O L . 2 , N O . 7 , 2 0 2 0

Overlapping PDS and SCM Post-Pericardiocentesis J U N E 1 7 , 2 0 2 0 : 1 0 0 9 – 1 3

1012

with echocardiographic features consistent withSCM (3).

Our case supports the autonomic imbalance hy-pothesis: systolic dysfunction with apical ballooningand preserved basal function, chest pain, elevatedCEs, and normal coronary arteries, with subsequentimprovement of LV function, are suggestive of SCM.It is possible that the sympathetic overdrive stimu-lated by the tamponade-induced reduction in cardiacoutput causes PDS and/or SCM after pericardiocent-esis. After the abrupt removal of the tamponadephysiology, and thus the source for the sympatheticoverdrive, with pericardiocentesis, a reduction insympathetic stimulation can result in systolicdysfunction, shock, and pulmonary edema with orwithout CE rise (Figure 3).

The management of PDS is generally supportive,and early intervention (fluids or diuretic agentsdepending on volume status, inotropes, and me-chanical support devices if needed) is imperative, asmortality can be as high as 29% (1).

Risk factors for developing hemodynamic insta-bility after pericardiocentesis are currently unknown.The only factor associated with increased mortality inPDS is surgical pericardiostomy when compared withneedle pericardiostomy (1). Expert opinion suggeststhat pericardial fluid should be removed only untiltamponade physiology is resolved, and the restshould be slowly drained to permit adaptive changesin coronary flow, wall stress, and myocardial me-chanics (2). Currently, there are no guidelines toprevent or manage PDS. It is unclear if PDS and SCM

Page 33: Full Issue PDF · The JACC: Case Reports Editorial Board and ACC editorial office would like to express their sincere gratitude to the guest editors below who worked tirelessly …

J A C C : C A S E R E P O R T S , V O L . 2 , N O . 7 , 2 0 2 0 Cerrud-Rodriguez et al.J U N E 1 7 , 2 0 2 0 : 1 0 0 9 – 1 3 Overlapping PDS and SCM Post-Pericardiocentesis

1013

are distinct entities or if they belong to the samespectrum of disease. Studies need to be conducted toinvestigate the possible association between PDS andSCM.

FOLLOW-UP

The patient was discharged and was seen 1 monthlater at outpatient follow-up, without recurrence ofsymptoms.

CONCLUSIONS

The underlying mechanism of PDS is unknown. It ispossible that sympathetic overdrive caused by thetamponade-induced reduction in cardiac output

causes a SCM. A reduction in sympathetic stimulationafter removal of the tamponade physiology can resultin hypotension and even shock with pulmonaryedema. It is important to recognize that SCM and PDSmight not be entirely distinct processes and mightactually belong to the same spectrum of disease.Further studies are needed to establish evidence-based guidelines to prevent and treat these entities.

ADDRESS FOR CORRESPONDENCE: Dr. Roberto C.Cerrud-Rodriguez, Montefiore-Einstein Center forHeart and Vascular Care, Montefiore Medical Center,Albert Einstein College of Medicine, 111 East 210thStreet, Bronx, New York 10467. E-mail:[email protected].

RE F E RENCE S

1. Pradhan R, Okabe T, Yoshida K, Angouras DC,DeCaro MV, Marhefka GD. Patient characteristicsand predictors of mortality associated with peri-cardial decompression syndrome: a comprehen-sive analysis of published cases. Eur Heart J AcuteCardiovasc Care 2015;4:113–20.

2. Mahajan A, Mahajan N, Suri A, Mahajan K. Per-icardiocentesis decompression syndrome: An un-common complication of a common procedure.IHJ Cardiovasc Case Reports 2019;3:18–20.

3. Ayoub C, Chang M, Kritharides L. A casereport of ventricular dysfunction post peri-cardiocentesis: stress cardiomyopathy or peri-cardial decompression syndrome? CardiovascUltrasound 2015;13:32.

4. Ayoub C, Lekhakul A, Assawakawintip C, et al.Abstract 17022: pericardial decompression syn-drome: incidence in a large consecutive series ofechocardiographic-guided pericardiocentesis pro-cedures. Circulation 2018;138:A17022.

5. Vandyke WH Jr., Cure J, Chakko CS,Gheorghiade M. Pulmonary edema after peri-cardiocentesis for cardiac tamponade. N Engl JMed 1983;309:595–6.

6. Kumar R, Sinha A, Lin MJ, et al. Complicationsof pericardiocentesis: a clinical synopsis. Int J CritIlln Inj Sci 2015;5:206–12.

7. Versaci F, Donati R, Mezzanotte R, Chiariello L,Ammirati F. An unusual complication followingpericardiocentesis: reversible left ventricular

dysfunction. J Cardiovasc Med (Hagerstown) 2015;16 Suppl 2:S133–5.

8. Wolfe MW, Edelman ER. Transient systolicdysfunction after relief of cardiac tamponade. AnnIntern Med 1993;119:42–4.

KEY WORDS cardiomyopathy, pericardialdecompression syndrome, pericardialeffusion, stress cardiomyopathy, systolicdysfunction

APPENDIX For supplemental videos,please see the online version of this paper.

Page 34: Full Issue PDF · The JACC: Case Reports Editorial Board and ACC editorial office would like to express their sincere gratitude to the guest editors below who worked tirelessly …

J A C C : C A S E R E P O R T S V O L . 2 , N O . 7 , 2 0 2 0

ª 2 0 2 0 T H E A U T H O R S . P U B L I S H E D B Y E L S E V I E R O N B E H A L F O F T H E AM E R I C A N

C O L L E G E O F C A R D I O L O G Y F O U N DA T I O N . T H I S I S A N O P E N A C C E S S A R T I C L E U N D E R

T H E C C B Y - N C - N D L I C E N S E ( h t t p : / / c r e a t i v e c o mm o n s . o r g / l i c e n s e s / b y - n c - n d / 4 . 0 / ) .

CASE REPORT

CLINICAL CASE

Recurrent Takotsubo Cardiomyopathyin a Patient With HypertrophicCardiomyopathy Leading to CardiogenicShock Requiring VA-ECMO

Mustafa Husaini, MD,a Joshua N. Baker, MD,b Sharon Cresci, MD,a Richard Bach, MD,a Shane J. LaRue, MDa

ABSTRACT

L

ISS

Fro

the

Ka

the

Th

sti

the

Ma

Providing hemodynamic support for patients with hypertrophic cardiomyopathy and cardiogenic shock can be chal-

lenging because inotropic medications worsen intraventricular obstruction, and the effect of appropriate mechanical

support remains undefined. We report a patient with hypertrophic cardiomyopathy in shock because of takotsubo

cardiomyopathy requiring venoarterial extracorporeal membrane oxygenation and septal reduction for full recovery.

(Level of Difficulty: Advanced.) (J Am Coll Cardiol Case Rep 2020;2:1014–8) © 2020 The Authors. Published by Elsevier

on behalf of the American College of Cardiology Foundation. This is an open access article under the CC BY-NC-ND license

(http://creativecommons.org/licenses/by-nc-nd/4.0/).

HISTORY OF PRESENTATION

A 68-year-old woman with a history of hypertensionand takotsubo cardiomyopathy (9 years prior, recov-ered) presented with back pain, nausea, and diapho-resis. Her admission electrocardiogram demonstrateda previously known left bundle branch block, andher initial serum troponin T level was elevated at0.46 ng/ml. She underwent coronary angiography,

EARNING OBJECTIVES

To discuss the differential diagnosis of shockin patients with HCM and takotsubocardiomyopathy.To discuss options for management ofcardiogenic shock in patients with HCM andpotential consequences.

N 2666-0849

m theaDepartment of Medicine, Cardiovascular Division, Washington UnbMissouri Baptist, St. Louis, Missouri. Dr. Bach has received institution

rdia and Cytokinetics for conducting clinical trials. All other authors have

contents of this paper to disclose.

e authors attest they are in compliance with human studies committees

tutions and Food and Drug Administration guidelines, including patient co

JACC: Case Reports author instructions page.

nuscript received January 24, 2020; revised manuscript received March 1

which demonstrated no obstructive coronary arterydisease. Left ventriculography demonstrated apicalballooning with hyperdynamic basal contractilityconsistent with recurrent takotsubo cardiomyopathy.By catheter assessment, she was found to have anintraventricular gradient of 46 mm Hg at rest. Subse-quently, she developed hypotension and signs of shock.

MEDICAL HISTORY

The patient’s medical history included hypertensionand an episode of takotsubo cardiomyopathy 9 yearsprior, which was attributed to multiple stressorsincluding the death of a parent.

DIFFERENTIAL DIAGNOSIS

The differential diagnosis of shock developing in apatient with an intraventricular gradient (suggesting

https://doi.org/10.1016/j.jaccas.2020.04.028

iversity School of Medicine, St. Louis, Missouri; and

al research funding but no personal fees from Myo-

reported that they have no relationships relevant to

and animal welfare regulations of the authors’ in-

nsent where appropriate. For more information, visit

6, 2020, accepted April 23, 2020.

Page 35: Full Issue PDF · The JACC: Case Reports Editorial Board and ACC editorial office would like to express their sincere gratitude to the guest editors below who worked tirelessly …

AB BR E V I A T I O N S

AND ACRONYM S

HCM = hypertrophic

cardiomyopathy

LV = left ventricular

LVOT = left ventricular

outflow tract

SAM = systolic anterior motion

VA-ECMO = venoarterial

extracorporeal membrane

enation

J A C C : C A S E R E P O R T S , V O L . 2 , N O . 7 , 2 0 2 0 Husaini et al.J U N E 1 7 , 2 0 2 0 : 1 0 1 4 – 8 Takotsubo Cardiomyopathy, Obstructive HCM, and Cardiogenic Shock

1015

hypertrophic cardiomyopathy [HCM] physiology),coincident apical dyskinesis, and left ventricular (LV)systolic dysfunction may include low cardiac outputdue to LV pump failure and/or severe left ventricularoutflow tract (LVOT) obstruction. The observation ofapical ballooning raises the possibility that hypoten-sion and shock are due primarily to pump failure, ashas been observed in up to 20% of patients withtakotsubo cardiomyopathy (1). The presence of asystolic murmur and a systolic intraventriculargradient at cardiac catheterization suggest LVOTobstruction may contribute significantly to hypoten-sion and shock.

INVESTIGATIONS

Transthoracic echocardiography showed severe LVdysfunction with apical ballooning and an estimated

FIGURE 1 Transthoracic Echocardiography 13 Months Prior to Curre

Parasternal long-axis in end-diastole (A) revealing asymmetrical septal h

anterior motion (SAM) of the mitral valve (MV). M-mode imaging throug

intraventricular gradient of 16 mm Hg (* ¼ gradient, # ¼ mitral regurgit

outflow tract (not shown). The gradient increased to 45 mm Hg with Va

LV ejection fraction of 30%. There was alsohyperdynamic contraction of the basal seg-ments, systolic anterior motion (SAM) of theanterior mitral valve leaflet, and suggestionof a late-peaking gradient in the LVOT of 25 to35 mm Hg (Video 1).

Although prior echocardiography per-formed 13 months prior to the patient’s pre-sentation was reported as demonstratingconcentric LV hypertrophy, review of theimages revealed asymmetrical septal hyper-

trophy with a septal wall thickness of 1.9 cm and aposterior wall thickness of 1.0 cm, SAM of the mitralvalve, and a late-peaking dynamic LVOT gradient of16 mm Hg at rest that increased to 45 mm Hg withValsalva maneuver (Figure 1, Video 1). These echo-cardiographic characteristics are consistent with adiagnosis of obstructive HCM.

oxyg

nt Presentation

ypertrophy (septal wall 1.9 cm vs. posterior wall 1.0 cm) and in systole (B) revealing systolic

h the MV (C) also demonstrating SAM of the MV. Continuous-wave Doppler (D) revealed an

ation). Pulsed-wave Doppler confirmed that the gradient was localized to the left ventricular

lsalva maneuver (E).

Page 36: Full Issue PDF · The JACC: Case Reports Editorial Board and ACC editorial office would like to express their sincere gratitude to the guest editors below who worked tirelessly …

TABLE 1 Effects of Various Vasoactive Medications and

Mechanical Support Devices in HCM

TherapyEffect onInotropy

Effect onAfterload

Potential Effect on LVOTObstruction in HCM

Dobutamine [[ YY [[

Norepinephrine [ [[ 4/[

Epinephrine [[ [[ [

Phenylephrine 4 [ 4/Y

IABP 4/[ YY [

Impella 4/[ 4 4/[

VA-ECMO 4 [[ Y (presuming adequatepreload)

HCM ¼ hypertrophic cardiomyopathy; IABP ¼ intra-aortic balloon pump;LVOT ¼ left ventricular outflow tract; VA-ECMO ¼ venoarterial extracorporealmembrane oxygenation.

TABLE 2 Summary o

First Author (Year) (Re

Modi et al. (2011) (2)

Nalluri et al. (2017) (3)

Arakawa et al. (2018) (

Sossalla et al. (2019) (

DBA ¼ dobutamine; ECG ¼Table 1.

Husaini et al. J A C C : C A S E R E P O R T S , V O L . 2 , N O . 7 , 2 0 2 0

Takotsubo Cardiomyopathy, Obstructive HCM, and Cardiogenic Shock J U N E 1 7 , 2 0 2 0 : 1 0 1 4 – 8

1016

MANAGEMENT

Some options beneficial for the management ofcardiogenic shock in patients with normal or dilatedhearts may carry hazard for those with HCM (Table 1).Inotropic and vasopressor medications increasemyocardial contractility and may aggravate dynamicLVOT obstruction, if present, while agents known toreduce myocardial contractility and LVOT obstruc-tion, such as beta-adrenergic blockers, that may bebeneficial for HCM may also reduce cardiac outputand worsen hypotension and shock for a patient withacute LV systolic dysfunction. Mechanical circulatory

f Published Cases and Management of Takotsubo Cardiomyopathy Comp

f. #) Journal

International Journal ofCardiology

A 54-year-old woman with quadand hypotension, with ECG ilEchocardiography illustratedCoronary angiography illustramid-LAD muscle bridge.

Intervention: NE, DBA, and IABPVital status: death 3 days after p

Cardiovascular Imaging CaseReports

An 81-year-old woman with knocardiomyopathy was subsequcardiogenic shock post-proce

Intervention: phenylephrine for sVital status: discharged home 3

4) Internal Medicine A 62-year-old woman with a famcardiogenic shock treated bycardiomyopathy with a 50 m12 mm, but RV biopsy illustra

Intervention: intravenous vasoacVital status: discharged home

5) Circulation: CardiovascularInterventions

A 78-year-old woman with knowrecurrence of a LVOT gradientakotsubo cardiomyopathy. BVA-ECMO.

Intervention: repeat alcohol septVital status: discharged to rehab

electrocardiography; LAD ¼ left anterior descending coronary artery; NE ¼ norepinephrin

support options that cause ventricular unloading,such as intra-aortic balloon pump counterpulsation orthe Impella percutaneous LV assist device (Abiomed,Danvers, Massachusetts), may reduce load on thefailing left ventricle and augment cardiac output;however, they may aggravate dynamic LVOTobstruction.

In this patient, intravenous vasoactive medica-tions, including dobutamine and dopamine, wereinitiated without improvement in her hemodynamicstatus. As epinephrine and, to a lesser degree,norepinephrine can stimulate cardiac beta-sympathetic receptors that increase myocardialcontractility, these agents were avoided, as use inpatients with obstructive physiology has the potentialto precipitate worsening cardiogenic shock. Despiteplacement of an intra-aortic balloon pump and sub-sequent placement of an Impella CP catheter, thepatient’s shock continued to worsen. She thenrequired cannulation for venoarterial extracorporealmembrane oxygenation (VA-ECMO) support. Over thenext few days, this patient improved clinically on VA-ECMO support such that the Impella CP catheter wasremoved; however, with evident SAM of the mitralvalve and significant LVOT obstruction, she was re-fractory to weaning from VA-ECMO support. Aftermultiple heart team discussions, the patient under-went a minimally invasive extended septal myectomyand subsequent VA-ECMO decannulation.

licated by Shock in Patients With HCM

Information

riplegia from advanced motor neuron disease presented with chest painlustrating T-wave inversions and positive cardiac biomarkers.asymmetric septal hypertrophy (19 mm), SAM, and apical ballooning.ted no obstructive coronary artery disease but a did demonstrate a

resentation

wn obstructive HCM presented with chest pain and dyspnea; takotsuboently diagnosed during cardiac catheterization. The patient developeddure necessitating phenylephrine infusions.hock, then medical therapydays after presentation

ily history of obstructive HCM presented with chest pain and developednorepinephrine infusion. Cardiac catheterization diagnosed takotsubom Hg LVOT gradient. Echocardiography showed septal thickness ofted myocyte disarray consistent with HCM.tive medications, then medical therapy

n obstructive HCM and prior alcohol septal ablation was found to have at. She then presented with dyspnea and catheterization diagnosedecause of cardiogenic shock, the patient was transferred and placed on

al ablationilitation

e; RV ¼ right ventricular; SAM ¼ systolic anterior motion; other abbreviations as in

Page 37: Full Issue PDF · The JACC: Case Reports Editorial Board and ACC editorial office would like to express their sincere gratitude to the guest editors below who worked tirelessly …

FIGURE 2 Transthoracic Echocardiography 3 Months After Current Presentation

Parasternal long-axis (A) and apical four-chamber (B) illustrating evidence of the extended septal myectomy and absence of systolic anterior motion. Continuous-wave

Doppler (C) and pulsed-wave Doppler (D) demonstrating no dynamic left ventricular outflow tract obstruction. Note that in addition to being lower velocity, the

profiles are now early peaking, not late peaking.

J A C C : C A S E R E P O R T S , V O L . 2 , N O . 7 , 2 0 2 0 Husaini et al.J U N E 1 7 , 2 0 2 0 : 1 0 1 4 – 8 Takotsubo Cardiomyopathy, Obstructive HCM, and Cardiogenic Shock

1017

DISCUSSION

Managing a patient such as this, with apicalballooning, cardiogenic shock, coexistent LVOTobstruction due to basal asymmetrical septal hyper-trophy and SAM, can be complex and challenging, asconventional therapeutic interventions for the failingheart with systolic dysfunction may aggravate theLVOT obstruction. Although inotropic and vasoactivemedications may increase cardiac contractility, me-chanical circulatory support can have varying effectson ventricular loading conditions. As a result, inobstructive HCM, increasing inotropy and/ordecreasing pre-load or afterload can worsen LVOTobstruction. Therefore, for patients with obstructiveHCM, many conventional therapies beneficial for

other patients in cardiogenic shock can worsen LVOTobstruction and thereby worsen the severity ofcardiogenic shock, complicating the management ofthese patients.

There are only 4 other reported cases of patientswith HCM presenting with takotsubo cardiomyopathywho developed cardiogenic shock (Table 2) (2–5). Tothe best of our knowledge, the first such case wasreported in 2011 (2), and we report only the secondpublished case of a patient requiring the use of VA-ECMO. Both the previous case and our case illus-trate the difficulty of liberating a patient from VA-ECMO support in the recovery phase because ofongoing low output of the left ventricle and insuffi-cient LV filling from reduced pre-load (5). Our case isunique in that our patient was treated by a minimally

Page 38: Full Issue PDF · The JACC: Case Reports Editorial Board and ACC editorial office would like to express their sincere gratitude to the guest editors below who worked tirelessly …

Husaini et al. J A C C : C A S E R E P O R T S , V O L . 2 , N O . 7 , 2 0 2 0

Takotsubo Cardiomyopathy, Obstructive HCM, and Cardiogenic Shock J U N E 1 7 , 2 0 2 0 : 1 0 1 4 – 8

1018

invasive extended septal myectomy in order to sub-sequently liberate from VA-ECMO support.

This case highlights several teaching points. First,it illustrates the importance of reviewing prior imag-ing when confronted with a discordant clinical pic-ture and/or findings that suggest additionaldiagnoses. In this case, the cardiac catheterizationsuggested the diagnosis of HCM in addition torecurrent takotsubo cardiomyopathy. Reviewing thispatient’s echocardiogram led to confirmation of thediagnosis of HCM and the realization that septalreduction therapy was an important consideration toimprove her hemodynamic status.

Second, it is important to be aware of underlyingHCM and the degree of dynamic LVOT obstructionwhen considering advanced mechanical support incardiogenic shock, as the choice of mechanical sup-port device could lead to worsening LVOT obstruc-tion. Given the risks of unloading a heart withassociated LVOT obstruction, the preferred mechan-ical support in patients with HCM may be VA-ECMO,contingent on maintaining adequate pre-load to pre-vent worsening LVOT obstruction.

Last, once a diagnosis of HCM is made, it isimportant to communicate to the patient that lifelongfollow-up is needed and that there are importantimplications for family members (i.e., clinicalscreening for HCM).

FOLLOW-UP

The patient did well post-operatively, as all vasoac-tive medications were weaned off and she was dis-charged to a rehabilitation facility on post-operativeday 12. The patient did not require a pacemaker post-operatively and was discharged on metoprolol.Despite being counseled that HCM was a genetic

disease, she was not interested in obtaining genetictesting during the time of her diagnosis. Histologicalexamination of the myectomy specimen did notdemonstrate any evidence of a storage or infiltrativecardiomyopathy. Three months after discharge,echocardiography revealed a thinned basal septum atthe site of the extended septal myectomy, no dy-namic LVOT obstruction, and fully recovered LVsystolic function (Figure 2, Video 1).

CONCLUSIONS

We report the case of a patient with HCM who pre-sented in cardiogenic shock due to recurrent takot-subo cardiomyopathy, needing mechanicalcirculatory support, and requiring an extended septalmyectomy to be liberated from VA-ECMO. Positiveinotropic medications and the use of certain tempo-rary mechanical circulatory support devices canworsen LVOT obstruction in these patients, posingchallenges to appropriate management. In select pa-tients, VA-ECMO with careful attention to pre-loadmay be the preferred option for circulatory supportwhen indicated for patients with HCM in cardiogenicshock. There are currently limited data on the recur-rence risk for episodes of takotsubo cardiomyopathyafter septal reduction therapy. However, if this pa-tient were to again develop takotsubo cardiomyopa-thy, she would theoretically be at a lower risk forhemodynamic decompensation as a result of herobstructive physiology being surgically alleviated.

ADDRESS FOR CORRESPONDENCE: Dr. MustafaHusaini, Washington University in Saint Louis, 660South Euclid Avenue, CB 8086, St. Louis, Mis-souri 63110. E-mail: [email protected]. Twitter:@husainim, @ShaneRue.

RE F E RENCE S

1. Sharkey SW, Pink VR, Lesser JR, Garberich RF,Maron MS, Maron BJ. Clinical profile of patientswith high-risk tako-tsubo cardiomyopathy. Am JCardiol 2015;116:765–72.

2. Modi S, Ramsdale D. Tako-tsubo, hypertrophicobstructive cardiomyopathy & muscle bridging—separate disease entities or a single condition? IntJ Cardiol 2011;147:133–4.

3. Nalluri N, Asti D, Anugu VR, Ibrahim U,Lafferty JC, Olkovsky Y. Cardiogenic shocksecondary to takotsubo cardiomyopathy in a

patient with preexisting hypertrophic obstruc-tive cardiomyopathy. CASE (Phila) 2017;2:78–81.

4. Arakawa K, Gondo T, Matsushita K, Himeno H,Kimura K, Tamura K. Takotsubo cardiomyopathy ina patient with previously undiagnosed hypertro-phic cardiomyopathy with latent obstruction.Intern Med 2018;57:2969–73.

5. Sossalla S, Meindl C, Fischer M, Lubnow M,Müller T, Maier LS. Bail-out alcohol septal ablationfor hypertrophic obstructive cardiomyopathy in a

patient with takotsubo cardiomyopathy-inducedcardiogenic shock. Circ Cardiovasc Interv 2019;12:e007425.

KEY WORDS acute heart failure, cardiacassist devices, cardiomyopathy

APPENDIX For a supplemental video,please see the online version of this paper.

Page 39: Full Issue PDF · The JACC: Case Reports Editorial Board and ACC editorial office would like to express their sincere gratitude to the guest editors below who worked tirelessly …

J A C C : C A S E R E P O R T S VO L . 2 , N O . 7 , 2 0 2 0

ª 2 0 2 0 T H E A U T H O R S . P U B L I S H E D B Y E L S E V I E R O N B E H A L F O F T H E A M E R I C A N

C O L L E G E O F C A R D I O L O G Y F OU N D A T I O N . T H I S I S A N O P E N A C C E S S A R T I C L E U N D E R

T H E C C B Y - N C - N D L I C E N S E ( h t t p : / / c r e a t i v e c o mm o n s . o r g / l i c e n s e s / b y - n c - n d / 4 . 0 / ) .

CASE REPORTS

CLINICAL CASE

Elevated Diaphragm Causing MarkedKinking of the Inferior Vena Cava

Akshar Jaglan, DO, Andy Kieu, DO, Asad Ghafoor, MD, Tanvir Bajwa, MD, A. Jamil Tajik, MD

ABSTRACT

ISS

Fro

Th

Th

ins

vis

Ma

Inferior vena cava (IVC) syndrome is commonly caused by deep vein thrombosis or a mass effect. We present an unusual

case of IVC kinking and obstruction detected by echocardiography and confirmed by cardiac magnetic resonance and

angiography. Management of marked tortuosity of the IVC secondary to a paralyzed hemidiaphragm is discussed.

(Level of Difficulty: Beginner.) (J Am Coll Cardiol Case Rep 2020;2:1019–20) © 2020 The Authors. Published by

Elsevier on behalf of the American College of Cardiology Foundation. This is an open access article under the CC BY-NC-

ND license (http://creativecommons.org/licenses/by-nc-nd/4.0/).

A 74-year-old woman presented to the hospital with reports of worsening dyspnea and lower extremityswelling. She had chronic swelling in her lower extremities, but over the past few weeks the swellingsignificantly worsened. Physical examination revealed bilateral 3þ lower extremity pitting edema,

abdominal distention, and decreased breath sounds on the right lower lung field. Vital signs included bloodpressure of 125/63 mm Hg, heart rate of 87 beats/min, and blood oxygen saturation at 83% on room air.

At 30 years of age, she had a sky diving accident, secondary to failure of parachute deployment, whichresulted in an L1 vertebral fracture that caused paraplegia. Her medical history also included chronic lym-phedema, paroxysmal atrial fibrillation, right hemidiaphragm elevation, hypertrophic cardiomyopathy, andpulmonary hypertension.

An electrocardiogram demonstrated normal sinus rhythm. Her troponin level was not elevated, and theN-terminal pro–B-type natriuretic peptide level was slightly elevated at 298 pg/ml (referencerange: <126 pg/ml). Chest radiography and chest computed tomography showed a markedly elevated, para-lyzed, right hemidiaphragm (Figure 1A, Supplemental Figure 1). Echocardiography revealed a stenotic inferiorvena cava (IVC) with an elevated velocity (2.0 m/s) at the cavoatrial junction (Figures 1B and 1C). The patient’spulmonary artery systolic pressure was 50 mm Hg. Further imaging was pursued, with cardiac magneticresonance confirming hemidiaphragm elevation causing significant tortuosity of the IVC at the cavoatrialjunction, which measured approximately 8 mm (Figure 1D). Several venous collateral vessels, related to the IVCstenosis and that drained into the subclavian veins, were seen in the anterior abdominal wall and chest wall(Figure 1E). No evidence of thrombus formation or mass burden was seen. We postulated that the patient’smarkedly elevated hemidiaphragm had, over time, resulted in the elongation, kinking, and severe stenosis ofthe IVC that led to her current presentation. Selective angiography showed a tortuous IVC that possibly couldbe amenable to percutaneous stenting (Figure 1F, Video 1). Right-sided heart catheterization demonstrated a

N 2666-0849 https://doi.org/10.1016/j.jaccas.2020.04.032

m the Aurora Cardiovascular and Thoracic Services, Aurora Sinai/Aurora St. Luke’s Medical Centers, Milwaukee, Wisconsin.

e authors have reported that they have no relationships relevant to the contents of this paper to disclose.

e authors attest they are in compliance with human studies committees and animal welfare regulations of the authors’

titutions and Food and Drug Administration guidelines, including patient consent where appropriate. For more information,

it the JACC: Case Reports author instructions page.

nuscript received April 29, 2020; accepted April 30, 2020.

Page 40: Full Issue PDF · The JACC: Case Reports Editorial Board and ACC editorial office would like to express their sincere gratitude to the guest editors below who worked tirelessly …

FIGURE 1 Kinked IVC

(A) Chest radiograph shows an

with severe narrowing at the ca

the turbulent flow at the cavoa

diaphragm elevation causing sig

demonstrates several venous co

kinking of the IVC (arrow) (Vide

ABBR EV I A T I ON S

AND ACRONYMS

IVC = inferior vena cava

Jaglan et al. J A C C : C A S E R E P O R T S , V O L . 2 , N O . 7 , 2 0 2 0

Kinked Inferior Vena Cava J U N E 1 7 , 2 0 2 0 : 1 0 1 9 – 2 0

1020

mean gradient of 10 mm Hg between the IVC and the right atrium (mean right atrial pressure,12 mm Hg; mean IVC pressure, 22 mm Hg).

Medical therapy was initiated with torsemide and spironolactone. The patient spent a total of2 weeks in the hospital, with diuresis of 22.3 l, before discharge. Her symptoms improved, and she wasdischarged on the foregoing regimen.

This report, to our knowledge, is the first to describe marked kinking or tortuosity of the IVC at the cavoatrialjunction secondary to elevation of the right hemidiaphragm, as demonstrated with multimodality imaging.

ACKNOWLEDGMENTS The authors are grateful to Rahul Sawlani, MD, of the Aurora St. Luke’s Medical CenterRadiology Department for his assistance with the MRI images (Figure 1D and 1E); Jennifer Pfaff and Susan Nordof Aurora Cardiovascular and Thoracic Services for editorial preparation of the manuscript; and Brian Millerand Brian Schurrer of Advocate Aurora Research for assistance with the figures.

elevated right diaphragm (arrows) (Supplemental Figure 1). (B) Subcostal echocardiography reveals a dilated inferior vena cava (IVC)

voatrial junction (arrow) accompanied by turbulent flow on color Doppler imaging. (C) Continuous-wave Doppler imaging directed at

trial junction reveals high velocity (2.0 m/s) indicative of severe obstruction. (D) Cardiac magnetic resonance demonstrates hemi-

nificant tortuosity of the IVC at the cavoatrial junction, which measured approximately 8 mm (arrow). (E) Magnetic resonance imaging

llateral vessels in the anterior abdominal and chest wall. (F) Selective angiography demonstrates marked dilation, elongation, and

o 1). RA ¼ right atrium.

ADDRESS FOR CORRESPONDENCE: Dr. A. Jamil Tajik, Aurora St. Luke’s Medical Center, 2801 West Kinnick-innic River Parkway, Suite 880, Milwaukee, Wisconsin 53215. E-mail: [email protected].

KEY WORDS angiogram, echocardiogram, inferior vena cava, magnetic resonance imaging

APPENDIX For a supplemental figure and video, please see the online version of this paper.

Page 41: Full Issue PDF · The JACC: Case Reports Editorial Board and ACC editorial office would like to express their sincere gratitude to the guest editors below who worked tirelessly …

J A C C : C A S E R E P O R T S VO L . 2 , N O . 7 , 2 0 2 0

ª 2 0 2 0 T H E A U T H O R S . P U B L I S H E D B Y E L S E V I E R O N B E H A L F O F T H E A M E R I C A N

C O L L E G E O F C A R D I O L O G Y F OU N D A T I O N . T H I S I S A N O P E N A C C E S S A R T I C L E U N D E R

T H E C C B Y - N C - N D L I C E N S E ( h t t p : / / c r e a t i v e c o mm o n s . o r g / l i c e n s e s / b y - n c - n d / 4 . 0 / ) .

CASE REPORT

CLINICAL CASE

Echocardiographic Diagnosis ofBland-White-Garland Syndromein an Asymptomatic Adult

Shruti Hegde, MD,a Jennifer Bell, MD,a Benoy Zachariah, MD,b Emani Sitaram, MD,c Michael Maysky, MDa

ABSTRACT

L

ISS

Fro

of

Su

the

Th

sti

the

Ma

Anomalous origin of the left coronary artery from the pulmonary artery (ALCAPA) is a rare congenital defect and usually

diagnosed within the first 2 months of life. Only 10% of patients survive to adulthood largely in part to the formation of

extensive collaterals from the right to left coronary arteries. We present a case of ALCAPA diagnosed in an asymptomatic

adult through a transthoracic echocardiogram (TTE). (Level of Difficulty: Beginner.) (J Am Coll Cardiol Case Rep

2020;2:1021–4) © 2020 The Authors. Published by Elsevier on behalf of the American College of Cardiology Foundation.

This is an open access article under the CC BY-NC-ND license (http://creativecommons.org/licenses/by-nc-nd/4.0/).

HISTORY OF PRESENT ILLNESS

A 37-year healthy man presented to establish carewith a primary care physician. He had no specificcomplaints. His vitals were within normal limits witha heart rate of 74 beats/min, blood pressure of 124/

EARNING OBJECTIVES

To highlight the importance of understand-ing the vascular anatomy of ALCAPA, clinicalpresentation in adults, and diagnosis.To identify the echocardiographic featurestypical of ALCAPA and use in combinationwith clinical data to significantly reducemisdiagnosis or nondiagnosis, therebyimproving survival.To understand the post-operative manage-ment of patients with ALCAPA.

N 2666-0849

m the aDepartment of Medicine, Division of Cardiology, St. Elizabeth’s Me

Medicine, Division of Cardiology, Good Samaritan Hospital, Brockton, Ma

rgery, Boston Children’s Hospital, Boston, Massachusetts. The authors hav

contents of this paper to disclose.

e authors attest they are in compliance with human studies committees

tutions and Food and Drug Administration guidelines, including patient co

JACC: Case Reports author instructions page.

nuscript received February 18, 2020; accepted March 19, 2020.

60 mm Hg, respiratory rate of 16 breaths/min, andoxygen saturation of 98% on room air. His physicalexamination was notable for a grade 3 of 6 hol-osystolic murmur heard best at the apex that radiatedto his back.

The patient had a history of measles when he was 2years of age. Otherwise, his past medical history wasunremarkable.

DIFFERENTIAL DIAGNOSIS

The differential diagnosis for an apical holosystolicmurmur is mitral regurgitation owing to mitral valvedysfunction.

INVESTIGATIONS

Electrocardiography was notable for a left anteriorfascicular block. His laboratory work-up was within

https://doi.org/10.1016/j.jaccas.2020.03.036

dical Center, Brighton, Massachusetts; bDepartment

ssachusetts; and the cDepartment of Cardiothoracic

e reported that they have no relationships relevant to

and animal welfare regulations of the authors’ in-

nsent where appropriate. For more information, visit

Page 42: Full Issue PDF · The JACC: Case Reports Editorial Board and ACC editorial office would like to express their sincere gratitude to the guest editors below who worked tirelessly …

FIGURE 1 Dilated

Angiogram

Coronary angiogram

nary artery with ex

ABBR EV I A T I ON S

AND ACRONYMS

ALCAPA = anomalous origin of

the left coronary artery from

the pulmonary artery

LCA = left coronary artery

RCA = right coronary artery

Hegde et al. J A C C : C A S E R E P O R T S , V O L . 2 , N O . 7 , 2 0 2 0

Echocardiographic Diagnosis of Bland-White-Garland Syndrome J U N E 1 7 , 2 0 2 0 : 1 0 2 1 – 4

1022

normal limits. Transthoracic echocardiogra-phy revealed an ejection fraction of 45%,moderate posteriorly directed mitral regur-gitation, mild hypokinesis of anteroseptumand anterior, anterolateral, and apical seg-ments of the left ventricle. Color Dopplershowed low-velocity, diastolic linear flow inthe septum from inferoseptum to ante-

roseptum, consistent with coronary vessel flow.Low-velocity diastolic flow was also noted into thepulmonary artery, representing reversed flow froman anomalous left main emptying into the pulmo-nary artery (Videos 1, 2, 3, and 4). Overall findingswere highly characteristic of an anomalous origin ofthe left coronary artery from the pulmonary artery(ALCAPA). He underwent a stress echocardiogram inwhich he achieved target heart rate and exercisedfor 13 min without experiencing chest pain ordiscomfort. Stress electrocardiography was notablefor 3-mm ST-segment depressions in the lateralleads (V4 to V6) and 1-mm ST-segment elevation inthe aVR lead. Post-exercise echocardiographyshowed more prominent hypokinesis of the anteriorwall and hypokinesis of anteroseptum. Mitral

Tortuous Right Coronary Artery as Seen on Coronary

(arrow) showing the dilated and tortuous right coro-

tensive collaterals to the left coronary artery.

regurgitation remained unchanged from baseline.Cardiac catheterization confirmed the presence ofan anomalous left coronary artery (LCA) originatingfrom the pulmonary artery. Right coronary angiog-raphy showed a very large right coronary artery(RCA) that filled the entire LCA retrograde viacollaterals with flow reversal in the LCA andcontrast emptying into the pulmonary artery(Figures 1 and 2). Ascending aortography confirmedthat there was no LCA arising from the ascendingaorta (Figure 3). His filling pressures were normal,and there was no step-up in oxygen saturationpresent.

MANAGEMENT

The patient then underwent successful reimplanta-tion of the LCA with concurrent mitral valve repair(Figures 4 to 6). His post-operative course remaineduneventful, and he was discharged on aspirin andwarfarin, given the very robust competing collateralflow with a dilated RCA, which carries a risk of coro-nary thrombosis.

FIGURE 2 Filling of Left Coronary Artery Through an Extensive

Network of Collaterals From the Right System

Coronary angiogram showing filling of left coronary artery via collat-

erals (red arrow). The blue arrow points to the origin of the left

coronary artery from the pulmonary artery.

Page 43: Full Issue PDF · The JACC: Case Reports Editorial Board and ACC editorial office would like to express their sincere gratitude to the guest editors below who worked tirelessly …

FIGURE 3 The Origin of the Left Coronary Artery From the Pulmonary Artery and

Right Coronary From the Aorta

Aortogram shows origin of the left coronary artery from the pulmonary artery

(blue arrow). The red arrow points to the origin of the right coronary artery from the

right coronary sinus aorta.

FIGURE 4 Dilated Tortuous Epicardial Right Coronary Artery as Seen

Intraoperatively

Intraoperative photo showing the tortuous and dilated right coronary artery

(blue arrow) with extensive collaterals (red arrow).

J A C C : C A S E R E P O R T S , V O L . 2 , N O . 7 , 2 0 2 0 Hegde et al.J U N E 1 7 , 2 0 2 0 : 1 0 2 1 – 4 Echocardiographic Diagnosis of Bland-White-Garland Syndrome

1023

FOLLOW-UP

On his 1-month post-operative follow-up, he re-ported doing well. He admitted to realizing that hemay have had some chest discomfort whileclimbing hills but that this has completelyresolved since his surgery. A murmur was nolonger heard on a physical examination. He iscompliant with his aspirin and coumadin withoutbleeding issues.

DISCUSSION

Bland-White-Garland syndrome, also known asALCAPA, was first described in the literature in 1933by Bland, White, and Garland (1). ALCAPA is a veryrare congenital anomaly that occurs in 1 in 300,000births, with a 90% mortality within the first year oflife when left untreated (2). Although it mostcommonly presents in infancy, 10% of children withthis congenital anomaly will survive to adulthood.Case reports on adults with ALCAPA show that theirfirst presentation is on a spectrum of beingasymptomatic to acute heart failure, myocardialischemia, and sudden cardiac death (3,4). The de-gree of collateral formation between the RCA andLCA serves as a significant predictor for symptoms.Dilation of the RCA and the extensive collaterali-zation from the RCA to the left system occurs as acompensation for the lower blood oxygen level inthe LCA. Continuous blood from the RCA into theLCA and then into the low-pressure pulmonary ar-tery results in coronary steal phenomenon. Papil-lary muscle and LV lateral wall dysfunction inducedby chronic ischemia result in mitral regurgitation(5). Because the incidence of an adult type ofALCAPA is very low, often asymptomatic, andlacking ST-T changes on electrocardiography, pa-tients with this congenital anomaly go through lifeeither undiagnosed or misdiagnosed. Echocardiog-raphy is usually the initial diagnostic tool. Echo-cardiographic features including color Dopplermapping can show a continuous shunt retrogradeinto the pulmonary artery lumen at the abnormalorigin of the LCA. Abundant collateral bloodflow may be observed in the myocardium, espe-cially in the interventricular septum. Other sec-ondary features include left ventricular dilation anddysfunction, papillary muscle fibrosis, mitral valveprolapse or regurgitation, abnormal wall motion,and dilatated RCA (5). Although magnetic resonanceangiography and computed tomography angiog-raphy have been increasingly used for the diagnosisof ALCAPA, these modalities are unable to

determine intravascular blood flow conditions andrequire administration of contrast dye. Echocardi-ography is an indispensable diagnostic toolbecause it is noninvasive, is low cost, and hasthe ability to show the origin of the coronary

Page 44: Full Issue PDF · The JACC: Case Reports Editorial Board and ACC editorial office would like to express their sincere gratitude to the guest editors below who worked tirelessly …

FIGURE 5 Intraoperative Image of Mitral Valve Repair

Intraoperative photo showing the pulmonary artery (blue arrow) being

opened and the deep-seated origin of left coronary artery (red arrow).

FIGURE 6 Intraoperative Image Showing Mitral Valve Repair

Hegde et al. J A C C : C A S E R E P O R T S , V O L . 2 , N O . 7 , 2 0 2 0

Echocardiographic Diagnosis of Bland-White-Garland Syndrome J U N E 1 7 , 2 0 2 0 : 1 0 2 1 – 4

1024

artery. An obvious drawback is when the echocar-diographer does not have comprehensive knowl-edge of ALCAPA. Of note, the diagnosis may bedifficult in the setting of pulmonary hypertension,as the blood flow in the LCA might be forward, andthe “stealing blood” sign of the coronary artery isnot present.

CONCLUSIONS

ALCAPA is a rare congenital heart disease thatcarries a high mortality rate and may lead to suddendeath if left untreated. An understanding of thetypical echocardiographic features of ALCAPA iscritical in order to obtain the often elusive diag-nosis. Relatively young patients noted to have

depressed left ventricular systolic function andconcomitant mitral regurgitation without underlyingcoronary artery disease should raise suspicion foranomalous origin of a coronary artery. Ultrasoundimaging with transthoracic echocardiography servesas an essential tool for the early diagnosis of thiscongenital anomaly. An operator’s comprehensiveknowledge, diagnostic awareness, and echocardio-graphic skills are critical for the accurate diagnosisof ALCAPA.

ACKNOWLEDGMENT The authors would like tothank Robin Brides, RDCS, for performing thetransthoracic electrocardiography and obtaininggreat images.

ADDRESS FOR CORRESPONDENCE: Dr. ShrutiHegde, Cardiovascular Division, St. Elizabeth’s Med-ical Center, 736 Cambridge Street, Brighton, Massa-chusetts 02135. E-mail: [email protected].

RE F E RENCE S

1. Bland EF, White PD, Garland J. Congenitalanomalies of the coronary arteries: report of anunusual case associated with cardiac hypertrophy.Am Heart J 1933;8:787–801.

2. Yau JM, Singh R, Halpern EJ, Fischman D.Anomalous origin of the left coronary arteryfrom the pulmonary artery in adults: a compre-hensive review of 151 adult cases and a newdiagnosis in a 53-year-old woman. Clin Cardiol2011;34:204–10.

3. Fierens C, Budts W, Denef B, et al. A 72-year-oldwoman with ALCAPA. Heart 2000;83:e2.

4. Lin S, Xie M, Lv Q, et al. Misdiagnosis ofanomalous origin of the left coronary artery fromthe pulmonary artery by echocardiography: single-center experience from China. Echocardiography2020;37:104–13.

5. Ghaffari S, Ranjibar A, Hajizadeh R, et al. Asso-ciation of significant mitral regurgitation and leftventricular dysfunction with ALCAPA syndrome in

a young patient. Crescent J Med Biol Sci 2018;5:67–8.

KEY WORDS ALCAPA, coronary anomaly,dilated coronary artery, echocardiography

APPENDIX For supplemental videos,please see the online version of this paper.

Page 45: Full Issue PDF · The JACC: Case Reports Editorial Board and ACC editorial office would like to express their sincere gratitude to the guest editors below who worked tirelessly …

J A C C : C A S E R E P O R T S VO L . 2 , N O . 7 , 2 0 2 0

ª 2 0 2 0 T H E A U T H O R S . P U B L I S H E D B Y E L S E V I E R O N B E H A L F O F T H E A M E R I C A N

C O L L E G E O F C A R D I O L O G Y F OU N D A T I O N . T H I S I S A N O P E N A C C E S S A R T I C L E U N D E R

T H E C C B Y - N C - N D L I C E N S E ( h t t p : / / c r e a t i v e c o mm o n s . o r g / l i c e n s e s / b y - n c - n d / 4 . 0 / ) .

CASE REPORT

CLINICAL CASE

Obstructing Chiari Network FacilitatingBlood Flow Across a Patent ForamenOvale Causing Hypoxia

Rimmy Garg, MD,a Tyler Wark, MD,a John Dudley, MD,a,b James Robertson, MDa,b

ABSTRACT

L

ISS

Fro

the

rel

Th

sti

the

Ma

A 36-year-old man with progressive dyspnea and hypoxia was found to have a large, partially fenestrated Chiari network

accelerating blood flow through a patent foramen ovale with preservation of an embryonic right-to-left atrial flow

pattern. He underwent successful percutaneous patent foramen ovale closure with resolution of his exertional symptoms.

(Level of Difficulty: Beginner.) (J Am Coll Cardiol Case Rep 2020;2:1025–8) © 2020 The Authors. Published by Elsevier

on behalf of the American College of Cardiology Foundation. This is an open access article under the CC BY-NC-ND license

(http://creativecommons.org/licenses/by-nc-nd/4.0/).

HISTORY OF PRESENTATION

A 36-year-old man incarcerated at the Adult Correc-tional Institutions site in Rhode Island, with a historyof metabolic syndrome, presented with approxi-mately 2 years of chronic dyspnea on exertion, asso-ciated nonproductive cough, and intermittentsubsternal chest discomfort. He also related episodesof paroxysmal nocturnal dyspnea, coughing andchoking arousals several times weekly while

EARNING OBJECTIVES

To consider right-to-left intracardiac shunt-ing in cases of unexplained hypoxemia.To understand that a physiologically impor-tant right-to-left intracardiac shunt can becaused by a PFO combined with a large re-sidual Chiari network directing venous returntoward a PFO and obstructing the normalflow pattern toward the right ventricle.

N 2666-0849

m the aDepartment of Internal Medicine, Warren Alpert Medical SchoolbLifespan Cardiovascular Institute, Providence, Rhode Island. The auth

evant to the contents of this paper to disclose.

e authors attest they are in compliance with human studies committees

tutions and Food and Drug Administration guidelines, including patient co

JACC: Case Reports author instructions page.

nuscript received January 30, 2020; revised manuscript received April 1,

recumbent, worsening daytime somnolence, and oc-casional headaches. He was known at the AdultCorrectional Institutions to snore heavily. He deniedsore throat, sinus pressure, orthopnea, uncontrolledheartburn or reflux, abdominal pain, peripheraledema, and weight gain. He recalled being diagnosedwith a “hole in his heart” during childhood but de-nied any deleterious clinical sequelae related to this.He denied a history of stroke. He had normal vitalsigns at rest with an unremarkable physical exami-nation. However, he became hypoxemic to 88% withminimal ambulation in 4-point shackles, requiringseveral minutes to improve back to baseline of 94% to96%.

MEDICAL HISTORY

The patient had a history of hypertension, insulin-dependent diabetes mellitus, dyslipidemia, unspeci-fied mood disorder with psychotic features,sickle-cell trait, gastroesophageal reflux disease, and

https://doi.org/10.1016/j.jaccas.2020.04.030

of Brown University, Providence, Rhode Island; and

ors have reported that they have no relationships

and animal welfare regulations of the authors’ in-

nsent where appropriate. For more information, visit

2020, accepted April 3, 2020.

Page 46: Full Issue PDF · The JACC: Case Reports Editorial Board and ACC editorial office would like to express their sincere gratitude to the guest editors below who worked tirelessly …

FIGURE 1 Large Patent Foramen Ovale on Bubble StudyABBR EV I A T I ON S

AND ACRONYMS

CT = computed tomography

PFO = patent foramen ovale

Garg et al. J A C C : C A S E R E P O R T S , V O L . 2 , N O . 7 , 2 0 2 0

Hypoxia From Chiari Network-Facilitated Shunt J U N E 1 7 , 2 0 2 0 : 1 0 2 5 – 8

1026

hepatic steatosis. There was no family historyof premature coronary artery disease,asthma, chronic obstructive pulmonary dis-ease, or cirrhosis.

DIFFERENTIAL DIAGNOSIS

Bubble study with agitated saline on transthoracic echocardi-

ography demonstrating the presence of a patent foramen

ovale.

Given the patient’s worsening dyspnea and limitedwork-up to date, the differential diagnosis for hisunexplained hypoxemia remained broad. Thisincluded adult congenital heart diseases causingright-to-left shunting such as transposition of thegreat arteries, tricuspid disease with an atrial septaldefect or ventricular septal defect including Ebstein’sanomaly, tetralogy of Fallot, Taussig-Bing anomaly,truncus arteriosus, total anomalous pulmonaryvenous return with atrial septal defect, and Eisen-menger syndrome associated with atrial septal defect,ventricular septal defect, and patent ductus arterio-sus. Pulmonary causes for hypoxia included pulmo-nary arteriovenous fistula and hepatopulmonarysyndrome. Pulmonary causes of ventilation-perfusionmismatch such as pulmonary hypertension, intersti-tial lung disease, and chronic pulmonary emboli werealso considered. Finally, the differential diagnosisincluded platypnea-orthodeoxia syndrome.

INVESTIGATIONS

Laboratory data were notable for abnormal restingarterial blood gas (pH ¼ 7.37, pCO2 ¼ 45 mm Hg,pO2 ¼ 50 mm Hg) and hemoglobin of 14 g/dl. Hiselectrolyte, serial troponin, and B-type natriureticpeptide levels were normal. Initial imaging demon-strated normal results on chest radiography andmyocardial perfusion imaging. Transthoracic echo-cardiography was unremarkable.

After the index hospitalization, the patient wasempirically treated with bronchodilators and dis-charged to the Adult Correctional Institutions withclose pulmonology follow-up. His symptoms per-sisted and he was rehospitalized. At this time, pul-monary function tests demonstrated a reduced ratioof forced expiratory volume in 1 s to forced vital ca-pacity and diffusion capacity of the lungs for carbonmonoxide suggestive of restrictive lung disease, buthigh-resolution chest computed tomography ruledout intraparenchymal pulmonary fibrosis. There wasnow concern for a chronic vascular and/or intracar-diac shunting process given his ongoing intermittenthypoxemia.

In the respiratory clinic, the patient reported noimprovement with his inhalers, and hypoxemia wasdocumented on a 6-min walk test. At this time, repeat

transthoracic echocardiography with a bubble study(Figure 1, Video 1) noted a patent foramen ovale (PFO)with a large right-to-left shunt and an atrial septalaneurysm. Pulmonary pressures were normal. Assuch, he underwent transesophageal echocardiogra-phy, which was notable for an atrial septal aneurysmwith a large PFO (Figure 2, Video 2) with left-to-rightflow by color Doppler and right-to-left flow by salinecontrast. A large fenestrated residual Chiari complexwas also observed (Figure 3, Video 3), with flow ac-celeration across this membrane suggesting somedegree of obstruction.

MANAGEMENT

The patient was scheduled for PFO closure to defini-tively address his persistent hypoxemia. Intra-operative echocardiography showed a PFO with anatrial septal aneurysm with bidirectional shunting atbaseline. His hemodynamic pressures were normal.He underwent successful transfemoral percutaneousPFO closure with no post-procedural shunting notedby color Doppler flow. His oxygen saturation was 93%to 95% on 2 l supplemental oxygen pre-procedure andimproved to 100% on 2 l oxygen post-procedure. Hewas discharged on aspirin 81 mg/day and clopidogrel75 mg/day.

DISCUSSION

A Chiari network is a fenestrated network of fibers inthe right atrium that are in connection with theeustachian and Thebesian valves at the opening of

Page 47: Full Issue PDF · The JACC: Case Reports Editorial Board and ACC editorial office would like to express their sincere gratitude to the guest editors below who worked tirelessly …

FIGURE 3 Transesophageal Echocardiography With Chiari

Network

White arrow points to the Chiari network. Yellow arrow points

to the interatrial septum.

FIGURE 2 Transesophageal Echocardiography With Color

Doppler at Patent Foramen Ovale

Transesophageal echocardiography with Doppler color flow

across the interatrial septum at the patent foramen ovale.

J A C C : C A S E R E P O R T S , V O L . 2 , N O . 7 , 2 0 2 0 Garg et al.J U N E 1 7 , 2 0 2 0 : 1 0 2 5 – 8 Hypoxia From Chiari Network-Facilitated Shunt

1027

the inferior vena cava and the coronary sinus thatresults from an incomplete resorption of the rem-nants of the right valve of the sinus venosus duringembryonic development (1). This network usually hasno clinical significance, but there have been reports ofits role in forming thrombi, atrial septal aneurysms,facilitating paradoxical emboli, arrhythmias, andinfective endocarditis. The network also contributesto difficult placement of devices such as catheters,guidewires, and pacemakers and acts as a protectivefilter against pulmonary emboli (2,3).

Of note, the Chiari network is associated with thepersistence of a PFO, as the fibers may direct bloodflow from the inferior vena cava to the interatrialseptum, causing a right-to-left shunt (1). Generally,flow across an interatrial septal defect is left to right,but transient right to left shunting can occur, partic-ularly with isometric strain (4) such as with a Valsalvamaneuver or coughing. Right-to-left shunting is alsoassociated with high pulmonary pressures as well asother anatomic changes such as thoracic aortic an-eurysms, thoracic trauma with tricuspid regurgita-tion, and mediastinal shifts after thoracic surgery (5).Right-to-left shunting, although not as common asleft-to-right shunting, can lead to profoundhypoxemia.

In an asymptomatic patient, a Chiari network re-quires no intervention, but surgical resection can beconsidered if there is persistent hypoxemia, cyanosis,or obstruction (6). Similarly, an incidentally discov-ered PFO requires no follow-up or treatment unless ithas high-risk features. Cryptogenic stroke is associ-ated with a high-risk PFO score ($2), which includes aChiari network (7). However, PFO closure has been

found to be beneficial in a variety of cases, such asreducing recurrent stroke or transient ischemic at-tacks compared with medical therapy (8), alleviatingsymptoms in patients with platypnea-orthodeoxia(9), and can potentially improve quality of life inpatients with chronic obstructive pulmonary diseasewith hypoxemia (10).

In our patient with unexplained hypoxemia, whowas found to have a large residual Chiari network inthe setting of a large PFO, it was believed that he washaving intermittent hypoxemia secondary to thestructural cardiac defects. As such, he underwent PFOclosure.

Right heart catheterization was not performed toquantify the right-to-left shunt, as it would not havechanged clinical decision making. There was ananatomic reason for significant shunting, directing usto PFO closure. Furthermore, calculation of the shunt(Qp/Qs) in this case would be prone to errors. Systemicarterial flow (Qs) would be calculated using Fick’sprinciple and pulmonary flow (Qp) using thermodi-lution from the catheterization. However, much ofthe temperature bolus from the saline could bedirected by the Chiari network through the PFO andreduce the peak of the time/temperature curve seenby the thermistor in the pulmonary artery, resultingin a falsely elevated circulation of pulmonary flow.

FOLLOW-UP

The patient’s PFO closure was successful, withoutpost-procedural complications. He had resolution ofhis exertional symptoms at a 1-month post-procedurefollow-up visit.

Page 48: Full Issue PDF · The JACC: Case Reports Editorial Board and ACC editorial office would like to express their sincere gratitude to the guest editors below who worked tirelessly …

Garg et al. J A C C : C A S E R E P O R T S , V O L . 2 , N O . 7 , 2 0 2 0

Hypoxia From Chiari Network-Facilitated Shunt J U N E 1 7 , 2 0 2 0 : 1 0 2 5 – 8

1028

CONCLUSIONS

This case illustrates an unusual anatomic variation oftwo commonly benign findings on echocardiography,a Chiari network and PFO. These findings resulted inclinically significant and late presenting cyanoticheart disease with right-to-left shunting and shouldbe considered in the spectrum of late presentingcyanotic heart disease in adults. Marked clinical

improvement resulted from percutaneous PFOclosure.

ADDRESS FOR CORRESPONDENCE: Dr. RimmyGarg, Department of Internal Medicine, WarrenAlpert Medical School, Brown University, 593 EddyStreet, Providence, Rhode Island 02903. E-mail:[email protected].

RE F E RENCE S

1. Schneider B, Hofmann T, Justen MH, Meinertz T.Chiari’s network: normal variant or risk factor forarterial embolic events? J Am Coll Cardiol 1995;26:203–10.

2. Islam AKMM, Sayami LA, Zaman S. Chiarinetwork: a case report and brief overview. J SaudiHeart Assoc 2013;25:225–9.

3. Obaji SG, Cooper R, Somauroo J. Chiari network:a protective filter against pulmonary embolism incase of polycythaemia. BMJ Case Rep 2012;2012:bcr0520114289.

4. Somme RJ, Hijazi Z, Rhodes JF. Pathophysi-ology of congenital heart disease in the adult: partI: shunt lesions. Circulation 2008;117:1090–9.

5. Szirt R, Youssef GS. Acute right to left shunt—combination of tricuspid valve endocarditis, Chiari

network, and ostium secundum atrial septaldefect. J Cardiol Cases 2017;16:151–3.

6. Morgan CT, Gross G, Hickey E, Mertens L.Surgical resection of an obstructive Chiarinetwork. Eur Heart J Cardiovasc Imaging 2017;18:1070.

7. Nakayama R, Takaya Y, Akagi R, et al. Identifi-cation of high-risk patent foramen ovale associ-ated with cryptogenic stroke: development of ascoring system. J Am Soc Echocardiogr 2019;32:811–6.

8. Vaduganathan M, Qamar A, Gupta A, et al.Patent foramen ovale closure for secondary pre-vention of cryptogenic stroke: updated meta-analysis of randomized clinical trials. Am J Med2018;131:575–7.

9. Pederson TA, Larsen SH, Nielsen-Kudsk JE.Closure of a patent foramen ovale in patients withplatypnea-orthodeoxia: a rare and overlookedcause of dyspnoea and hypoxaemia. Scand Car-diovasc J 2015;49:357–60.

10. Layoun ME, Aboulhosn JA, Tobis JM. Potentialrole of patent foramen ovale in exacerbatinghypoxemia in chronic pulmonary disease. TexHeart Inst J 2017;44:189–97.

KEY WORDS Chiari network, congenitalheart defect, dyspnea, hypoxemia

APPENDIX For supplemental videos,please see the online version of this paper.

Page 49: Full Issue PDF · The JACC: Case Reports Editorial Board and ACC editorial office would like to express their sincere gratitude to the guest editors below who worked tirelessly …

J A C C : C A S E R E P O R T S VO L . 2 , N O . 7 , 2 0 2 0

ª 2 0 2 0 T H E A U T H O R S . P U B L I S H E D B Y E L S E V I E R O N B E H A L F O F T H E A M E R I C A N

C O L L E G E O F C A R D I O L O G Y F OU N D A T I O N . T H I S I S A N O P E N A C C E S S A R T I C L E U N D E R

T H E C C B Y - N C - N D L I C E N S E ( h t t p : / / c r e a t i v e c o mm o n s . o r g / l i c e n s e s / b y - n c - n d / 4 . 0 / ) .

CASE REPORT

CLINICAL CASE

Brain Abscess and theNonfenestrated Fontan Circulation

Bethany G. Runkel, MD,a William B. Drake, MD, MS,b Geetha Raghuveer, MD, MPHc

ABSTRACT

L

ISS

Fro

Mi

rel

Th

sti

the

Ma

Brain abscess is a rare but life-threatening condition. Intracardiac or extracardiac right-to-left shunting in patients with

unrepaired cyanotic congenital heart disease, pulmonary arteriovenous malformations, or venovenous collaterals

allows microbes unfiltered access to the brain. Brain abscess must be considered when cyanotic patients present

with headache. (Level of Difficulty: Beginner.) (J Am Coll Cardiol Case Rep 2020;2:1029–32) © 2020 The Authors.

Published by Elsevier on behalf of the American College of Cardiology Foundation. This is an open access article under

the CC BY-NC-ND license (http://creativecommons.org/licenses/by-nc-nd/4.0/).

HISTORY OF PRESENTATION

A 15-year-old female patient, status post–nonfenestrated extracardiac Fontan, presented tothe emergency department with throbbing frontalheadache and malaise. She reported temporary painrelief with acetaminophen but had increasing head-ache intensity over the preceding 5 days. There wasno photophobia, phonophobia, head trauma, fever,

EARNING OBJECTIVES

To understand that patients with palliatedcongenital heart disease, pulmonary AVMs,and venovenous collaterals are at increasedrisk for brain abscess.To recognize clinical factors that shouldheighten the suspicion of brain abscess for-mation in such patients, including desatura-tion that is otherwise unexplained.To identify situations in which a lowerthreshold for brain imaging should beconsidered in cyanotic patients presentingwith headache.

N 2666-0849

m the aHeart Center, Children’s Mercy Kansas City, Kansas City, Misso

ssouri; and the cHeart Center, Children’s Mercy Kansas City, Kansas City, M

ationships relevant to the contents of this paper to disclose.

e authors attest they are in compliance with human studies committees

tutions and Food and Drug Administration guidelines, including patient co

JACC: Case Reports author instructions page.

nuscript received February 18, 2020; revised manuscript received April 2

nausea, or vomiting. Parents had observed increasedsomnolence for 1 day prior to presentation. Neuro-logic status in the emergency room was normal andhead imaging was not performed. The patient had anoxygen saturation of 82% and an otherwise reassur-ing examination. A complete blood count, hepaticpanel, ammonia level, and urinalysis were normal.Aside from mild hyponatremia, there was no signifi-cant electrolyte disturbance. Admission for observa-tion was considered, but her pain improved withacetaminophen and she was discharged home. After48 h, she returned with recurrent symptoms.Neurology recommended migraine treatment withintravenous fluids, prochlorperazine, and valproicacid, none of which resulted in improvement. Giventhe persistent headache, she was admitted to thegeneral pediatrics service, where she received sub-sequent doses of analgesics and antiemetics. Oxygensaturations remained in the 80s. An echocardiogramwas unchanged from her prior studies. Head imagingwas not performed, as her neurologic exam remainednonfocal. She was discharged home the following dayafter reporting improvement in headache but

https://doi.org/10.1016/j.jaccas.2020.04.036

uri; bKansas City Pediatric Cardiology, Kansas City,

issouri. All authors have reported that they have no

and animal welfare regulations of the authors’ in-

nsent where appropriate. For more information, visit

6, 2020, accepted April 28, 2020.

Page 50: Full Issue PDF · The JACC: Case Reports Editorial Board and ACC editorial office would like to express their sincere gratitude to the guest editors below who worked tirelessly …

FIGURE 1 Line Drawing Representing the Patient’s Complex

Cardiac Anatomy

ABBR EV I A T I ON S

AND ACRONYMS

AVM = arteriovenous

malformation

CT = computed tomography

MRI = magnetic resonance

imaging

Runkel et al. J A C C : C A S E R E P O R T S , V O L . 2 , N O . 7 , 2 0 2 0

Brain Abscess In Fontan J U N E 1 7 , 2 0 2 0 : 1 0 2 9 – 3 2

1030

returned 5 days later with fever, tachycardia,and lethargy. Her oxygen saturation was88%.

PAST MEDICAL HISTORY

Born with heterotaxy, polysplenia, and anunbalanced, right-dominant atrioventricular

Pulmonary blood flow is passively supplied by bilateral bidi-

rectional Glenn circuits and a nonfenestrated extracardiac

Fontan conduit while systemic cardiac output is provided by

pulmonary venous blood flow through the reconstructed

aortic arch. Not depicted are the patient’s pulmonary arte-

riovenous malformations and venovenous collateral vessels.

canal defect that was deemed unsuitable forbiventricular repair, the patient followed a singleventricle pathway, with Fontan palliation at 3 yearsof age using a 16-mm extracardiac nonfenestratedconduit (Figure 1). Diagnostic catheterization at 12years of age showed progressive Fontan baffleobstruction and small left-sided pulmonary arterio-venous malformations (AVMs). Thus, she underwentrevision with a 20-mm Fontan conduit. Repeatcardiac catheterization was performed at 13 years ofage to address pulmonary artery stenosis, and anetwork of small venovenous collaterals was noted.Device occlusion of a single large collateral(Figure 2) was performed at that time. She had noprior history of migraines, serious bacterial in-fections, or endocarditis. Two capped teeth wereextracted approximately 2 months prior to presen-tation due to adjacent gum swelling, and a courseof antibiotics had been completed.

DIFFERENTIAL DIAGNOSIS

The differential diagnosis for headache, fever, andlethargy includes intracranial mass such as abscess ortumor, ischemic or hemorrhagic stroke, medicationoverdose or accidental ingestion, and overwhelmingsepsis, among other possibilities. Migraine or tensionheadache, while considerations initially, are lesslikely in the setting of an obtunded patient with feverand tachycardia.

INVESTIGATIONS

After rapid assessment in the emergency departmenton the third visit, noncontrast head computed to-mography (CT) showed a right temporal lobe masswith vasogenic edema and midline shift. Leukocy-tosis was present, with a white blood cell count ofnearly 20,000, and a C-reactive protein level waselevated at 8.3 mg/dl. Blood cultures were drawn andhypertonic saline, corticosteroids, and levetiracetamwere given. Rapid-sequence magnetic resonance im-aging (MRI) showed a large, peripherally enhancinglesion measuring approximately 6 cm in diameter,consistent with brain abscess (Figure 3). The patientwas taken to the operating room, where emergentcraniectomy confirmed a purulent abscess.

Transthoracic and transesophageal echocardiographyshowed normal single-ventricle systolic function,patent Fontan and Glenn circuits, and no evidence ofthrombus or vegetation. A dental examination per-formed during hospitalization was not concerning foractive dental decay or infection. No primary source ofinfection was identified by clinical examination orimaging.

MANAGEMENT

Abscess cultures grew Streptococcus intermedius,which was treated with intravenous ceftriaxone andmetronidazole. Serial brain imaging over the courseof hospitalization showed improvement in midlineshift and eventual resolution of cerebral edema. Thepatient returned to her neurologic baseline approxi-mately 2 weeks after abscess drainage and completedan 8-week antibiotic course. Nine months after diag-nosis, she continues to do well but remains desatu-rated to the upper 80s and low 90s due to the fine

Page 51: Full Issue PDF · The JACC: Case Reports Editorial Board and ACC editorial office would like to express their sincere gratitude to the guest editors below who worked tirelessly …

FIGURE 2 Still Image From the Patient’s Cardiac Catheterization Approximately

1 Year Prior to Her Case Presentation

In this anteroposterior projection, contrast is seen filling a large, right-sided collateral

vessel that arises from the inferior vena cava and drains desaturated blood into the right-

sided pulmonary veins.

J A C C : C A S E R E P O R T S , V O L . 2 , N O . 7 , 2 0 2 0 Runkel et al.J U N E 1 7 , 2 0 2 0 : 1 0 2 9 – 3 2 Brain Abscess In Fontan

1031

venovenous collateral network that is not amenableto coil embolization.

DISCUSSION

Because of its rarity in the pediatric population,cardiogenic brain abscess has been not beenextensively studied. Current literature suggests thatright-to-left shunting resulting from unrepairedcyanotic congenital heart disease and pulmonaryAVMs increases the likelihood of brain abscess byallowing pathogens direct access to the systemicvasculature. Focal areas of tissue hypoxia in thebrain may also create a suitable environment forabscess formation (1). Adult patients with pulmo-nary AVMs are at increased risk of brain abscesscompared with the general population, and the riskincreases with more profound hypoxia (2). Venove-nous collaterals create similar routes for bacteria tobypass the filtering effects of the lungs. Both ofthese vascular malformations are associated withthe Fontan circulation.

Patients with brain abscess often present withnonspecific features. In a recent retrospective anal-ysis of 26 patients with complex congenital heartdisease, headache, and vomiting were the mostcommon presenting symptoms. Fever was present inabout 30% of patients, laboratory markers of infectionin 20%, and focal neurologic deficits in a mere 15% (3).When brain abscess is suspected, recommendationsinclude rapid acquisition of blood work, includingcomplete blood count, inflammatory markers, and ablood culture, as well as urgent brain CT. Lumbarpuncture may be performed if not contraindicated,and if rapid-sequence MRI is available, this modalitymay be helpful in delineating the lesion further. Athorough otolaryngology examination, cardiac eval-uation with echocardiography, skin inspection, anddental examination are recommended as well toidentify any primary source of infection. Chestroentgenogram or chest CT may also be performed ifthere is suspicion for primary lung disease (4).

This case demonstrates the challenge of diagnosingbrain abscess in pediatric patients with complexcongenital heart disease. The report of frontal head-ache responsive to analgesia alone unaccompanied byfocal neurologic deficits or fever seemed to result indelayed diagnosis, yet the decision to perform headimaging is sometimes difficult, given the effects ofrepeated radiation exposure over time. In retrospect,the patient’s increased somnolence could have beeninterpreted as a change in baseline neurologic status,prompting earlier head imaging in the setting ofcyanosis and persistent headache.

FOLLOW-UP

After successful completion of her antibiotic course,the patient experienced no clinical or radiographicevidence of recurrent infection. There were noobvious residual neurologic deficits, although shedeveloped unusual, painful sensations in her lowerextremities near the end of her antibiotic treatmentthat have improved over time. Contrast MRI of herlower spine was negative for meningeal or nerve rootenhancement, and the etiology of this neuropathyremains unclear. She continues to follow-upfrequently with her cardiologist, given her single-ventricle physiology.

CONCLUSIONS

Conventional teaching is that intracardiac right-to-left shunting resulting from partially palliatedcyanotic heart disease results in an increased risk forbrain abscess formation, but we often neglect toremember that, despite surgical separation of thesystemic venous and arterial circulations in the non-fenestrated Fontan circuit, pulmonary AVMs orvenovenous collaterals develop spontaneously overtime, increasing the risk for brain abscess. Lower-than-normal oxygen saturations in Fontan patientswithout an alternate etiology should alert the

Page 52: Full Issue PDF · The JACC: Case Reports Editorial Board and ACC editorial office would like to express their sincere gratitude to the guest editors below who worked tirelessly …

FIGURE 3 Images From the Patient’s Brain Magnetic Resonance Imaging With Contrast Upon Presentation

Axial, coronal, and sagittal cuts demonstrate a large peripherally enhancing mass centered in the right anterior temporal lobe. The mass measures approximately 5.5 �3.8 � 5.2 cm and up to 6.3 cm in greatest oblique anteroposterior dimension. There is internal restricted diffusion consistent with abscess. Marked surrounding

vasogenic edema is present, with resultant mass effect on surrounding structures and 0.9-cm midline shift.

Runkel et al. J A C C : C A S E R E P O R T S , V O L . 2 , N O . 7 , 2 0 2 0

Brain Abscess In Fontan J U N E 1 7 , 2 0 2 0 : 1 0 2 9 – 3 2

1032

physician to these conditions. Although rare, brainabscess should remain on the differential for anyFontan patient with persistent headache, especiallywhen accompanied by hypoxia and neurologicchanges.

ADDRESS FOR CORRESPONDENCE: Dr. Bethany G.Runkel, Children’s Mercy Kansas City, 2401Gillham Road, Kansas City, Missouri 64108.E-mail: [email protected].

RE F E RENCE S

1. Takeshita M, Kagawa M, Yonetani H, et al. Risk fac-tors for brain abscess in patients with congenitalcyanotic heart disease. Neurol Med Chir (Tokyo) 1992;32:667–70.

2. Boother EJ, Brownlow S, Tighe HC, Bamford KB,Jackson JE, Shovlin CL. Cerebral abscess associatedwith odontogenic bacteremias, hypoxemia, and iron

loading in immunocompetent patients with right-to-left shunting through pulmonary arteriovenousmalformations. Clin Infect Dis 2017;65:595–603.

3. Udayakumaran S, Onyia CU, Kumar RK.Forgotten? Not yet. Cardiogenic brain abscess inchildren: a case series-based review. World Neu-rosurg 2017;107:124–9.

4. Mameli C, Genoni T, Madia C, Doneda C,Penagini F, Zuccotti G. Brain abscess in pediatricage: a review. Childs Nerv Syst 2019;35:1117–28.

KEY WORDS brain abscess, cyanosis,Fontan

Page 53: Full Issue PDF · The JACC: Case Reports Editorial Board and ACC editorial office would like to express their sincere gratitude to the guest editors below who worked tirelessly …

J A C C : C A S E R E P O R T S VO L . 2 , N O . 7 , 2 0 2 0

ª 2 0 2 0 T H E A U T H O R S . P U B L I S H E D B Y E L S E V I E R O N B E H A L F O F T H E A M E R I C A N

C O L L E G E O F C A R D I O L O G Y F OU N D A T I O N . T H I S I S A N O P E N A C C E S S A R T I C L E U N D E R

T H E C C B Y - N C - N D L I C E N S E ( h t t p : / / c r e a t i v e c o mm o n s . o r g / l i c e n s e s / b y - n c - n d / 4 . 0 / ) .

CASE REPORT

CLINICAL CASE

A Tale of Fits and FaintA Case of Hypervagotonic Nerve Stimulation

Akshar Jaglan, DO, Arshad Jahangir, MD, Atul Bhatia, MD

ABSTRACT

L

ISS

Fro

Th

Th

ins

vis

Ma

Vagal nerve stimulators are devices used to treat medically refractory epilepsy. Here, we present a rare, unique, and

life-threatening side effect of vagal nerve stimulator placement. (Level of Difficulty: Advanced.)

(J Am Coll Cardiol Case Rep 2020;2:1033–5) © 2020 The Authors. Published by Elsevier on behalf of the

American College of Cardiology Foundation. This is an open access article under the CC BY-NC-ND license

(http://creativecommons.org/licenses/by-nc-nd/4.0/).

HISTORY OF PRESENTATION

A 33-year-old woman with no known cardiac diseasewas referred to our electrophysiology clinic for eval-uation of new-onset lightheadedness, dizziness, andsyncope. She states that her symptoms have becomemore frequent over the past few months. Movementdoes not worsen her symptoms, and they occurrandomly throughout the day.

MEDICAL HISTORY

The patient was known to have seizures since 16 yearsof age and underwent right temporal lobectomy for

EARNING OBJECTIVES

Physicians should recognize the possibilityof bradyarrhythmias as a cause of new-onset,unexplained syncope in patients who arebeing treated for epilepsy with the concom-itant use of a VNS.If symptom correlation with bradycardiaevents (i.e., sinus arrest, AV block) is docu-mented, such patients may benefit frompacemaker implantation.

N 2666-0849

m the Aurora Cardiovascular and Thoracic Services, Aurora Sinai/Aurora

e authors have reported that they have no relationships relevant to the c

e authors attest they are in compliance with human studies committe

titutions and Food and Drug Administration guidelines, including patien

it the JACC: Case Reports author instructions page.

nuscript received October 10, 2019; revised manuscript received January

seizures resistant to antiepileptic drug therapy. Aftersurgery, she continued to have breakthrough seizureson lamotrigine and lacosamide. An electroencepha-logram showed left anterior temporal sharp dis-charges as well as polyspike wave discharges. Forrecurrent seizures despite surgical and pharmaco-logical therapy, a vagal nerve stimulator (VNS) wasimplanted on the left pectoral site to modulateseizure foci. This approach was successful in con-trolling her seizures by activating vagal nerve stimu-lation with the onset of aura. After 2 years of beingseizure-free, post-VNS implant, she presented withrecurrent episodes of syncope and near-syncopewithout tonic-clonic movements.

DIFFERENTIAL DIAGNOSIS

While keeping a broad differential diagnosis, wewanted to rule out new seizures, as well as evaluateany arrhythmia causing the patient’s recurrentsyncope.

INVESTIGATION

A repeat electroencephalogram did not reveal anynew epileptiform abnormalities correlating with the

https://doi.org/10.1016/j.jaccas.2020.02.019

St. Luke’s Medical Centers, Milwaukee, Wisconsin.

ontents of this paper to disclose.

es and animal welfare regulations of the authors’

t consent where appropriate. For more information,

23, 2020, accepted February 19, 2020.

Page 54: Full Issue PDF · The JACC: Case Reports Editorial Board and ACC editorial office would like to express their sincere gratitude to the guest editors below who worked tirelessly …

FIGURE 1 Complete Timeline

The graphic depicts the chronolo

resolution of symptoms after th

ABBR EV I A T I ON S

AND ACRONYMS

AV = atrioventricular

EEG = electroencephalogram

mA = milliamps

VNS = vagal nerve stimula

Jaglan et al. J A C C : C A S E R E P O R T S , V O L . 2 , N O . 7 , 2 0 2 0

Hyper-Vagotonic Nerve Stimulation J U N E 1 7 , 2 0 2 0 : 1 0 3 3 – 5

1034

patient’s syncopal episodes. The VNS wasprogrammed to deliver periodic impulses fora 30-s duration with an intervening period ofno impulse delivered for 5 min throughout a24-h period. The programmed output was a1.5 milliamps (mA), magnet output of 1.75mA, with a pulse width of 500 ms and a signal

frequency of 30 Hz for the past 2 years, which effec-tively controlled her seizures.

The patient’s baseline electrocardiogram showednormal sinus rhythm with no conduction abnormal-ities. On continuous 24-h rhythm monitoring, notachyarrhythmias or bradyarrhythmias were detec-ted. Further VNS testing to assess stimulator outputrhythm correlation was performed as an outpatientwith an ambulatory cardiac telemetry monitor. At thetherapeutic VNS output settings (current output 1.5mA; magnet output 1.75 mA), the patient had

tor

of the Study Experiment

gical series of events that lead us to diagnose hypervagotonic ner

e pacemaker is placed and an example of the paced rhythm. AV

recurrent syncopal and near-syncopal episodes. Theambulatory cardiac telemetry monitor caught multi-ple 6.8-s pauses (Figure 1) correlating with thesyncopal and near-syncopal episodes. The findingssuggested marked hypervagotonic response withatrioventricular (AV) block leading to syncopal epi-sodes. The patient’s VNS parameters were reducedto a current output of 1.25 mA and magnet outputof 1.5 mA. No further syncopal episodes or AV blockwas observed at the lower VNS output; however,after 2 weeks, her prodromal seizure symptomsreappeared.

MANAGEMENT

Given the concerns regarding the reoccurrence of thepatient’s aura due to the reduction in the VNS output,and the obvious profound cardio-inhibition noted by

ve response to the vagal nerve stimulator (VNS). Also depicted is the

¼ atrioventricular; mA ¼ milliamps.

Page 55: Full Issue PDF · The JACC: Case Reports Editorial Board and ACC editorial office would like to express their sincere gratitude to the guest editors below who worked tirelessly …

J A C C : C A S E R E P O R T S , V O L . 2 , N O . 7 , 2 0 2 0 Jaglan et al.J U N E 1 7 , 2 0 2 0 : 1 0 3 3 – 5 Hyper-Vagotonic Nerve Stimulation

1035

higher output of VNS settings, a pacemaker implantwas recommended to allow the safe reprogrammingof higher output from the VNS for seizure control aswell as alleviating cardio-inhibition. After the pace-maker was placed, the current output strength wasset back to an optimal setting of 1.5 mA and themagnet output of 1.75 mA. The pacemaker’s pro-grammed sensed AV delay was initially kept short toenable constant ventricular pacing due to the severityof symptom recurrences while the VNS outputs werebeing adjusted under constant telemetry monitoring.We have since then reprogrammed the AV delay toallow for maximal intrinsic conduction. The patienttolerated these settings well, with no further heartblock with backup pacing to support ventricular rate(Figure 1). Episodes of paced rhythm during whatwould have been episodes of AV block have beendocumented. A timeline of our experiment is depic-ted in Figure 1.

DISCUSSION

VNS devices have been approved by the U.S. Foodand Drug Administration in the United States since1997 as ancillary treatment for patients with medi-cally refractory epilepsy (1). As of January 2016, thereare 133,000 known implanted VNS in patients (2). Theexact mechanism of how the VNS works is not trulyunderstood. The device is implanted in the left pec-toral site, and it provides high-frequency electricalstimulation to the left vagus nerve. The right vagusnerve has a greater effect on the sinoatrial node,whereas the left vagus nerve has a greater effect onthe AV node (3). Common side effects of the VNS areshortness of breath, hoarse voice, pain, nausea, andheadaches. It is believed that the effect of excessivevagal nerve stimulation, specifically to the afferentfibers, activates multiple systems and pathwayswithin the brain that prevent seizures, while efferentactivation of the vagus nerve by VNS is extremelyuncommon, with only a few case reports (4,5).

The current case presents a rare, potentiallylife-threatening side effect of VNS on the cardiacconduction system involving the AV nodes late afterVNS implantation. The treatment of choice in thissituation is to either decrease the current output orstop the VNS altogether; however, this approachis fraught with the danger of the reoccurrence ofseizures, which was evident in our patient. Hence,the choice of permanent pacemaker was helpful inalleviating both the cardiac and neurological symp-toms with concomitant use of VNS and backuppacing.

FOLLOW-UP

The patient has been followed up in the electro-physiology clinic for 6 years since the implantation ofthe pacemaker. The patient is now 39 years of age andhas had no further seizures or syncopal episodes.

CONCLUSIONS

Cardiac dysrhythmias are rare secondary to VNSplacement for control of seizures. In these patients,both the neurologist and electrophysiologist mustwork closely together to choose a treatment plan thatis most beneficial to the patient. An attempt should bemade to decrease the frequency or discontinue theVNS if possible.

ACKNOWLEDGMENTS The authors are grateful toJennifer Pfaff and Susan Nord of Aurora Cardiovas-cular and Thoracic Services for editorial preparationof the manuscript and Brian Miller and Brian Schurrerof Advocate Aurora Research for assistance with thefigures.

ADDRESS FOR CORRESPONDENCE: Dr. Atul Bhatia,Aurora St. Luke’s Medical Center, 2801 W. Kinnick-innic River Parkway, Suite 880, Milwaukee, Wiscon-sin 53215. E-mail: [email protected].

RE F E RENCE S

1. Schachter SC. Vagus nerve stimulationtherapy summary: five years after FDAapproval. Neurology 2002;59 Suppl 4:S15–20.

2. Roberts C, Bullis C. Vagus nerve stimulation. In:Raslan AM, Burchiel KJ, editors. FunctionalNeurosurgery and Neuromodulation. St. Louis,MO: Elsevier Inc., 2018:109–12.

3. Panebianco M, Rigby A, Weston J,Marson AG. The Cochrane Database of System-atic Reviews. London, England: Wiley, August2019. Available at: https://www.cochranelibrary.com/search. Accessed August 10, 2019.

4. Tomova GS, Do DH, Krokhaleva Y,Honda H, Buch E, Boyle NG. ’Runaway’vagal nerve stimulator: a case of cyclic

asystole. HeartRhythm Case Rep 2016;2:388–90.

5. Ratajczak T, Blank R, Parikh A,WaseA. Late-onsetasystolic episodes in a patient with a vagal nervestimulator. HeartRhythm Case Rep 2018;4:314–7.

KEY WORDS atrioventricular block,pacemaker, vagal nerve stimulator

Page 56: Full Issue PDF · The JACC: Case Reports Editorial Board and ACC editorial office would like to express their sincere gratitude to the guest editors below who worked tirelessly …

J A C C : C A S E R E P O R T S V O L . 2 , N O . 7 , 2 0 2 0

ª 2 0 2 0 T H E A U T H O R S . P U B L I S H E D B Y E L S E V I E R O N B E H A L F O F T H E AM E R I C A N

C O L L E G E O F C A R D I O L O G Y F O U N DA T I O N . T H I S I S A N O P E N A C C E S S A R T I C L E U N D E R

T H E C C B Y - N C - N D L I C E N S E ( h t t p : / / c r e a t i v e c o mm o n s . o r g / l i c e n s e s / b y - n c - n d / 4 . 0 / ) .

CASE REPORT

CLINICAL CASE

Catheter-Directed MechanicalThrombectomy in Massive PulmonaryEmbolism With Cardiogenic Shock

Manju Bengaluru Jayanna, MBBS, MS, Tariq Ali Ahmad, MD, Marc Maalouf, MD, Arthur Omondi, MD,Rachel Bobby, DO, Mara Caroline, MD, Eric M. Gnall, DO

ABSTRACT

L

ISS

Fro

wo

rel

Th

sti

the

Ma

We discuss a patient who presented with cardiogenic shock secondary to massive pulmonary embolism and right ven-

tricular failure. She was managed by a multidisciplinary heart team and treated with catheter-directed thrombectomy,

followed by ProtekDuo (Tandem [Liva Nova], London, United Kingdom) heart percutaneous right ventricular support

leading tocomplete recovery fromthisoften fatal condition. (LevelofDifficulty: Intermediate.) (JAmColl Cardiol CaseRep

2020;2:1036–41) © 2020 The Authors. Published by Elsevier on behalf of the American College of Cardiology Foundation.

This is an open access article under the CC BY-NC-ND license (http://creativecommons.org/licenses/by-nc-nd/4.0/).

HISTORY OF PRESENTATION

A 72-year-old woman presented to the emergencydepartment (ED) with symptoms of rapidly worseningshortness of breath over the past 36 to 48 h. Hersymptoms had progressed rapidly from her usualstate of health to not being able to take a couple ofsteps and to significant dyspnea at even at rest. She

EARNING OBJECTIVES

To make a rapid diagnosis of PE with hemo-dynamic instability by multidisciplinary team(PERT approach), reducing invasive diag-nostic tests, allowing rapid diagnosis, andapplying advanced therapies with likelyreduction in morbidity and mortality.To understand catheter-directed mechanicalthrombectomy use in the setting of massivePE without the risk of critical and fatalhemorrhage associated with thrombolysis.

N 2666-0849

m the Division of Cardiovascular Diseases, Lankenau Medical Center an

od, Pennsylvania. Ashwin Ravichandran, MD, served as Guest Editor for th

ationships relevant to the contents of this paper to disclose.

e authors attest they are in compliance with human studies committees

tutions and Food and Drug Administration guidelines, including patient co

JACC: Case Reports author instructions page.

nuscript received February 24, 2020; revised manuscript received April 2

had no significant chest pain. Physical examinationrevealed the following: blood pressure, 103/56 mm Hg; heart rate, 118 beats/min; respiratory rate,22 breaths/min; and hypoxia with oxygen saturationin the low 80s with respiratory distress. Her lungfields were clear to auscultation, and there was nosignificant evidence of volume overload byexamination.

MEDICAL HISTORY

She had a known history of immunoglobulin G kappamultiple myeloma treated with lenalidomide anddexamethasone, previous history of pulmonary em-bolism (PE), hypertension, hypothyroidism, and dia-betes mellitus. She had completed 1 year ofanticoagulant therapy for a previous episode of PEapproximately 9 years earlier and was not currentlyreceiving anticoagulant therapy. Her other significantmedications included amlodipine, glimepiride, levo-thyroxine, and losartan.

https://doi.org/10.1016/j.jaccas.2020.04.031

d Lankenau Institute for Medical Research, Wynne-

is paper. All authors have reported that they have no

and animal welfare regulations of the authors’ in-

nsent where appropriate. For more information, visit

6, 2020, accepted April 28, 2020.

Page 57: Full Issue PDF · The JACC: Case Reports Editorial Board and ACC editorial office would like to express their sincere gratitude to the guest editors below who worked tirelessly …

AB BR E V I A T I O N S

AND ACRONYM S

AKI = acute kidney injury

CT = computed tomography

ED = emergency department

IVC = inferior vena cava

PA = pulmonary artery

PE = pulmonary embolism

PERT = pulmonary embolism

response team

RV = right ventricular

TTE = transthoracic

echocardiography

J A C C : C A S E R E P O R T S , V O L . 2 , N O . 7 , 2 0 2 0 Jayanna et al.J U N E 1 7 , 2 0 2 0 : 1 0 3 6 – 4 1 Thrombectomy in Pulmonary Embolism With Shock

1037

DIFFERENTIAL DIAGNOSIS

The differential diagnosis included PE, decom-pensated heart failure, pneumonia, and atypical pre-sentation of acute coronary syndrome.

INVESTIGATIONS

On presentation, the electrocardiogram showed si-nus tachycardia, new right bundle branch block, aqR pattern in lead V1, and T-wave inversions inleads V1 and V2 (Figure 1A). Chest radiographyshowed no acute cardiopulmonary disease. Labora-tory study results were significant for the following:mild troponin T elevation, 0.19 ng/ml (referencerange <0.005 ng/ml); elevated lactate, 5.1 mmol/l,with combined metabolic and respiratory acidosison arterial blood gas determination with pH 7.19;partial pressure of carbon dioxide, 42 mm Hg;serum bicarbonate, 14 mEq/l; and creatinine,2.7 mg/dl, compared with baseline creatinine of1.0 mg/dl, indicating acute kidney injury (AKI). Ur-gent bedside transthoracic echocardiography (TTE)showed a small hyperdynamic left ventricular cavitywith hyperdynamic systolic function with an esti-mated ejection fraction >75%, a severely dilatedright ventricle with severely reduced right ventric-ular (RV) systolic function and a hyperdynamic RVapex (the McConnell sign), moderate to severetricuspid regurgitation with elevated estimated RVsystolic pressure of 50 to 55 mm Hg, and a dilatedinferior vena cava (IVC).

MANAGEMENT

The patient was evaluated by a multidisciplinaryheart team comprising ED, pulmonary and criticalcare, and interventional cardiology physicians. Onthe basis of clinical presentation and TTE imaging, adiagnosis of cardiogenic shock secondary to RVfailure resulting from massive PE was made.Because of the patient’s AKI and unstable hemo-dynamics, the multidisciplinary team decided to notpursue confirmatory computed tomography (CT)with angiography. She was treated with intravenousheparin for anticoagulation and underwent emer-gency pulmonary angiography and intervention inthe cardiac catheterization laboratory. She wasevaluated by the multidisciplinary team within60 min, and bedside TTE was performed within90 min. The time from presentation to the EDand the start of mechanical thrombectomy was<120 min.

Right-sided heart catheterization was per-formed through a right femoral veinapproach. The pulmonary angiogram showeda large clot burden in the right pulmonaryartery (PA). The right femoral venous sheathwas up-sized to a 22-F DrySeal sheath (W.L.Gore & Associates, Newark, Delaware). AnAmplatz Super Stiff wire (Boston Scientific,Natick, Massachusetts) was placed underfluoroscopic guidance in the right PA, and aFlowTriever catheter (Inari Medical, Irvine,California) was advanced, followed by suc-cessful mechanical aspiration of large clotfragments and excellent flow noted on the

angiogram. Subsequently, a pulmonary angiogram ofthe left PA showed complete obstruction of flow(Figure 2A). Several clot aspirations were similarlyperformed. A large clot was noticed on the tip of theFlowTriever cannula; this clot was unable to bemoved despite disk deployment. With negative suc-tion maintained, the clot was pulled down to the iliacvein, and an infrarenal IVC filter was placed through aleft jugular vein approach. A repeat angiogram of theleft PA showed the establishment of significant flow(Figure 2B).

The procedure was complicated by hemoptysisrequiring reversal of anticoagulation with protamine.Emergency intubation, mechanical ventilation, andflexible bronchoscopy were performed, with no activebleeding. Small to moderate-sized blood clots wereseen in the airways, without obstruction. These clotswere removed with saline wash and suctioning.

The patient continued to have persistent hypo-tension, tachycardia with narrow pulse pressure, andRV dysfunction noted on TTE. RV failure was treatedwith a ProtekDuo (Tandem [Liva Nova], London,United Kingdom) heart percutaneous RV assist de-vice. The device was placed through a right internaljugular access, with inotropic support and an epo-prostenol infusion. She had an episode of atrialfibrillation with rapid ventricular response requiringDC cardioversion, and sinus rhythm was maintainedwith an amiodarone infusion.

FOLLOW-UP

Follow-up TTE within 24 h showed significantimprovement, with only mild RV dilation and mod-erate systolic dysfunction (Figure 3B). Her hemody-namics improved over 48 h, and she was successfullyweaned from the RV assist device and epoprostenol.Repeat bronchoscopy showed no evidence of

Page 58: Full Issue PDF · The JACC: Case Reports Editorial Board and ACC editorial office would like to express their sincere gratitude to the guest editors below who worked tirelessly …

FIGURE 1 Initial and Post-Procedure ECGs

(A) Electrocardiogram (ECG) on presentation showing sinus tachycardia, right bundle branch block, and a qR pattern in lead V1. (B) Elec-

trocardiogram 24 h post-procedure showing resolution of sinus tachycardia, right bundle branch block, and the qR pattern in lead V1.

Jayanna et al. J A C C : C A S E R E P O R T S , V O L . 2 , N O . 7 , 2 0 2 0

Thrombectomy in Pulmonary Embolism With Shock J U N E 1 7 , 2 0 2 0 : 1 0 3 6 – 4 1

1038

bleeding, and she was extubated in 72 h, followed bysignificant improvement in clinical status. She wasdischarged within 1 week of admission on an antico-agulant regimen with rivaroxaban. Follow-up TTE5 weeks post-discharge showed normal RV size andfunction with mild tricuspid regurgitation and RVsystolic pressure of 26 mm Hg (Figures 3A to 3F, whichalso show the patient’s initial findings for compari-son). An IVC filter was removed in 3 months.

DISCUSSION

The PE response team (PERT) approach to patientswho present with intermediate- to high-risk PE hasbeen described to provide rapid access to advancedtherapies (1,2). Our patient presented with clinicalmanifestations consistent with massive PE withseveral features suggestive of adverse prognosis,including sinus tachycardia, new right bundle

Page 59: Full Issue PDF · The JACC: Case Reports Editorial Board and ACC editorial office would like to express their sincere gratitude to the guest editors below who worked tirelessly …

FIGURE 2 Initial and Post-Procedure Angiograms

(A) Pulmonary angiogram showing complete obstruction of the left pulmonary artery. (B) Pulmonary angiogram showing re-establishment of

flow in the left pulmonary artery following mechanical thrombectomy.

J A C C : C A S E R E P O R T S , V O L . 2 , N O . 7 , 2 0 2 0 Jayanna et al.J U N E 1 7 , 2 0 2 0 : 1 0 3 6 – 4 1 Thrombectomy in Pulmonary Embolism With Shock

1039

branch block, a qR pattern in lead V1, an elevatedtroponin T level, RV dilatation, hypokinesis, theMcConnell sign, severe tricuspid regurgitation, andevidence of pulmonary hypertension on TTE. TheMcConnell sign has a 94% specificity for the diag-nosis of acute PE, and with the clinical presentationsuggesting a high pre-test probability of PE, confir-matory testing with CT angiography was not per-formed (3). Acute massive or submassive PE has anexceedingly high mortality rate, and an elevatedlactate level is a strong independent predictor ofmortality (4,5). The magnitude of lactate elevationcorrelates with the risk of death and is >35% forlactate levels higher than 5 mmol/l (5). Systemicthrombolysis has been the recommended treatmentof choice in this setting; however, it is associatedwith major systemic bleeding, including a risk ofintracranial hemorrhage (6,7).

Catheter-directed mechanical thrombectomy withthe FlowTriever Retrieval/Aspiration System hasbeen found to be beneficial in intermediate-risk PE(8). This case demonstrates the use of this system

in a patient with PE with cardiogenic shock.Thrombectomy was performed by engaging thethrombus, disrupting it with self-expanding nitinoldisks, and extracting it by simultaneously aspiratingand withdrawing it through a 20-F guide catheter,with appropriate mechanical circulatory supportused for shock. Hemoptysis is a known complicationof PE as a result of ischemic pulmonary paren-chymal necrosis. Although an obvious site ofbleeding could not be identified on bronchoscopy,wire perforation as a cause of hemoptysis could notbe ruled out and should be recognized as a poten-tial complication of the procedure. In this case, thePERT approach with multidisciplinary expertsallowed treatment without confirmatory CT angi-ography and immediate treatment with advancedtherapies, thus reducing the time to treatment. Atotal of 70 ml of iodine contrast material was usedfor the procedure. The CT angiogram PE protocolrequires approximately 60 to 150 ml of iodinecontrast material (9). Hence possible harm fromadditional iodine contrast medium exposure–

Page 60: Full Issue PDF · The JACC: Case Reports Editorial Board and ACC editorial office would like to express their sincere gratitude to the guest editors below who worked tirelessly …

FIGURE 3 Transthoracic Echocardiograms

(A to F) Transthoracic echocardiograms on presentation and at follow-up. (A, C, E) Initial images: Severe right ventricular dilatation, severe

tricuspid regurgitation with elevated pulmonary artery pressure, and dilated inferior vena cava. (B, D, F) Follow-up images: Normal right

ventricular size, mild tricuspid regurgitation with normal pulmonary artery pressure, and normal-size inferior vena cava. Grad ¼ gradient;

Max ¼ maximum; PG ¼ peak gradient; TR ¼ tricuspid regurgitation; Vmax ¼ maximum velocity.

Jayanna et al. J A C C : C A S E R E P O R T S , V O L . 2 , N O . 7 , 2 0 2 0

Thrombectomy in Pulmonary Embolism With Shock J U N E 1 7 , 2 0 2 0 : 1 0 3 6 – 4 1

1040

Page 61: Full Issue PDF · The JACC: Case Reports Editorial Board and ACC editorial office would like to express their sincere gratitude to the guest editors below who worked tirelessly …

J A C C : C A S E R E P O R T S , V O L . 2 , N O . 7 , 2 0 2 0 Jayanna et al.J U N E 1 7 , 2 0 2 0 : 1 0 3 6 – 4 1 Thrombectomy in Pulmonary Embolism With Shock

1041

induced renal dysfunction in the setting of AKI wasavoided.

CONCLUSIONS

This case highlights a multidisciplinary approach(PERT) to the use of catheter-directed thrombectomyin massive PE with cardiogenic shock, with anexcellent long-term outcome. This approach avoids

the risk of critical and fatal hemorrhage associatedwith thrombolysis.

ADDRESS FOR CORRESPONDENCE: Dr. Manju Ben-galuru Jayanna, Lankenau Medical Center and Lan-kenau Institute for Medical Research, 100 EastLancaster Avenue, 356 MOB East, Wynnewood,Pennsylvania 19096. E-mail: [email protected].

RE F E RENCE S

1. Rosovsky R, Zhao K, Sista A, Rivera-Lebron B,Kabrhel C. Pulmonary embolism response teams:purpose, evidence for efficacy, and future researchdirections. Res Pract Thromb Haemost 2019;3:315–30.

2. Mahar JH, Haddadin I, Sadana D, et al.A pulmonary embolism response team (PERT)approach: initial experience from the ClevelandClinic. J Thromb Thrombolysis 2018;46:186–92.

3. McConnell MV, Solomon SD, Rayan ME,Come PC, Goldhaber SZ, Lee RT. Regional rightventricular dysfunction detected by echocardiog-raphy in acute pulmonary embolism. Am J Cardiol1996;78:469–73.

4. Kucher N, Rossi E, De Rosa M, Goldhaber SZ.Massive pulmonary embolism. Circulation 2006;113:577–82.

5. Vanni S, Viviani G, Baioni M, et al. Prognostic valueof plasma lactate levels among patients with acutepulmonary embolism: the thrombo-embolism lactateoutcome study. Ann Emerg Med 2013;61:330–8.

6. Jaff MR, McMurtry MS, Archer SL, et al. Man-agement of massive and submassive pulmonaryembolism, iliofemoral deep vein thrombosis, andchronic thromboembolic pulmonary hypertension:a scientific statement from the American HeartAssociation. Circulation 2011;123:1788–830.

7. Marti C, John G, Konstantinides S, et al. Sys-temic thrombolytic therapy for acute pulmonary

embolism: a systematic review and meta-analysis.Eur Heart J 2015;36:605–14.

8. Tu T, Toma C, Tapson VF, et al. A prospective,single-arm, multicenter trial of catheter-directedmechanical thrombectomy for intermediate-riskacute pulmonary embolism: the FLARE study.J Am Coll Cardiol Intv 2019;12:859–69.

9. Moore AJE, Wachsmann J, Chamarthy MR,Panjikaran L, Tanabe Y, Rajiah P. Imaging of acutepulmonary embolism: an update. Cardiovasc DiagnTher 2018;8:225–223.

KEY WORDS pulmonary hypertensionright-sided catheterization, thrombosis

Page 62: Full Issue PDF · The JACC: Case Reports Editorial Board and ACC editorial office would like to express their sincere gratitude to the guest editors below who worked tirelessly …

J A C C : C A S E R E P O R T S V O L . 2 , N O . 7 , 2 0 2 0

P U B L I S H E D B Y E L S E V I E R O N B E H A L F O F T H E A M E R I C A N C O L L E G E O F

C A R D I O L O G Y F O UN DA T I O N . T H I S I S A N O P E N A C C E S S A R T I C L E U N D E R T H E

C C B Y - N C - N D L I C E N S E ( h t t p : / / c r e a t i v e c o mm o n s . o r g / l i c e n s e s / b y - n c - n d / 4 . 0 / ) .

CASE REPORT

CLINICAL CASE

Acute Tubular Injury in a Patient on aProprotein Convertase Subtilisin/KexinType 9 Inhibitor

June K. Pickett, MD,a Maulin Shah, MD,a,b,c Michael Gillette, PHARMD,b Peter Jones, MD,a Salim Virani, MD, PHD,a,b

Christie Ballantyne, MD,a Vijay Nambi, MD, PHDa,b,d

ABSTRACT

L

ISS

Fro

Ho

Va

pri

co

He

of

Du

su

Re

Ar

Me

1I0

Ba

au

Th

ins

vis

Ma

A 72-year-old man with coronary artery disease, statin intolerance, and chronic kidney disease stage IIIa was initiated on

alirocumab, a proprotein convertase subtilisin/kexin type 9 inhibitor, and developed acute kidney injury. A kidney biopsy

was performed and suggested acute tubular injury. The serum creatinine returned to baseline after discontinuation

of alirocumab. (Level of Difficulty: Intermediate.) (J Am Coll Cardiol Case Rep 2020;2:1042–5) Published by Elsevier on

behalf of the American College of Cardiology Foundation. This is an open access article under the CC BY-NC-ND license

(http://creativecommons.org/licenses/by-nc-nd/4.0/).

A 72-year-old man with a past medical historyof coronary artery disease (CAD), atrial fibril-lation, diabetes mellitus, chronic kidney dis-

ease (CKD) IIIa, esophageal cancer in remission,

EARNING OBJECTIVE

To consider acute kidney injury as a potentialrare side effect of PCSK9-I.

N 2666-0849

m the aDepartment of Medicine, Baylor College of Medicine, Houston, T

uston, Texas; cSection of Nephrology, Baylor College of Medicine, Houst

scular Medicine, Methodist DeBakey Heart and Vascular Center, Methodi

ncipal investigator for a Leonard-Meron Biosciences, Inc-sponsored tr

nsultant for Esperion and Amarin. Dr. Virani has received research supp

art Federation, Jooma and Tahir Family Honorarium, and serves as Associ

Cardiology; and is a Steering Committee member, Patient and Provider Ass

ke Clinical Research Institute, for which he receives no financial remunera

pport paid to Baylor Medical College; has received support from Abb

generon, Roche Diagnostic, NIH, AHA, and ADA; and has served as consu

rowhead, Astra Zeneca, Boehringer Ingelheim, Corvidia, Denka Seiken,

rck, Novartis, Novo Nordisk, Regeneron, Roche Diagnostic, and Sanofi-S

1CX001112-01; and is named on provisional patent no. 61721475 Biomarker

ylor College of Medicine and Roche; and is the site principal investigator f

thors have reported that they have no relationships relevant to the conte

e authors attest they are in compliance with human studies committe

titutions and Food and Drug Administration guidelines, including patien

it the JACC: Case Reports author instructions page.

nuscript received February 20, 2020; revised manuscript received April 2

hyperlipidemia, and statin intolerance was seen inthe cardiology clinic for his CAD. The patient wasasymptomatic and, overall, doing well. His medica-tions included losartan, aspirin, ezetimibe, apixaban,diltiazem, furosemide, omeprazole, and gabapentin.He was a nonsmoker.

Physical examination demonstrated a blood pres-sure of 125/75 mm Hg and normal cardiovascular,respiratory, abdominal, and neurological examina-tion. Laboratory examination was significant for a

https://doi.org/10.1016/j.jaccas.2020.04.039

exas; bMichael E DeBakey Veterans Affairs Hospital,

on, Texas; and the dDivision of Atherosclerosis and

st Hospital, Houston, Texas. Dr. Shah has served as

ial. Dr. Jones has served as a scientific advisory

ort from the Department of Veterans Affairs, World

ate Editor for Innovations, acc.org, American College

essment of Lipid Management (PALM registry) at the

tion. Dr. Ballantyne has received grant and research

ott Diagnostic, Akcea, Amgen, Esperion, Novartis,

ltant for Abbott Diagnostics, Akcea, Amarin, Amgen,

Esperion, Gilead, Janssen, Matinas BioPharma Inc.,

ynthelabo. Dr. Nambi has received VA MERIT grant

s to Improve Prediction of Heart Failure Risk, filed by

or a study sponsored by Merck and Amgen. All other

nts of this paper to disclose.

es and animal welfare regulations of the authors’

t consent where appropriate. For more information,

4, 2020, accepted April 28, 2020.

Page 63: Full Issue PDF · The JACC: Case Reports Editorial Board and ACC editorial office would like to express their sincere gratitude to the guest editors below who worked tirelessly …

FIGURE 1 Light Microscopy With Trichrome Staining

The orange arrow shows tubular injury as evidenced by dilated

tubules, flattened epithelium due to loss of brush border.

Blue staining in the interstitium indicated interstitial fibrosis

and atrophy (IFTA). In this biopsy, there is moderate IFTA of

approximately 20% to 50%.

AB BR E V I A T I O N S

AND ACRONYM S

AKI = acute kidney injury

ASCVD = atherosclerotic

cardiovascular disease

CAD = coronary artery disease

CKD = chronic kidney disease

IFTA = interstitial fibrosis and

atrophy

LDL-C = low-density

lipoprotein-cholesterol

PCSK9-I = proprotein

convertase subtilisin/kexin

type 9 inhibitor

SCr = serum creatinine

J A C C : C A S E R E P O R T S , V O L . 2 , N O . 7 , 2 0 2 0 Pickett et al.J U N E 1 7 , 2 0 2 0 : 1 0 4 2 – 5 Acute Tubular Injury in a Patient on a PCSK9-I

1043

low-density lipoprotein-cholesterol (LDL-C) of163 mg/dl on ezetimibe therapy. Patient noted he hadbeen on simvastatin and atorvastatin previously;however, he did not tolerate these medicationsbecause of severe leg weakness and subsequentfunctional limitation and did not want to initiate anystatin at any dose. Given his CAD and poorlycontrolled hyperlipidemia in the setting of statinintolerance, he was initiated on alirocumab, a pro-protein convertase subtilisin/kexin type 9 inhibitor(PCSK9-I). Two months after being on alirocumab150 mg every 2 weeks, he had a decline in renalfunction, with his baseline serum creatinine (SCr) ofapproximately 1.3 mg/dl (estimated glomerularfiltration rate [eGFR] w55 ml/min/1.73 m2), increasingto 2.3 mg/dl. Upon outpatient evaluation, the patientnoted that he was in his usual state of health and hadno new complaints or interval events. He hadreceived no iodinated contrast material and had noknown medication changes other than addition ofalirocumab. He was referred to nephrology for furtherassessment.

PAST MEDICAL HISTORY

The patient has a history of stable CAD, permanentatrial fibrillation, diabetes mellitus, familial hyper-cholesterolemia (LDL- C had been as high as 217 mg/dlin the past), CKDIIIa, and esophageal cancer inremission. Last coronary angiography done forsymptoms of exertional dyspnea in 2018 revealed 50%stenosis of left anterior descending (LAD) artery with

a fractional flow reserve of 0.91 as well as 70%mid-vessel stenosis of a small right coronaryartery.

DIFFERENTIAL DIAGNOSIS

The differential diagnosis for the acute kid-ney injury (AKI) in this patient after initialphysical examination and preliminary labo-ratory data (see the following) included uratenephropathy, glomerulonephritis, interstitialnephritis, or toxic mediated acute tubularinjury.

INVESTIGATIONS

Urinalysis revealed subnephrotic proteinuria (urineprotein 30 mg/dl), microscopic hematuria (3 red bloodcells per high power field) and pyuria (1 white bloodcell per high power field). Additional work-uprevealed an elevated serum uric acid of 20 mg/dl.Allopurinol was started, and uric acid improved to9.0 mg/dl within 1 month. However, the SCr did notimprove after reduction and stabilization of uric acidand discontinuation of losartan; therefore, the pa-tient’s nephrologist recommended a kidney biopsy.The kidney biopsy (Figures 1 and 2) revealed dilatedtubules and flattened epithelium from loss of prox-imal tubular cell brush border, consistent with acutetubular injury. There was no evidence of uric acidcrystals on biopsy. There were also thickenedglomerular basement membranes, nodular mesangialmatrix expansion, and hyaline material within arte-riole vessel walls, which are features consistent witha background of diabetic nephropathy.

MANAGEMENT

Given the time course and no other clear explanationof persistent AKI after discontinuation of medicationsand improvement in uric acid, it was suspected thatalirocumab may be contributing to the cause of theacute tubular injury. Alirocumab was discontinued,and the serum creatinine subsequently improved andreturned to a baseline creatinine of 1.3 mg/dl inapproximately 3 months (Figure 3).

DISCUSSION

The American Heart Association/American College ofCardiology Multisociety 2018 Cholesterol Guidelineshave recommended as a Class I indication the initia-tion of high -intensity or maximal statin therapy forthose with atherosclerotic cardiovascular disease(ASCVD). They also recommend that if LDL-C is

Page 64: Full Issue PDF · The JACC: Case Reports Editorial Board and ACC editorial office would like to express their sincere gratitude to the guest editors below who worked tirelessly …

FIGURE 2 Light Microscopy With Methenamine

Silver-Periodic Acid-Schiff (Jones) Stain

The blue arrow shows glomeruli have thickened glomerular

basement membrane. The yellow arrow shows evidence of

mesangial matrix expansion. The green arrow shows hyaline

material within arteriole vessel walls.

Pickett et al. J A C C : C A S E R E P O R T S , V O L . 2 , N O . 7 , 2 0 2 0

Acute Tubular Injury in a Patient on a PCSK9-I J U N E 1 7 , 2 0 2 0 : 1 0 4 2 – 5

1044

>70 mg/dl on maximally tolerated statin, it is appro-priate to initiate ezetimibe in very high-risk patientswith ASCVD. If, after this patient is still in needof LDL-C reduction, it is a Class IIa indication toinitiate PCSK9-I, as was done in our patient (LDL-C of163 mg/dl) (1).

In our review of literature, there has been only 1other report of alirocumab-induced kidney injury in apatient with stage IV CKD as well as a report of apatient with normal renal function who developed

FIGURE 3 Time Frame of Kidney Injury Related to Alirocumab Initia

AKI after receiving an experimental drug SPC5001, anantisense oligonucleotide directed against PCSK9(2,3). The mechanisms of kidney injury were unclearin these cases, and both were managed conserva-tively. In the clinical trials that have evaluated alir-ocumab and evolocumab, they have been welltolerated, in general, with the most common reportedadverse events leading to discontinuation beinginjection-site reactions, myalgias, neurocognitiveevents, and ophthalmologic events. Additional side-effects include nasopharyngitis, hypersensitivityvasculitis, and influenza-like symptoms. There are noreports of AKI in the alirocumab safety trials,although subjects with GFRs of <30 ml/min/m2,HbA1c >10.0% were excluded (4). It is unclear if themechanism of AKI with alirocumab in our case isdirect toxicity or if the patient was predisposed to AKIfrom another cause (5). However, in general, patientsat risk for drug-induced nephrotoxicity include thosewith CKD, diabetes, intravascular volume depletion,congestive heart failure, and sepsis (5).

FOLLOW-UP

The SCr slowly returned to baseline 3 months afterdiscontinuation of alirocumab, and his LDL-C was118 mg/dl. The LDL-C after 8 doses (approximately16 weeks) of alirocumab was 94 mg/dl. Thus, webelieve that the LDL-C of 118 mg/dl after discontinu-ation of alirocumab was likely reflective of the re-sidual effect of medication and that the LDL-C willcontinue to increase. The patient is hesitant to

tion

Page 65: Full Issue PDF · The JACC: Case Reports Editorial Board and ACC editorial office would like to express their sincere gratitude to the guest editors below who worked tirelessly …

J A C C : C A S E R E P O R T S , V O L . 2 , N O . 7 , 2 0 2 0 Pickett et al.J U N E 1 7 , 2 0 2 0 : 1 0 4 2 – 5 Acute Tubular Injury in a Patient on a PCSK9-I

1045

initiate a statin, even at lower doses, given previoussymptoms of severe leg weakness, and he continuesto take ezetimibe. We will consider the addition ofbempedoic acid when it is available. Alternatively,after discussion with nephrology, we have consideredinitiating evolocumab instead of alirocumab, with theconsideration that some idiosyncratic reactionsrelated to other constituents that make up alirocumabmay have played a role in the renal injury that maynot be related to the PCSK9-I. If he is startedon evolocumab, we would monitor kidney functionearlier in treatment and more frequently. However,we are waiting 3 months after stabilization of renalfunction before considering evolocumab.

CONCLUSIONS

This case highlights the temporal development ofAKI—specifically, acute tubular injury—after initiation

of the PCSK9-I alirocumab. The time course and res-olution of AKI after stopping alirocumab suggestedthe AKI might have been associated with the PCSK9-I,although the mechanism is unclear. Given the morefrequent use of the PCSK9-I, clinicians should beaware of this rare possible side effect in patients athigh risk for AKI, especially as the effects will belonger term. Subsequent management strategies forhyperlipidemia will need to involve clinician-patientdiscussion with consideration of alternate therapies.Additional studies should be considered to study theefficacy and safety of PCSK9-I in patients with moreadvanced CKD.

ADDRESS FOR CORRESPONDENCE: Dr. Vijay Nambi,Baylor College of Medicine, 6565 Fannin Street, MSA601/STE B160, Houston, Texas 77030. E-mail:[email protected].

RE F E RENCE S

1. Grundy SM, Stone NJ, Bailey AL, et al. 2018AHA/ACC/AACVPR/AAPA/ABC/ACPM/ADA/AGS/APhA/ASPC/NLA/PCNA guideline on themanagement of blood cholesterol: execu-tive summary: a report of the AmericanCollege of Cardiology/American Heart As-sociation Task Force on Clinical PracticeGuidelines. J Am Coll Cardiol 2019;73:3168–209.

2. Jhaveri KD, Barta VS, Pullman J. Praluent (alir-ocumab)-induced renal injury. J Pharm Pract 2017;30:7–8.3. Van Poelgeest EP, Swart RM, Betjes MG, et al.Acute kidney injury during therapy with an anti-sense oligonucleotide directed against PCSK9. AmJ Kidney Dis 2013;62:796–800.4. Robinson JG, Famier M, Krempf M, et al. Effi-cacy and safety of alirocumab in reducing lipids

and cardiovascular events. N Engl J Med 2015;372:1489–99.

5. Makris K, Spanou L. Acute kidney injury: defi-nition, pathophysiology and clinical phenotypes.Clin Biochem Rev 2016;37:85–98.

KEY WORDS cardiovascular disease,hypercholesterolemia, secondary prevention

Page 66: Full Issue PDF · The JACC: Case Reports Editorial Board and ACC editorial office would like to express their sincere gratitude to the guest editors below who worked tirelessly …

J A C C : C A S E R E P O R T S V O L . 2 , N O . 7 , 2 0 2 0

ª 2 0 2 0 T H E A U T H O R S . P U B L I S H E D B Y E L S E V I E R O N B E H A L F O F T H E AM E R I C A N

C O L L E G E O F C A R D I O L O G Y F O U N DA T I O N . T H I S I S A N O P E N A C C E S S A R T I C L E U N D E R

T H E C C B Y - N C - N D L I C E N S E ( h t t p : / / c r e a t i v e c o mm o n s . o r g / l i c e n s e s / b y - n c - n d / 4 . 0 / ) .

IMAGING VIGNETTE

CLINICAL VIGNETTE

Wandering Atrial Pacemaker Wire

Migration of a Temporary Epicardial Pacing Wire Intothe Left Heart

Jennifer D. Hua, AB,a Shoheb S. Ali, MD,b Vamsi Reddy, BS,c Salil J. Patel, MDd

ABSTRACT

ISS

Fro

No

the

the

Th

ins

vis

Ma

Temporary epicardial pacing, routinely used after cardiac surgery, employs wires anchored to the epicardium allowing

removal via traction. In cases of resistance, the temporary wires are cut flush at the skin. We present a rare noninfectious

case of a migrated retained temporary pacing wire into the left heart. (Level of Difficulty: Beginner.)

(J Am Coll Cardiol Case Rep 2020;2:1046–8) © 2020 The Authors. Published by Elsevier on behalf of the

American College of Cardiology Foundation. This is an open access article under the CC BY-NC-ND license

(http://creativecommons.org/licenses/by-nc-nd/4.0/).

A n asymptomatic 65-year-old man, with a history of minimally invasive aortic valve replacement 2years prior, presented for an outpatient transesophageal echocardiogram (TEE). The TEE revealed alinear density beginning in the left ventricle (LV) adjacent to the anterior leaflet of the mitral valve

(Figure 1A) before crossing through the bioprosthetic aortic valve and finally terminating in the ascending aortaproximal to the brachiocephalic artery. Computed tomography angiography confirmed a thin linear metallicforeign body measuring up to 15 cm (Figure 1B). On subsequent fluoroscopy, the object appeared to be fixedin the LV, free-floating at the level of the aortic valve as well as within the ascending aorta (Figure 1C).

Employing the aforementioned noninvasive imaging modalities, we formulated a plan to retrieve theforeign body. The TEE first revealed the linear density, and computed tomography angiography ruled outaortic dissection and offered further insight into its position. Additionally, use of fluoroscopy allowed us toevaluate its movement and determined the distal end was unattached. Interventional radiology was consultedto attempt percutaneous snare removal of the object with cardiac surgery on standby. Successful retrieval ofthe foreign body revealed a migrated temporary epicardial pacing wire (Figure 1D). After tolerating the pro-cedure, the patient had stable mitral valve function and was subsequently discharged.

During the aortic valve replacement procedure 2 years prior, bipolar pacing wires were placed on theventricle while 2 atrial wires were attached to the atrial epicardium and tunneled through the skin. Temporaryepicardial pacing, routinely used after cardiac surgery, employs wires lightly anchored to the epicardiumwhich allows for removal via gentle traction through the skin. In cases of resistance, the temporary wires arecut flush at the skin (1). The migration of retained temporary epicardial pacing wires is very rare but has beensuspected to be due to direct perforation of the cardiac musculature or neighboring vascular structures. Most

N 2666-0849 https://doi.org/10.1016/j.jaccas.2020.04.035

m the aPhiladelphia College of Osteopathic Medicine Georgia, Suwanee, Georgia; bInternal Medicine Residency Program,

rthside Hospital Gwinnett, Lawrenceville, Georgia; cMedical College of Georgia, Augusta University, Augusta, Georgia; anddCardiovascular Group P.C., Lawrenceville, Georgia. The authors have reported that they have no relationships relevant to

contents of this paper to disclose.

e authors attest they are in compliance with human studies committees and animal welfare regulations of the authors’

titutions and Food and Drug Administration guidelines, including patient consent where appropriate. For more information,

it the JACC: Case Reports author instructions page.

nuscript received February 12, 2020; revised manuscript received April 19, 2020, accepted April 28, 2020.

Page 67: Full Issue PDF · The JACC: Case Reports Editorial Board and ACC editorial office would like to express their sincere gratitude to the guest editors below who worked tirelessly …

AB BR E V I A T I O N S

AND ACRONYM S

LV = left ventricle/ventricular

TEE = transesophageal

echocardiogram

J A C C : C A S E R E P O R T S , V O L . 2 , N O . 7 , 2 0 2 0 Hua et al.J U N E 1 7 , 2 0 2 0 : 1 0 4 6 – 8 Wandering Atrial Pacemaker Wire

1047

reported cases occurred within the context of an infection—a possible source oftissue weakening and disruption leading to direct perforation (2).

The mechanism of this wire’s migration from its original attachment at theepicardium of the right atrium into the LV is still anatomically unclear, as it wasfound entirely within the left heart and aorta under noninfectious conditions. It wasfortuitously affixed to a chord of the mitral valve apparatus, preventing further

travel into the aorta. Among previously reported cases, the few wires that had migrated into the left heart wereall found within the aorta or carotid arteries—none had been found within the LV. Only 1 other case (2) de-scribes left-sided migration under noninfectious conditions. Previous cases hypothesized that the wires mayhave perforated through the aortic wall (3). In our patient, the chance event of the wire fortuitously attachingto the mitral valve apparatus and its predominantly LV location rule out aortic perforation. The treatmentoptions are percutaneous endovascular retrieval or surgical repair, with endovascular intervention being thepreferred, less invasive option (3).

FIGURE 1 Imaging of the Migrated Temporary Epicardial Pacemaker Wire

(A) Transesophageal echocardiogram revealing a linear density (W) in the left ventricle (LV) attached to the mitral valve (MV) traveling

through the bioprosthetic aortic valve (AV) and into the ascending aorta (A). (B) Computed tomography angiogram showing the 15-cm

density (W) in the LV coursing through the bioprosthetic AV with redundancy in the aorta (A). (C) Fluoroscopy showing migrated temporary

pacing wire (W) running through the bioprosthetic AV as well as other retained epicardial pacing wires (arrows). (D) Retrieved temporary

epicardial pacing wire. LA ¼ left atrium; RV ¼ right ventricle.

Page 68: Full Issue PDF · The JACC: Case Reports Editorial Board and ACC editorial office would like to express their sincere gratitude to the guest editors below who worked tirelessly …

Hua et al. J A C C : C A S E R E P O R T S , V O L . 2 , N O . 7 , 2 0 2 0

Wandering Atrial Pacemaker Wire J U N E 1 7 , 2 0 2 0 : 1 0 4 6 – 8

1048

ADDRESS FOR CORRESPONDENCE: Ms. Jennifer D. Hua, Cardiovascular Group, 755 Walther Road, Lawren-ceville, Georgia 30046. E-mail: [email protected].

RE F E RENCE S

1. Reade MC. Temporary epicardial pacing after car-diacsurgery:apractical review.Anaesthesia2007;62:264–71. Erratum in: Anaesthesia 2007;62:644.

2. Mukaihara K, YotsumotoG,MatsumotoK, ImotoY.Migration of a retained temporary epicardial pacing

wire into an abdominal aortic aneurysm. Eur J Car-diothorac Surg 2015;48:169–70.

3. Wolf LG, Scaffa R, Maselli D, et al. Intraaorticmigration of an epicardial pacing wire: percuta-neous extraction. Ann Thorac Surg 2013;96:e7–8.

KEY WORDS cardiac pacemaker,echocardiography, imaging, temporarycardiac pacing, temporary epicardialpacemaker, x-ray fluoroscopy

Page 69: Full Issue PDF · The JACC: Case Reports Editorial Board and ACC editorial office would like to express their sincere gratitude to the guest editors below who worked tirelessly …

J A C C : C A S E R E P O R T S VO L . 2 , N O . 7 , 2 0 2 0

ª 2 0 2 0 T H E A U T H O R S . P U B L I S H E D B Y E L S E V I E R O N B E H A L F O F T H E A M E R I C A N

C O L L E G E O F C A R D I O L O G Y F OU N D A T I O N . T H I S I S A N O P E N A C C E S S A R T I C L E U N D E R

T H E C C B Y - N C - N D L I C E N S E ( h t t p : / / c r e a t i v e c o mm o n s . o r g / l i c e n s e s / b y - n c - n d / 4 . 0 / ) .

CASE REPORT

CLINICAL CASE

A Case of Ventricular TachycardiaCaused by a Rare CardiacMesenchymal Hamartoma

Zekun Feng, MD, PHARMD,a Daniel Philipson, MD,a Jamar P. Uzzell, MD,b Ashley Stein-Merlob, MD,a

Eric H. Yang, MD,a,c Holly R. Middlekauff, MD,a Ryan P. Lau, MD,b Gregory A. Fishbein, MD,b

Jason S. Bradfield, MD,a,c Olujimi A. Ajijola, MD, PHDa,c,d

ABSTRACT

ISS

Fro

Pa

De

Da

ho

Ne

Th

ins

vis

Ma

The presentation of a cardiac hamartoma, an exceedingly rare and histologically benign cardiac tumor, can be variable.

We describe a case of refractory ventricular tachycardia in a patient with a cardiac mass failing multiple pharmacologic

and procedural interventions, ultimately treated by cardiac transplantation and diagnosed with a mesenchymal cardiac

hamartoma. (Level of Difficulty: Intermediate.) (J Am Coll Cardiol Case Rep 2020;2:1049–55) © 2020 The Authors.

Published by Elsevier on behalf of the American College of Cardiology Foundation. This is an open access article under the

CC BY-NC-ND license (http://creativecommons.org/licenses/by-nc-nd/4.0/).

LEARNING OBJECTIVES

� To recognize the challenges in diagnosingcardiac hamartoma.

� To recognize the difficulties in controllingventricular tachycardia associated withcardiac hamartoma.

� To review the treatment options reported inthe literature for ventricular tachyarrhyth-mias associated with cardiac hamartoma.

� To review the histopathology of the raremesenchymal cardiac hamartoma.

HISTORY OF PRESENTATION

A 33-year-old woman with 3-year history of asym-metric cardiac hypertrophy and sustained ventriculartachycardia (VT) was treated with antiarrhythmicdrug (AAD) therapy and placement of an implantablecardioverter-defibrillator (ICD). Prior to the currentadmission, she had recurrent VT despite multipleAADs (metoprolol, amiodarone, sotalol, and dofeti-lide), requiring several ICD shocks, prompting anunsuccessful endocardial radiofrequency catheterablation and subcutaneous ICD array placement forelevated defibrillation thresholds. Six months prior toher current presentation, she underwent cardiacsympathetic denervation followed by renal artery

N 2666-0849

m the aDivision of Cardiology, Department of Medicine, UCLA Medical C

thology and Laboratory Medicine, UCLA Health, Los Angeles, California;

partment of Medicine, University of California-Los Angeles, Los Angeles, C

vid Geffen School of Medicine, University of California-Los Angeles, Los A

noraria from Biosense Webster. Dr. Ajijola has received research grant s

uCures. All other authors have reported that they have no relationships r

e authors attest they are in compliance with human studies committe

titutions and Food and Drug Administration guidelines, including patien

it the JACC: Case Reports author instructions page.

nuscript received February 18, 2020; revised manuscript received April 1

denervation, given that her VT was precipitated bystress. The current admission was precipitated bysymptomatic recurrent VT and a total of 22 ineffective

https://doi.org/10.1016/j.jaccas.2020.04.038

enter, Los Angeles, California; bUCLA Department ofcCardio-Oncology Program, Division of Cardiology,

alifornia; and the dUCLA Cardiac Arrhythmia Center,

ngeles, California. Dr. Bradfield has received speaker

upport from Biosense Webster; and owns equity in

elevant to the contents of this paper to disclose.

es and animal welfare regulations of the authors’

t consent where appropriate. For more information,

9, 2020, accepted April 28, 2020.

Page 70: Full Issue PDF · The JACC: Case Reports Editorial Board and ACC editorial office would like to express their sincere gratitude to the guest editors below who worked tirelessly …

ABBR EV I A T I ON S

AND ACRONYMS

AAD = antiarrhythmic drug

CMR = cardiac magnetic

resonance

HCM = hypertrophic

cardiomyopathy

ICD = implantable

cardioverter-defibrillator

LV = left ventricular

VT = ventricular tachycardia

Feng et al. J A C C : C A S E R E P O R T S , V O L . 2 , N O . 7 , 2 0 2 0

VT Caused by Rare Cardiac Hamartoma J U N E 1 7 , 2 0 2 0 : 1 0 4 9 – 5 5

1050

ICD shocks. A single external 200 J shocksuccessfully converted her to sinus rhythm,and she was transferred to our center. Onarrival, her heart rate and blood pressurewere within normal limits. Her physical ex-amination was unremarkable, and labsshowed troponin of 11.1 ng/ml with normalelectrolyte levels. A 12-lead electrocardio-gram showed normal sinus rhythm with T-wave inversions in the inferior and septalleads, extreme left axis deviation, left ven-tricular (LV) hypertrophy, and incomplete

right bundle branch block (Figure 1A). Prior trans-thoracic echocardiography revealed asymmetricinferolateral LV wall hypertrophy with myocardialenhancement measuring 19 � 45 mm and an ejectionfraction of 55% to 60% (Figures 2A and 2B).

DIFFERENTIAL DIAGNOSIS

Atypical hypertrophic cardiomyopathy (HCM) wasconsidered, given the atypical isolated segmentinvolvement of inferolateral LV wall seen ontransthoracic echocardiography. Benign primarycardiac tumors were considered more likely thanprimary malignant cardiac tumors or metastaticcardiac tumors, given the patient’s age and herotherwise unremarkable medical history. Althoughmany benign tumors present themselves early onin infancy or childhood, benign tumors includingfibroma, hemangioma, lipoma, and hamartoma mayalso present in adulthood (1). Cardiac sarcoidosiswas also considered. Diagnoses including channe-lopathies such as congenital long QT syndromeand catecholaminergic polymorphic VT wereexcluded based on electrocardiogram findings andlack of response to denervation procedures,respectively.

INVESTIGATIONS

The UCLA Institutional Review Board approved thisstudy. The patient underwent extensive diagnosticwork-up in the 3 years prior to presentation. Cardiacmagnetic resonance (CMR) showed similar findingswith delayed hyperenhancement and a hypokineticcenter suggesting asymmetric HCM versus infiltra-tive process (Figures 3A and 3B). Coronary angiog-raphy revealed normal coronary arteries. Afluorodeoxyglucose positron emission tomographyscan did not demonstrate evidence of activesarcoidosis and high-grade neoplasm. An incisionalmyocardial biopsy was performed, with pathologyshowing fibrosis and nonspecific myocardial hyper-trophy. At electrophysiology study, multiple VT

morphologies were induced, and ablation was un-successful (Figures 4A and 4B).

MANAGEMENT

Upon arrival, oral metoprolol and intravenous lido-caine suppressed her VT, but neurotoxicity necessi-tated lidocaine discontinuation. Mexiletine (150 mg 3times a day), metoprolol tartrate (200 mg twice aday), and dofetilide (250 mg twice a day) were thenstarted. Given the difficulty in controlling her VTdespite multiple antiarrhythmic therapies, failure oftachyarrhythmia suppression despite sympatheticand renal artery denervation, and multiple VT mor-phologies with poor ablation targets on electrophys-iology study, the team felt that the myocardialenhancement seen on imaging studies was mostconsistent with a cardiac tumor, rather than withasymmetric HCM. Cardiothoracic surgery was con-sulted for debulking of the mass but was deemed nota candidate because of its large size. Stereotactic bodyradiation therapy to the LV was considered for drug-refractory VT but was declined by the patient. Withlimited noninvasive options remaining but persistentVT, she underwent orthotopic heart transplantation.The final histopathology revealed mature cardiacmyocytes with fibrosis, adipose tissue, nerves, andsmooth muscle, consistent with a cardiac mesen-chymal hamartoma (Figure 5D).

DISCUSSION

Cardiac hamartomas, including rhabdomyomas andfibromas (2), are rare and typically present in thepediatric population. A mesenchymal subtype ofcardiac hamartoma, as seen in our patient, isexceedingly rare, with only 1 previously reported casein the literature (2). Cardiac hamartomas presentvariably, ranging from asymptomatic to sudden car-diac death (2). The cardiac hamartoma literatureconsists of case reports, with limited treatment op-tions for VT. Our case demonstrates the challenge indiagnosing cardiac hamartomas and the difficulty inmanaging associated life-threatening refractory VT.

Owing to rarity and nonspecific presentation, car-diac hamartomas are difficult to diagnose non-invasively. Differentiating cardiac hamartomas fromother masses such as HCM, thrombus, or malignantcardiac tumors can be challenging, as imaging find-ings are often nonspecific. On echocardiography,hamartomas are reported as hyperechoic intra-cavitary masses with predilection for the LV (3,4). OnCMR, the appearance of a hamartoma ranges frommildly hypointense to hyperintense signal on T2-weighted images, and enhancement on early and

Page 71: Full Issue PDF · The JACC: Case Reports Editorial Board and ACC editorial office would like to express their sincere gratitude to the guest editors below who worked tirelessly …

FIGURE 1 Electrocardiogram

(A) Electrocardiogram at baseline showing normal sinus rhythm with extreme left axis deviation, left ventricular hypertrophy, and

incomplete right bundle branch block. (B) Electrocardiogram during VT episode showing monomorphic wide complex tachycardia at a rate of

150 beats/min.

J A C C : C A S E R E P O R T S , V O L . 2 , N O . 7 , 2 0 2 0 Feng et al.J U N E 1 7 , 2 0 2 0 : 1 0 4 9 – 5 5 VT Caused by Rare Cardiac Hamartoma

1051

delayed phase post-contrast images (4). CMR mayexclude thrombus or malignant lesions by first passand delayed contrast enhancement, and regionalvariation in vascularity (3). Fluorodeoxyglucosepositron emission tomography may also help differ-entiate benign from malignant lesions (4). Incisionalbiopsy may be inconclusive (as in this case), as thehamartoma tissue can be nonspecific and similar tothe surrounding myocardium.

Given its rarity, current guidelines do not addressVT management related to cardiac hamartoma.Existing published case reports of VT associated withhamartoma are mostly in the pediatric population. Ina case series of 21 infants (mean 14.9 months of age)with incessant VT owing to cardiac hamartoma, AADs(lidocaine, procainamide, amiodarone, propranolol,digoxin, verapamil, mexiletine, phenytoin, andpropafenone) failed in all patients (5). Surgical

Page 72: Full Issue PDF · The JACC: Case Reports Editorial Board and ACC editorial office would like to express their sincere gratitude to the guest editors below who worked tirelessly …

FIGURE 2 Transthoracic Echocardiogram

(A) Parasternal long-axis and (B) short-axis views of transthoracic echocardiogram demonstrating an asymmetrical and enlarged thickened

inferolateral left ventricular wall (red arrow).

Feng et al. J A C C : C A S E R E P O R T S , V O L . 2 , N O . 7 , 2 0 2 0

VT Caused by Rare Cardiac Hamartoma J U N E 1 7 , 2 0 2 0 : 1 0 4 9 – 5 5

1052

intervention with epicardial mapping–guided resec-tion and cryoablation resulted in complete cure ofVT in all 21 pediatric patients (5). Case reports ofadults with VT owing to hamartoma are limited, butall cases required surgery. For example, a 33-year-old patient underwent successful surgical resectionwith resolution of VT (6). A 19-year-old patientwhose large hamartoma was a contraindication tosurgical resection underwent successful cardiactransplantation (7). A 55-year-old patient with partialresection of cardiac hamartoma presented with a laterecurrence of VT, which was successfully treatedwith radiofrequency catheter ablation (8). In ourcase, surgical debulking may have eliminated VT;

FIGURE 3 Cardiac Magnetic Resonance

Cardiac magnetic resonance in (A) short-axis view and (B) 3-chamber vi

enhancement (red arrows).

however, this was contraindicated because of thesize of the mass.

ICDs remain a cornerstone of VT therapy to reducesudden cardiac death but do not address the under-lying cause. In this case, the asymmetric hypertrophyrelative to the ICD shock vector may have causedineffective shocks, as more than 20 ICD shocks failedto cardiovert her VT, whereas 1 external shock wassuccessful. Novel interventions such as stellate gan-glion block and renal artery denervation have shownpromise as adjunctive therapy for recurrent VT,especially those thought to be triggered by sympa-thetic activation accompanying physical or emotionalstress, as in our patient (9,10). Stereotactic body

ew showing asymmetric inferolateral wall with late gadolinium

Page 73: Full Issue PDF · The JACC: Case Reports Editorial Board and ACC editorial office would like to express their sincere gratitude to the guest editors below who worked tirelessly …

FIGURE 4 Electrophysiology Study and VT Ablation

(A) Multiple ventricular tachycardia (VT) morphologies observed during electrophysiology study and programmed stimulation; VT ablation

was unsuccessful due to the thickened extensive transmural scar without good endocardial targets and ineffective ablation lesions. (B)

Activation map for VT 1 (the predominant VT morphology) showing the earliest activation (exit) site on the basal anterolateral LV (red arrows

point to exit site). Right anterior oblique view on the left panel, left anterior oblique view on center panel, and posteroanterior view on the

right panel.

J A C C : C A S E R E P O R T S , V O L . 2 , N O . 7 , 2 0 2 0 Feng et al.J U N E 1 7 , 2 0 2 0 : 1 0 4 9 – 5 5 VT Caused by Rare Cardiac Hamartoma

1053

radiotherapy is a noninvasive and reportedly effec-tive strategy to reduce VT burden for patients withstructural heart disease failing catheter ablation (11).Definitive therapy with cardiac transplantation incases not amenable for resection is considered atherapy of last resort.

Cardiac hamartomas include hamartomas ofmature cardiac myocytes, vascular hamartomas, andleast commonly, cardiac mesenchymal hamartomas.Hamartomas of mature cardiac myocytes arecomposed of a well-circumscribed proliferation ofcardiac myocytes with myofiber disarray. In contrast,HCM is characterized by a poorly demarcated area of

cardiac hypertrophy most frequently located in theanterior ventricular septum, with microscopic find-ings of diffuse myocyte hypertrophy, focal myofiberdisarray, and intramural coronary thickening (12).Vascular hamartomas are characterized by markedlyabnormal vascular proliferation, whereas cardiacmesenchymal hamartomas are composed of awell-circumscribed but disorganized proliferation ofcardiac myocytes, smooth muscle, blood vessels, fi-broblasts, mature fat, and nerves (2). The presence ofall 6 components distinguishes mesenchymalhamartoma from other benign cardiac tumors(Figure 5D).

Page 74: Full Issue PDF · The JACC: Case Reports Editorial Board and ACC editorial office would like to express their sincere gratitude to the guest editors below who worked tirelessly …

FIGURE 5 Gross and Histopathology

(A, B) Gross specimen images showing thickened left ventricular wall and mass (black arrow). (C) Histopathology slide showing the ill-defined

tumor (right side) with an infiltrative border, along with the normal myocardium (left side) (hematoxylin and eosin, 40� magnification). (D)

Tumor contains cardiac myocytes (yellow arrow), fibrosis (black arrow), adipose tissue (blue arrow), blood vessels (red arrow), and nerve

tissue (green arrow) (trichrome a elastin stain, 100� magnification), consistent with the mesenchymal subtype of cardiac hamartoma. The

inset shows nerve tissue (hematoxylin and eosin, 100x magnification).

Feng et al. J A C C : C A S E R E P O R T S , V O L . 2 , N O . 7 , 2 0 2 0

VT Caused by Rare Cardiac Hamartoma J U N E 1 7 , 2 0 2 0 : 1 0 4 9 – 5 5

1054

FOLLOW-UP

The patient underwent uncomplicated orthotopicheart transplant and later removal of her ICD gener-ator and leads. She continues close follow-up with hercare team.

CONCLUSIONS

Benign cardiac tumors are rare, and cardiac hamar-tomas are among the rarest. This case report high-lights the diagnostic challenge posed by the raresubtype of mesenchymal cardiac hamartoma and thetherapeutic dilemma in managing associated tachy-arrhythmias. Given the similarities to HCM on clinical

presentation and diagnostic imaging, it is prudent tomaintain an appropriate index of suspicion and toseek for an alternative diagnosis in future cases ofyoung patients with treatment refractory VT in thesetting of an atypical cardiac mass. Cardiac trans-plantation should remain in the diagnostic and ther-apeutic armamentarium—but should be used as a lastresort.

ADDRESS FOR CORRESPONDENCE: Dr. Olujimi A.Ajijola, UCLA Cardiac Arrhythmia Center, David Gef-fen School of Medicine at UCLA, 100 Medical Plaza,Suite 660, Los Angeles, California 90095. E-mail:[email protected]. Twitter: @zekunfengmd.

Page 75: Full Issue PDF · The JACC: Case Reports Editorial Board and ACC editorial office would like to express their sincere gratitude to the guest editors below who worked tirelessly …

J A C C : C A S E R E P O R T S , V O L . 2 , N O . 7 , 2 0 2 0 Feng et al.J U N E 1 7 , 2 0 2 0 : 1 0 4 9 – 5 5 VT Caused by Rare Cardiac Hamartoma

1055

RE F E RENCE S

1. Jayaprakash S. Clinical presentations, diag-nosis, and management of arrhythmias associ-ated with cardiac tumors. J Arrhythm 2018;34:384–93.

2. Bradshaw SH, Hendry P, Boodhwani M,Dennie C, Veinot JP. Left ventricular mesenchymalhamartoma, a new hamartoma of the heart. Car-diovasc Pathol 2011;20:307–14.

3. Hoey ET, Mankad K, Puppala S, Gopalan D,Sivananthan MU. MRI and CT appearances of car-diac tumours in adults. Clin Radiol 2009;64:1214–30.

4. Abuzaid AS, Gakhal M, Montgomery E,LaPoint R, Horn R, Banbury MK. Cardiac hamar-toma: a diagnostic challenge. CASE (Phila) 2017;1:59–61.

5. Garson A Jr., Smith RT Jr., Moak JP, et al.Incessant ventricular tachycardia in infants:

myocardial hamartomas and surgical cure. J AmColl Cardiol 1987;10:619–26.

6. Dinh MH, Galvin JM, Aretz TH, Torchiana DF. Leftventricular hamartoma associated with ventriculartachycardia. Ann Thorac Surg 2001;71:1673–5.

7. Hsu PS, Chen JL, Hong GJ, Tsai YT, Tsai CS.Heart transplantation for ventricular arrhythmiacaused by a rare hamartoma. J Heart Lung Trans-plant 2009;28:1114–5.

8. Xu J, Chen Y, Ying X, He B. Radiofrequencyablation of ventricular tachycardia originatingfrom a lipomatous hamartoma localized in theright ventricle cavity. HeartRhythm Case Rep2017;3:369–72.

9. Meng L, Tseng CH, Shivkumar K, Ajijola O. Ef-ficacy of stellate ganglion blockade in managingelectrical storm: a systematic review. J Am CollCardiol EP 2017;3:942–9.

10. Remo BF, Preminger M, Bradfield J, et al.Safety and efficacy of renal denervation as a noveltreatment of ventricular tachycardia storm in pa-tients with cardiomyopathy. Heart Rhythm 2014;11:541–6.

11. Cuculich PS, Schill MR, Kashani R, et al.Noninvasive cardiac radiation for ablation of ven-tricular tachycardia. N Engl J Med 2017;377:2325–36.

12. Burke AP, Ribe JK, Bajaj AK, Edwards WD,Farb A, Virmani R. Hamartoma of mature cardiacmyocytes. Hum Pathol 1998;29:904–9.

KEY WORDS cardiac hamartoma,electrophysiology, refractory ventriculartachycardia

Page 76: Full Issue PDF · The JACC: Case Reports Editorial Board and ACC editorial office would like to express their sincere gratitude to the guest editors below who worked tirelessly …

J A C C : C A S E R E P O R T S V O L . 2 , N O . 7 , 2 0 2 0

ª 2 0 2 0 T H E A U T H O R S . P U B L I S H E D B Y E L S E V I E R O N B E H A L F O F T H E AM E R I C A N

C O L L E G E O F C A R D I O L O G Y F O U N DA T I O N . T H I S I S A N O P E N A C C E S S A R T I C L E U N D E R

T H E C C B Y - N C - N D L I C E N S E ( h t t p : / / c r e a t i v e c o mm o n s . o r g / l i c e n s e s / b y - n c - n d / 4 . 0 / ) .

CASE REPORT

CLINICAL CASE

Cardiac Sarcoidosis Causing VentricularTachycardia After Myocardial InfarctionA Shocking Diagnosis

Jayshiv T. Badlani, MD, MS, Indu G. Poornima, MD, Amit Thosani, MD, Robert W.W. Biederman, MD

ABSTRACT

L

ISS

Fro

ha

Th

ins

vis

Ma

Scar-mediated ventricular tachycardia (VT) commonly results from ischemic heart disease. We present a case of recurrent

VT, which was initially attributed to ischemic disease; however, the scar location pointed to an alternate pathology.

This case demonstrates the utility of multimodality imaging in diagnosing sarcoidosis as a cause of VT.

(Level of Difficulty: Intermediate.) (J Am Coll Cardiol Case Rep 2020;2:1056–61) © 2020 The Authors. Published by

Elsevier on behalf of the American College of Cardiology Foundation. This is an open access article under the CC BY-NC-ND

license (http://creativecommons.org/licenses/by-nc-nd/4.0/).

HISTORY OF PRESENTATION

A 57-year-old man with a history of presumed infarct-related cardiomyopathy, ventricular tachycardia(VT), and implantable cardioverter-defibrillator (ICD)presented with 2 ICD shocks. He was using the bath-room while at work and was shocked by his ICD. Hethen walked out of the bathroom and once again wasshocked, prompting a call to emergency services. Hepresented to the emergency room with normal vitalsigns and without distress; interrogation of his ICD

EARNING OBJECTIVES

To appreciate that, even when coronaryartery disease is present, not all VT is due toischemia or infarction.To understand the benefits of CMR in evalu-ating scar-related VT.To understand the benefits of FDG PET indiagnosing and monitoring cardiacsarcoidosis.

N 2666-0849

m the Department of Cardiology, Allegheny General Hospital, Pittsburgh

ve no relationships relevant to the contents of this paper to disclose.

e authors attest they are in compliance with human studies committe

titutions and Food and Drug Administration guidelines, including patien

it the JACC: Case Reports author instructions page.

nuscript received February 18, 2020; revised manuscript received April 1

revealed appropriately treated episodes of poly-morphic VT (Figure 1).

MEDICAL HISTORY

One year prior to the current presentation, the patientpresented to his local hospital with an episode ofsyncope while driving. In the hospital, the patienthad recurrent ventricular arrythmias on cardiacmonitoring. Work-up at that time had revealed asignificant stenosis in the left anterior descendingartery (LAD), and it was presumed that LAD territoryischemia was responsible for the VT and syncope. Thepatient received 2 drug-eluting stents and was dis-charged on amiodarone and with an external defi-brillator—ejection fraction (EF) was noted to be 35%with an inferior wall motion abnormality onechocardiography.

A few months later, the patient’s echocardiogramrevealed normal chamber size, grade I diastolicdysfunction, and EF of 30% despite revascularizationand medical therapy. Electrocardiography revealed

https://doi.org/10.1016/j.jaccas.2020.04.037

, Pennsylvania. The authors have reported that they

es and animal welfare regulations of the authors’

t consent where appropriate. For more information,

7, 2020, accepted April 20, 2020.

Page 77: Full Issue PDF · The JACC: Case Reports Editorial Board and ACC editorial office would like to express their sincere gratitude to the guest editors below who worked tirelessly …

AB BR E V I A T I O N S

AND ACRONYM S

CMR = cardiac magnetic

resonance

EF = ejection fraction

FDG = fluorodeoxyglucose

ICD = implantable

cardioverter-defibrillator

LAD = left anterior descending

artery

PET = positron emission

tomography

PVC = premature ventricular

contraction

ventricular tachycardia

J A C C : C A S E R E P O R T S , V O L . 2 , N O . 7 , 2 0 2 0 Badlani et al.J U N E 1 7 , 2 0 2 0 : 1 0 5 6 – 6 1 Cardiac Sarcoidosis Causing VT After MI

1057

T-wave inversions in the inferior leads. An ICD wasimplanted, and amiodarone was discontinued toprevent long-term adverse effects.

Three months after implantation, he experiencedan ICD shock but did not seek medical attention.Two months later, he experienced a second shockand presented to his local hospital for evaluation.Device interrogation showed multiple episodes ofnonsustained VT and polymorphic VT, with poly-morphic VT episodes being treated by his device. Itwas presumed that the VT was scar mediated froma prior infarct, with premature ventricular contrac-tions (PVCs) being the trigger based on deviceinterrogation. Electrophysiology study and VTablation was performed, with PVCs in the mid andapical inferoseptum mapped as triggers for poly-morphic VT. The mid inferoseptal PVCs were abla-ted; however, the apical PVCs were not completelyablated, as they were associated with hemody-namically unstable VF; the facility did not havecapability to complete the ablation with hemody-namic support.

Two months after ablation, he had recurrent VTand ventricular fibrillation and once again presentedto his local hospital. Cardiac catheterization wasperformed at that time, which revealed patent LADstents and no significant coronary artery disease. Hewas started on antiarrhythmic therapy with mex-iletine 200 mg 3 times a day but 2 months later pre-sented to our hospital with recurrent ICD shocks forVT and ventricular fibrillation. Notably, the patienthas a family history of sudden cardiac death in hismother when she was in her 60s, as well as in 2 sib-lings who were in their 50s.

FIGURE 1 Device Interrogation

Intracardiac electrocardiogram’s from the patient’s implantable cardiove

ventricular arrhythmias. In this representative example, a premature ven

break the rhythm with antitachycardia pacing. The rhythm eventually se

DIFFERENTIAL DIAGNOSIS

The differential diagnosis for recurrent VTincludes ischemic heart disease, end-stageheart failure, and congenital heart diseaseincluding arrhythmogenic right ventriculardysplasia, hypertrophic cardiomyopathy, andinfiltrative heart disease.

INVESTIGATION

Electrocardiography on most recent presen-tation showed sinus bradycardia with inferiorT-wave inversions (Figure 2). Given therecurrence of VT despite revascularization,

VT ablation, and antiarrhythmic therapy, cardiacmagnetic resonance (CMR) with an ICD protocol wasperformed to investigate for infiltrative disease andscar burden (1).

The CMR showed normal left ventricle size withreduced function and a left ventricular EF of 32%,with mid to distal inferior, inferoseptal, and infero-lateral thinning with akinesis. Late gadoliniumenhancement was performed and was markedlyabnormal, with full-thickness transmural scarextending throughout the inferior, inferoseptal, andinferolateral walls (Figure 3).

Given that the scar location did not correspondto the patient’s treated coronary artery disease,infiltrative disease was suspected, with sarcoidosisbeing high on the differential. A cardiac fluo-rodeoxyglucose (FDG) positron emission tomogra-phy (PET) study was ordered to evaluate forinflammation, and showed increased FDG uptake in

VT =

rter-defibrillator showed multiple examples of premature ventricular contraction–triggered

tricular contraction triggers polymorphic ventricular tachycardia, and there is an attempt to

lf-terminated.

Page 78: Full Issue PDF · The JACC: Case Reports Editorial Board and ACC editorial office would like to express their sincere gratitude to the guest editors below who worked tirelessly …

FIGURE 2 Electrocardiography

The patient’s electrocardiography showed sinus bradycardia with inferior T-wave inversions.

Badlani et al. J A C C : C A S E R E P O R T S , V O L . 2 , N O . 7 , 2 0 2 0

Cardiac Sarcoidosis Causing VT After MI J U N E 1 7 , 2 0 2 0 : 1 0 5 6 – 6 1

1058

the basal inferior, inferoseptal, and inferolateralwalls. There was also hilar lymph node uptakealong with mid to basal inferior and inferoseptalresting perfusion defects on the rubidium perfusionimages, indicative of scar and inflammation—find-ings consistent with sarcoidosis (Figures 4 to 6).Maximum standardized uptake values were 21.8 inthe inferoseptum and 5.1 in the hilar nodes. Thepatient underwent bronchoscopy with lymph nodebiopsy, which confirmed the diagnosis ofsarcoidosis.

FIGURE 3 Cardiac Magnetic Resonance

The patient’s cardiac magnetic resonance showed thinning of the inferio

indicates scar. Arrows indicate areas of late gadolinium enhancement.

MANAGEMENT

The patient continued on mexiletine in the short termfor VT suppression; meanwhile, the underlying causeof his VT, sarcoidosis, was treated with oralprednisone.

DISCUSSION

Sarcoidosis is a granulomatous disease that can affectany organ. Among patients with sarcoidosis, rates of

r and inferolateral wall. Late gadolinium enhancement in this area

Page 79: Full Issue PDF · The JACC: Case Reports Editorial Board and ACC editorial office would like to express their sincere gratitude to the guest editors below who worked tirelessly …

FIGURE 4 Cardiac FDG PET Splash Images

The fluorodeoxyglucose (FDG) positron emission tomography showed a resting perfusion defect in the inferior, inferoseptal, and inferolateral walls, and increased

amount of FDG uptake in the matched area, consistent with sarcoidosis-mediated scar and inflammation.

J A C C : C A S E R E P O R T S , V O L . 2 , N O . 7 , 2 0 2 0 Badlani et al.J U N E 1 7 , 2 0 2 0 : 1 0 5 6 – 6 1 Cardiac Sarcoidosis Causing VT After MI

1059

cardiac involvement have been reported to be as lowas 5% to as high as 27% in autopsy studies (2,3).

Making the diagnosis of cardiac sarcoidosis ischallenging, but perhaps even more difficult isinitially suspecting cardiac sarcoidosis and knowingwhen to screen for it. The 2014 Heart Rhythm ExpertConsensus Statement on the Diagnosis and Manage-ment of Arrhythmias Associated With CardiacSarcoidosis provides some guidance (4). Based onthat statement, individuals with diagnosed extrac-ardiac sarcoidosis should be screened with at least a12-lead electrocardiogram, a history with emphasis onsyncope and palpitations, and an echocardiogram.More advanced imaging modalities are noted to beappropriate only when those initial tests areabnormal.

The consensus is less clear when there is no priordiagnosis of extracardiac sarcoidosis. This is relevantnot only because there are cases of isolated cardiac

sarcoidosis, but also because even when extracardiacsarcoidosis is present, the initial presentation, asdemonstrated by our case, may still be cardiac (5).Small studies and case series have describedsarcoidosis patients who had an initial presentationof VT (6,7). This is still thought to be rare, leading todiscord among the experts; a majority of the writinggroup for the 2014 Heart Rhythm Society ExpertStatement recommended screening for sarcoidosis inpatients with unexplained monomorphic VT, but itdid not meet the threshold to be included as a formalrecommendation.

Even if that recommendation had been made astandard of care, our patient would have been missedbecause it was assumed that ischemic heart diseasewas the explanation for VT. Nonetheless, severalclues were present that led to the further work-up ofour patient’s VT. Notably, review of echocardiogra-phy from his initial presentation revealed inferior

Page 80: Full Issue PDF · The JACC: Case Reports Editorial Board and ACC editorial office would like to express their sincere gratitude to the guest editors below who worked tirelessly …

FIGURE 5 Whole-Body Transverse Image of Cardiac

Fluorodeoxyglucose Positron Emission Tomography

Positron emission tomography revealed increased fluorodeox-

yglucose uptake in the inferior, inferoseptal, and inferolateral

walls. This is consistent with cardiac sarcoidosis.

FIGURE 6 Fluorodeoxyglucose Positron Emission

Tomography

This also revealed increased fluorodeoxyglucose in the hilar

lymph nodes, consistent with systemic sarcoidosis.

Badlani et al. J A C C : C A S E R E P O R T S , V O L . 2 , N O . 7 , 2 0 2 0

Cardiac Sarcoidosis Causing VT After MI J U N E 1 7 , 2 0 2 0 : 1 0 5 6 – 6 1

1060

wall hypokinesis. This should have raised a questionabout his initial syncopal event, as his coronary dis-ease was only significant in the LAD territory, sug-gesting an alternate etiology for the inferior wallhypokinesis. The question could have been raisedagain during his VT ablation when he was found tohave isolated scar in the inferior wall, which wasablated.

The lack of correlation between the location of scarand coronary disease, combined with his incessantpolymorphic VT despite a lack of active ischemia,appropriate revascularization, and antiarrhythmictherapy led to the further work-up of his VT. CMR is auseful modality not only to show the scar location,but also to provide the scar burden, tissue charac-teristics, and a reproducible EF.

Although endomyocardial biopsy is the gold stan-dard for diagnosing cardiac sarcoidosis, and couldhave been considered after our CMR, the patchy na-ture of sarcoidosis gives the test a low sensitivity. Inthe right clinical scenario, FDG-PET imaging has highdiagnostic yield in cardiac sarcoidosis, and can also beused for prognostication. A metanalysis of 7 studieswith a total of 164 patients calculated a pooledsensitivity of 89% and a pooled specificity of 78% (8).When combined with resting perfusion imaging, car-diac PET has the ability to colocalize scar and in-flammatory burden in the myocardium. An importantconsideration before performing cardiac PET forsarcoidosis is to exclude significant coronary arterydisease, prior myocardial infarction, resting ischemia,

or hibernating myocardium. The presence of bothscar and inflammation is considered an adverseprognostic sign associated with a higher risk of deathor VT. FDG-PET can also identify extracardiac sarcoidand in fact, the Heart Rhythm Society criteria preferextracardiac tissue biopsy over endomyocardial bi-opsy because of the safety and higher yield of theformer to confirm the diagnosis. In addition, it ispossible to quantify inflammatory disease activityusing standardized uptake values, and these valuescan be used to monitor response to treatment (9,10).Our patient’s FDG-PET showed not only cardiacinflammation and scar, but also typical hilar lymph-adenopathy, and confirmatory diagnosis was thusmade by lymph node biopsy.

FOLLOW-UP

The patient continued on oral steroids with a gradualtaper toward a maintenance dose. At 3-month follow-up, device interrogation revealed no episodes ofsustained ventricular arrhythmias. A repeat PET scanshowed less inflammation than the initial diagnosticscan, with a maximum cardiac SUV of 8.4 (prior 21.8)and maximum hilar SUV of 2.9 (decreased from 5.1).He will continue on oral steroids and have his doseadjusted based on symptoms and serial PET scanning.

CONCLUSIONS

Sarcoidosis can be a hidden cause of VT, even in pa-tients with coronary artery disease. Presence of scar

Page 81: Full Issue PDF · The JACC: Case Reports Editorial Board and ACC editorial office would like to express their sincere gratitude to the guest editors below who worked tirelessly …

J A C C : C A S E R E P O R T S , V O L . 2 , N O . 7 , 2 0 2 0 Badlani et al.J U N E 1 7 , 2 0 2 0 : 1 0 5 6 – 6 1 Cardiac Sarcoidosis Causing VT After MI

1061

on CMR or resting perfusion imaging may not alwaysrepresent myocardial infarction. An infiltrative pro-cess can be the culprit and the cause of recurrentventricular arrhythmias, and should be suspectedwhenever there is a discrepancy between scar loca-tion and angiographic coronary artery disease.

ADDRESS FOR CORRESPONDENCE: Dr. Jayshiv T.Badlani, Allegheny General Hospital, Department ofCardiology, 320 East North Avenue, 4th Floor SnyderPavilion, Pittsburgh, Pennsylvania 15212. E-mail:[email protected].

RE F E RENCE S

1. Russo RJ, Costa HS, Silva PD, et al. Assessingthe risks associated with MRI in patients with apacemaker or defibrillator. N Engl J Med 2017;376:755–64.

2. Okada DR, Smith J, Derakhshan A, et al. Ven-tricular arrythmias in cardiac sarcoidosis. Circula-tion 2018;138:1253–64.

3. Statement on sarcoidosis. Joint Statement ofthe American Thoracic Society (ATS), the EuropeanRespiratory Society (ERS) and the World Associa-tion of Sarcoidosis and Other Granulomatous Dis-orders (WASOG) adopted by the ATS Board ofDirectors and by the ERS Executive Committee,February 1999. Am J Respir Crit Care Med 1999;160:736–55.

4. Birnie DH, Sauer WH, Bogun F, et al. HRSexpert consensus statement on the diagnosisand management of arrhythmias associated with

cardiac sarcoidosis. Heart Rhythm 2014;11:1305–23.

5. Okada DR, Bravo PE, Vita T, et al. Isolatedcardiac sarcoidosis: a focused review of anunder-recognized entity. J Nucl Cardiol 2018;25:1136–46.

6. Nery PB, Mc Ardle B, Redpath C, et al. Preva-lence of cardiac sarcoidosis in patients presentingwith monomorphic ventricular tachycardia. PacingClin Electrophysiol 2014;37:364–74.

7. Koplan B, Soejima K, Baughman K, Epstein L,Stevenson W. Refractory ventricular tachycardiasecondary to cardiac sarcoid: electrophysiologiccharacteristics, mapping, and ablation. HeartRhythm 2006;3:924–9.

8. Youssef G, Leung E, Mylonas I, et al. The use of18F-FDG PET in the diagnosis of cardiac sarcoid-osis: a systematic review and metaanalysis

including the Ontario experience. J Nucl Med2012;53:241–8.

9. Blankstein R, Osborne M, Naya M, et al. Cardiacpositron emission tomography enhances prog-nostic assessments of patients with suspectedcardiac sarcoidosis. J Am Coll Cardiol 2014;63:329–36.

10. Chareonthaitawee P, Beanlands RS, Chen W,et al. Joint SNMMI-ASNC expert consensus docu-ment on the role of 18F-FDG-PET/CT in cardiacsarcoid detection and therapy monitoring. J NuclCardiol 2017;58:1341–53.

KEY WORDS autoimmune, cardiacmagnetic resonance, cardiac positronemission tomography, multimodalityimaging, nuclear medicine, ventriculartachycardia

Page 82: Full Issue PDF · The JACC: Case Reports Editorial Board and ACC editorial office would like to express their sincere gratitude to the guest editors below who worked tirelessly …

J A C C : C A S E R E P O R T S V O L . 2 , N O . 7 , 2 0 2 0

ª 2 0 2 0 T H E A U T H O R S . P U B L I S H E D B Y E L S E V I E R O N B E H A L F O F T H E AM E R I C A N

C O L L E G E O F C A R D I O L O G Y F O U N DA T I O N . T H I S I S A N O P E N A C C E S S A R T I C L E U N D E R

T H E C C B Y - N C - N D L I C E N S E ( h t t p : / / c r e a t i v e c o mm o n s . o r g / l i c e n s e s / b y - n c - n d / 4 . 0 / ) .

CASE REPORT

CLINICAL CASE

Eosinophilic Granulomatosis WithPolyangiitis Presenting as anAcute Coronary Syndrome

Jeffrey Kolominsky, MD,a Zackary Tushak, DO,b Jaideep Patel, MD,b Aaron Schatz, MD,b Ajay Pillai, MD,b

Jonathan Potfay, MDc

ABSTRACT

L

��

ISS

Fro

olo

olo

rel

Th

ins

vis

Ma

Eosinophilic granulomatosis with polyangiitis, formerly Churg-Strauss Syndrome, is an uncommon disorder that carries

a high mortality when coronary artery disease develops. Early recognition and treatment is crucial. We highlight

an unusual presentation of acute coronary syndrome not associated with atherosclerotic coronary disease. (Level of

Difficulty: Intermediate.) (J Am Coll Cardiol Case Rep 2020;2:1062–5) © 2020 The Authors. Published by Elsevier on

behalf of the American College of Cardiology Foundation. This is an open access article under the CC BY-NC-ND license

(http://creativecommons.org/licenses/by-nc-nd/4.0/).

A 39-year-old Senegalese man presented withchest pain and dyspnea after an acute ST-segment elevation myocardial infarction

(STEMI) 2 weeks earlier. Previous coronary angiog-raphy (CA) showed multivessel disease with a 99%proximal left anterior descending (LAD) artery lesionand high-grade stenosis of the left circumflex (LCx),obtuse marginal (OM1), posterior-descending artery(PDA), and right-posterior lateral branch (R-PLB).Coronary artery bypass grafting (CABG) was

EARNING OBJECTIVES

To consider the differential of eosinophilia.To appreciate that coronary vasculitis canpresent as an acute coronary syndrome muchlike atherosclerotic disease.To maintain a broad differential for prema-ture coronary artery disease.

N 2666-0849

m the aDepartment of Medicine, Virginia Commonwealth University Hea

gy, Pauley Heart Center, Virginia Commonwealth University Medical Cen

gy, Hunter Holmes McGuire Veteran Medical Center, Richmond, Virgi

ationships relevant to the contents of this paper to disclose.

e authors attest they are in compliance with human studies committe

titutions and Food and Drug Administration guidelines, including patien

it the JACC: Case Reports author instructions page.

nuscript received February 20, 2020; revised manuscript received May 1,

recommended, but the patient opted for a secondopinion. In the interim, he was medically managed,including with subcutaneous injections of enoxa-parin. On arrival, he complained of ongoing chestpain requiring a nitroglycerin infusion. Review of sys-tems included orthopnea, dyspnea on exertion, and 1year of unintentional weight loss. Physical examina-tion revealed bibasilar rales and lower-extremitypitting edema.

PAST MEDICAL HISTORY

The patient’s medical history included hypertension,type 2 diabetes mellitus, hyperlipidemia, and child-hood asthma.

INVESTIGATIONS

Admission electrocardiogram showed anterolateralischemia (Figure 1). Echocardiography demonstrated

https://doi.org/10.1016/j.jaccas.2020.05.001

lth System, Richmond, Virginia; bDivision of Cardi-

ter, Richmond, Virginia; and the cDivision of Cardi-

nia. The authors have reported that they have no

es and animal welfare regulations of the authors’

t consent where appropriate. For more information,

2020, accepted May 6, 2020.

Page 83: Full Issue PDF · The JACC: Case Reports Editorial Board and ACC editorial office would like to express their sincere gratitude to the guest editors below who worked tirelessly …

AB BR E V I A T I O N S

AND ACRONYM S

ANCA = antineutrophilic

cytoplasmic antibody

CA = coronary angiograph

CABG = coronary artery bypass

graft

CRP = C-reactive protein

DES = drug-eluting stent

EGPA = eosinophilic

granulomatosis with

polyangiitis

ESR = erythrocyte

sedimentation rate

LAD = left anterior descending

artery

LCx = left circumflex artery

LMCA = left main coronary

artery

OM1 = obtuse marginal artery

RCA = right coronary artery

R-PLB = right posterior-lateral

branch

J A C C : C A S E R E P O R T S , V O L . 2 , N O . 7 , 2 0 2 0 Kolominsky et al.J U N E 1 7 , 2 0 2 0 : 1 0 6 2 – 5 EGPA and Coronary Vasculitis

1063

a depressed ejection fraction (25% to 30%) withanteroapical hypokinesis. Serum troponin levelspeaked at 1.34 ng/ml (normal <0.49 ng/ml), and acomplete blood count (CBC) showed absolute eosin-ophilia of 1.6 103/ml (normal <0.7 103/ml). Erythrocytesedimentation rate (ESR) was elevated to 75 mm/h(normal <15 mm/h) with a C-reactive protein (CRP) of0.2 mg/ml (normal <0.3 mg/ml). Antineutrophiliccytoplasmic antibodies (ANCA) titers and results ofantimyeloperoxidase and antiproteinase 3 antibodieswere negative, with normal urinalysis test results.Chest computed tomography (CT) demonstrated peri-hilar and peri-bronchial consolidation. Cardiothoracicsurgery was consulted for revascularization, but thepatient was deemed a poor surgical candidatebecause of concerns regarding active pulmonaryinfection. The patient received salvage therapy withdrug-eluting stenting (DES) to the proximal LAD, LCx,and OM1 (Figure 1). Because of the presence of pul-monary infiltrates and endemic infections in hishome country of Senegal, a broad infectious work-upfor tuberculosis, invasive fungal disease, HIV,opportunistic infection, and parasitic infections wascompleted, the results of which were negative.

Two months later, he presented with non-ST-segment elevation myocardial infarction (NSTEMI)and persistent eosinophilia of 1.6 103/ml. His pulmo-nary infiltrates had resolved (Figure 2). He was ur-gently taken for CA and received DES to the RCA andR-PLB. One month later, he had another NSTEMI,complicated by hemodynamic instability withengagement of the left main coronary artery (LMCA)during CA, requiring placement of an intra-aorticballoon pump (IABP). The possibility of eosinophilicgranulomatosis with polyangiitis (EGPA) involvingthe coronary arteries was broached, given the pa-tient’s eosinophilia, negative infectious work-up, andrepeated coronary events.

DIFFERENTIAL DIAGNOSIS

The differential diagnosis of coronary artery diseaseincludes acute thrombotic plaque rupture, embolism,vasospasm, myocardial bridging, spontaneous coro-nary artery dissection, myocarditis, and vasculitis.

MANAGEMENT

The case was reviewed using a multidisciplinary teamapproach including cardiology, cardiac surgery, in-fectious disease, and rheumatology. Given the pa-tient’s ischemic burden and hemodynamic instabilityrequiring IABP, the decision was made to revascu-larize with CABG. Because of a suspected uncon-trolled systemic inflammatory disorder, evaluation

for heart transplantation was not pursued.Intraoperative right upper-lobe and left in-ternal mammary artery biopsies were ob-tained, which demonstrated prominentextravascular eosinophilic infiltrate consis-tent with EGPA (Figure 2). Because of concernfor loss of graft patency, the patient receivedhigh-dose prednisone and guideline-directedcardiac medical therapy post-operatively. Ondischarge, he was prescribed mepolizumab.

DISCUSSION

The American College of Rheumatology de-fines EGPA using 6 criteria: asthma, eosino-philia >10% on CBC, neuropathy, migratorypulmonary infiltrates, paranasal sinus ab-normality, and extravascular eosinophils onbiopsy (1,2). Using 4 of 6 criteria yields asensitivity of 85% and specificity of 99.7% (1).Our patient presented with migratory pul-monary infiltrates, a history of asthma, pe-ripheral eosinophilia, and biopsy-proven

eosinophilic pleuritis and arteritis (Figure 2). Taken incontext, his recurrent myocardial infarction wascaused by coronary vasculitis secondary to EGPA.Patients with EGPA coronary involvement can pre-sent with vasospasm, diffuse multivessel disease, orwithout a definable acute lesion (3,4); however, this isthe first case of hemodynamic instability requiringmechanical support.

The international incidence and prevalence ofEGPA is rare, with approximately 0.5 to 4.2 and 11 to14 cases per million, respectively (5). Epidemiologicdata have not demonstrated a relationship betweengender, ethnicity, or inheritance (5). Host immuno-genetic factors and environmental exposures mayplay a role in triggering EGPA; there are reportslinking leukotriene receptor antagonists, macrolideantibiotics, and silica particulates, although exactmechanisms remain poorly understood (5).

In a cross-sectional analysis of multicenter clin-ical trials in Europe, positive ANCA titers werefound in 38% of patients and associated with asmall-vessel vasculitis presentation, affecting thenerves and glomerulus (6). Our patient had normalrenal function, no neuropathy, normal urinalysis,and negative ANCA titers, suggesting a separatesubtype of EGPA. The ANCA-negative subtype ofEGPA is driven by eosinophilic infiltration withcardiopulmonary predominance and accounts for62% of all cases (6).

EGPA therapy is governed by the degree of organinvolvement at presentation; the Five Factor Score (7)

Page 84: Full Issue PDF · The JACC: Case Reports Editorial Board and ACC editorial office would like to express their sincere gratitude to the guest editors below who worked tirelessly …

FIGURE 1 Coronary Angiography and Electrocardiogram on Admission

Significant stenosis of the proximal LAD, LCx, and OM1 were present. Electrocardiogram (bottom) shows anterolateral ST-segment elevations,

lateral T-wave inversions, and Q waves through the anterior precordium. Angiography post-stenting (top, right) of the LAD, LCx, and OM1

with resolution of stenosis and flow. LAD ¼ left anterior descending; LCx ¼ left circumflex artery; OM1 ¼ obtuse marginal artery.

FIGURE 2 Computed Tomography

CT image during first admission (A) demonstrating diffuse infiltrates, which are resolved on second admission (B). Right upper lung wedge

biopsy during second admission (C) showing eosinophilic infiltrate (arrows) with respiratory bronchiole (i), pulmonary artery (ii), and pul-

monary parenchyma (iii). Wedge biopsy showing abundant fibrinous eosinophilic pleuritis (arrows) (D). Left internal mammary artery biopsy

(E) with eosinophil (chevron) infiltrating the media. Lamina propria of the vessel intima on far right (arrows).

Kolominsky et al. J A C C : C A S E R E P O R T S , V O L . 2 , N O . 7 , 2 0 2 0

EGPA and Coronary Vasculitis J U N E 1 7 , 2 0 2 0 : 1 0 6 2 – 5

1064

Page 85: Full Issue PDF · The JACC: Case Reports Editorial Board and ACC editorial office would like to express their sincere gratitude to the guest editors below who worked tirelessly …

J A C C : C A S E R E P O R T S , V O L . 2 , N O . 7 , 2 0 2 0 Kolominsky et al.J U N E 1 7 , 2 0 2 0 : 1 0 6 2 – 5 EGPA and Coronary Vasculitis

1065

is recommended for determining severity in eachpatient. Mild presentations can be treated withhigh-dose glucocorticoids, whereas more severe pre-sentations may require steroids and an immuno-modulator (8–10). Typical regimens includeprednisone at 1 mg/kg with adjunctive therapy ofeither cyclophosphamide or rituximab (8–11). The useof immunomodulatory therapy has been reported toinduce EGPA coronary vasculitis remission (3,4);however, as our patient had already suffered multipleischemic insults with hemodynamic instability, amore urgent solution was required. Remission ismaintained with steroids or steroid-sparing agents;mepolizumab, an anti-ILF immunomodulator, hasrecently gained FDA approval for relapsing EGPA (12).

FOLLOW-UP

The patient has been free of chest pain followingsurgery. Continued treatment with high-dose steroids

and immunomodulators has induced clinicalremission.

CONCLUSIONS

The presence of eosinophilia with the initial CT scanfindings and endemic nature of parasitic infections inSenegal prompted an extensive infectious work-upthat delayed diagnosis. At the time EGPA wasconsidered, the patient was in cardiac extremis andrequired urgent surgical revascularization. Our casehighlights the need to maintain a high index of clin-ical suspicion when laboratory values are incongruentwith typical patient presentations.

ADDRESS FOR CORRESPONDENCE: Dr. JeffreyKolominsky, Virginia Commonwealth University, Box980509, Richmond, Virginia 23298-0509. E-mail:[email protected].

RE F E RENCE S

1. Masi AT, Hunder GG, Lie JT, et al. The AmericanCollege of Rheumatology 1990 criteria for theclassification of churg-strauss syndrome (allergicgranulomatosis and angiitis). Arthritis Rheum2010;33:1094–100.

2. Churg J, Strauss L. Allergic angiitis and peri-arteritis nodosa. Am J Pathol 1951;27:277–301.

3. Chai JT, McGrath S, Lopez B, Dworakowski R.Eosinophilic granulomatosis with polyangiitis(Churg–Strauss syndrome) masquerading as acuteST-elevation myocardial infarction with completeresolution after immunosuppressive therapy: acase report. Eur Heart J Case Rep 2018;2:1–6.

4. Kakouros N, Bastiaenen R, Kourliouros A,Anderson L. Churg-Strauss presenting as acutecoronary syndrome: sometimes it’s zebras. BMJCase Rep 2011;2011:bcr0120113703.

5. Vaglio A, Buzio C, Zwerina J. Eosinophilicgranulomatosis with polyangiitis (Churg-Strauss):

state of the art. Allergy Eur J Allergy Clin Immunol2013;68:261–73.

6. Sablé-Fourtassou R, Cohen P, Mahr A, et al.Antineutrophil cytoplasmic antibodies and theChurg-Strauss syndrome. Ann Intern Med 2005;143:632–8.

7. Guillevin L, Pagnoux C, Seror R, et al. TheFive-Factor Score revisited: assessment ofprognoses of systemic necrotizing vasculitidesbased on the French Vasculitis Study Group(FVSG) cohort. Medicine (Baltimore) 2011;90:19–27.

8. McGeoc L, Twilt M, Famorca L, et al. CanVascrecommendations for the management of anti-neutrophil cytoplasm antibody-associated vascu-litides. J Rheumatol 2016;43:97–120.

9. Ntatsaki E, Carruthers D, Chakravarty K, et al.BSR and BHPR guideline for the management of

adults with ANCA-associated vasculitis. Rheuma-tology 2014;53:2306–9.

10. Groh M, Pagnoux C, Baldini C, et al. Eosino-philic granulomatosis with polyangiitis (Churg-Strauss) (EGPA) Consensus Task Force recom-mendations for evaluation and management. Eur JIntern Med 2015;26:545–53.

11. Bosch X, Guilabert A, Espinosa G, Mirapeix E.Treatment of antineutrophil cytoplasmicantibody-associated vasculitis. JAMA 2007;298:655–69.

12. Wechsler ME, Akuthota P, Jayne D, et al.Mepolizumab or placebo for eosinophilic gran-ulomatosis with polyangiitis. N Engl J Med 2017;376:1921–32.

KEY WORDS acute coronary syndrome,EGPA, eosinophilia, ischemic heart disease,premature coronary artery disease, vasculitis

Page 86: Full Issue PDF · The JACC: Case Reports Editorial Board and ACC editorial office would like to express their sincere gratitude to the guest editors below who worked tirelessly …

J A C C : C A S E R E P O R T S V O L . 2 , N O . 7 , 2 0 2 0

ª 2 0 2 0 T H E A U T H O R S . P U B L I S H E D B Y E L S E V I E R O N B E H A L F O F T H E AM E R I C A N

C O L L E G E O F C A R D I O L O G Y F O U N DA T I O N . T H I S I S A N O P E N A C C E S S A R T I C L E U N D E R

T H E C C B Y - N C - N D L I C E N S E ( h t t p : / / c r e a t i v e c o mm o n s . o r g / l i c e n s e s / b y - n c - n d / 4 . 0 / ) .

CASE REPORT

CLINICAL CASE

A Rare Presentation of Cardiomyopathyin Pregnancy

Elena Donald, MD,a Jay Leb, MD,a Martin Bialer, MD, PHD,b Kelly Axsom, MDa

ABSTRACT

L

ISS

Fro

Ch

La

Th

sti

the

Ma

Our patient presented in her third trimester of pregnancy with new onset of heart failure. A thorough workup in the initial

postpartum period with detailed past medical history, advanced imaging modalities, and a multidisciplinary

approach revealed a rare and treatable etiology of cardiomyopathy. (Level of Difficulty: Intermediate.)

(J Am Coll Cardiol Case Rep 2020;2:1066–9) © 2020 The Authors. Published by Elsevier on behalf of the

American College of Cardiology Foundation. This is an open access article under the CC BY-NC-ND license

(http://creativecommons.org/licenses/by-nc-nd/4.0/).

HISTORY OF PRESENTATION

A 33-year-old G1P1 woman presented 4 months post-partum for heart transplant evaluation.

Following an uneventful early pregnancy, the pa-tient presented during the third trimester with lowerextremity edema to her thighs, shortness of breathwith minimal exertion, and decreased urine output.She was admitted to her community hospital wheretransthoracic echocardiogram (TTE) showed leftventricular ejection fraction (LVEF) 20% to 25%, arestrictive filling pattern, left ventricular end-diastolic diameter 4.4 cm, intraventricular septum

EARNING OBJECTIVES

To be able to make a differential diagnosis ofcardiomyopathies in the peripartum/post-partum period.To understand the role of fatty acid oxida-tion disorders in the pathophysiology ofcardiomyopathy.

N 2666-0849

m the aNew York Presbyterian Hospital, Columbia University Irving Me

ildren’s Medical Center, Northwell Health System, Great Neck, New Yor

boratories. All other authors have reported that they have no relationship

e authors attest they are in compliance with human studies committees

tutions and Food and Drug Administration guidelines, including patient co

JACC: Case Reports author instructions page.

nuscript received February 18, 2020; revised manuscript received April 2

1.06 cm, mild right ventricular (RV) enlargement andreduced systolic function, moderate mitral andtricuspid regurgitation, and estimated pulmonaryartery systemic pressure 59 mm Hg. She had a suc-cessful vaginal delivery with hemodynamic moni-toring briefly requiring milrinone. Her laboratoryresults were remarkable for mildly elevated creati-nine kinase and creatinine 1.3 mg/dl. She was dis-charged home 10 days after delivery on guideline-directed medical therapy (GDMT). She lost 90 lbsduring this hospitalization.

One month later, cardiac magnetic resonance im-aging (CMR) showed mildly dilated left ventricle,LVEF 37% with diffuse hypokinesis, normal RV sizeand moderately reduced function, biatrial enlarge-ment, and moderate mitral and tricuspid regurgita-tion. Most notably, imaging showed diffuse andpatchy late gadolinium enhancement in the left andright ventricles, consistent with infiltrative or in-flammatory disease (Figure 1).

She was referred for heart transplant evaluationgiven her persistent New York Heart Associationfunctional class III symptoms.

https://doi.org/10.1016/j.jaccas.2020.05.022

dical Center, New York, New York; and the bCohen

k. Dr. Axsom has served as a consultant for Abbott

s relevant to the contents of this paper to disclose.

and animal welfare regulations of the authors’ in-

nsent where appropriate. For more information, visit

9, 2020, accepted May 06, 2020.

Page 87: Full Issue PDF · The JACC: Case Reports Editorial Board and ACC editorial office would like to express their sincere gratitude to the guest editors below who worked tirelessly …

AB BR E V I A T I O N S

AND ACRONYM S

GDMT = guideline-directed

medical therapy

LVEF = left ventricular

ejection fraction

MTPD = mitochondrial

trifunctional protein deficiency

RV = right ventricle/ventricular

TTE = transthoracic

cardiogram

J A C C : C A S E R E P O R T S , V O L . 2 , N O . 7 , 2 0 2 0 Donald et al.J U N E 1 7 , 2 0 2 0 : 1 0 6 6 – 9 A Rare Presentation of Cardiomyopathy in Pregnancy

1067

PAST MEDICAL HISTORY

Her past medical history is significant for 5 childhoodhospitalizations for rhabdomyolysis precipitated byinfection or exercise. Multiple muscle biopsies werenegative for glycogen storage disease or fatty acidoxidation disorders. She had no episodes in adulthood.

At 8 years of age, she had viral myocarditis pre-sumed secondary to coxsackievirus with fullrecovery.

Electrocardiogram at 12 years of age showednormal sinus rhythm with incomplete right bundlebranch block, left atrial enlargement, and possible RVenlargement.

Her sister had similar episodes of recurrent rhab-domyolysis in childhood.

DIFFERENTIAL DIAGNOSIS

� Peripartum cardiomyopathy� Glycogen storage disease/fatty acid oxidation

disorder� Malonic aciduria� Fabry disease� Danon disease� Friedreich’s ataxia� Amyloidosis� Other: human immunodeficiency virus, thyroid

disease, autoimmune etiology

INVESTIGATIONS

On the initial visit, the patient’s blood pressure was110/64 mm Hg, pulse was 77 beats/min, respiratory

FIGURE 1 CMR 1 Month Postpartum

Short-axis and 4-chamber post-contrast images, demonstrating diffuse,

involving both the left and right ventricular suggestive of a diffuse infil

rate was 16 breaths/min, and oxygen satura-tion was 98% on ambient air. Physical ex-amination demonstrated S3 on cardiacauscultation, jugular venous distension to12 cm, and lower extremity edema bilaterally.

Diagnostic testing was notable for a creat-inine of 1.28 mg/dl (estimated glomerularfiltration rate 48 ml/min/1.73 m2), mildmicrocytic anemia secondary to iron defi-ciency, and N-terminal pro–B-type natriureticpeptide elevated to 3,100 pg/ml (0 to 178 pg/

ml). Results of hemoglobin A1C, autoimmune panel,human immunodeficiency virus, troponin, creatininekinase, hepatobiliary panel, lipid panel, light chains,and thyroid function studies were normal.

Repeat TTE showed severely reduced LVEF at15%, left ventricular end-diastolic diameter 4.6 cm,normal wall thickness, RV mildly to moderatelyincreased in size with severely reduced systolicfunction, moderate biatrial enlargement, and esti-mated pulmonary artery systemic pressure60 mm Hg. Cardiopulmonary testing showed respi-ratory exchange ratio 1.13, VO2 15.4 (56% of age- andsize-predicted maximum, moderate to severe func-tional impairment), and VE/VCO2 elevated to 42.Holter monitoring showed a wandering atrial pace-maker arrhythmia.

MANAGEMENT

Given her childhood history and unusual CMR find-ings, the patient was referred to a geneticist. GDMTwas optimized with metoprolol succinate, spi-ronolactone, sacubitril/valsartan, and furosemide.

echo

predominantly mid-myocardial late gadolinium enhancement

trative myocardial process (red arrows).

Page 88: Full Issue PDF · The JACC: Case Reports Editorial Board and ACC editorial office would like to express their sincere gratitude to the guest editors below who worked tirelessly …

FIGURE 2 CMR After Dietary Modification, 1 Year Later

Short-axis and 4-chamber post-contrast images, demonstrates partial improvement in the diffuse and patchy late gadolinium enhancement

(red arrows).

Donald et al. J A C C : C A S E R E P O R T S , V O L . 2 , N O . 7 , 2 0 2 0

A Rare Presentation of Cardiomyopathy in Pregnancy J U N E 1 7 , 2 0 2 0 : 1 0 6 6 – 9

1068

Her biochemical genetics evaluation was signifi-cant for low total and free carnitine with normalacylcarnitine profile, urine organic acids, and serumlactic acid. Next-generation sequencing showed 2variants in the HADHB gene: 1 likely pathogenic(F430S) and 1 a variant of uncertain significance(D372V). One variant was inherited from each of herparents. A skin fibroblast culture was sent for fattyacid oxidation probe assay. Results were consistentwith HADHB deficiency, which results in mitochon-drial trifunctional protein deficiency. She was startedon a low-fat diet supplemented with medium chaintriglycerides and L-carnitine.

After 6 months of dietary modification, TTE showsregained cardiac function with LVEF 60% to 65%.CMR continues to show diffuse infiltrative diseasewith recovery of myocardial function (Figure 2).Repeat cardiopulmonary testing shows improvementin peak VO2 25 (87% of age- and size-predictedmaximum, mild functional impairment), VE/VCO2

26.4.

DISCUSSION

The patient originally presented to our advancedheart failure clinic for cardiac transplant evaluationwith a working diagnosis of peripartum cardiomyop-athy, a diagnosis of exclusion. CMR can be used on anindividual basis if alternative etiologies of cardio-myopathy are suspected, as seen in our patient. Herchildhood history of recurrent rhabdomyolysis inboth the patient and sister, “viral myocarditis,” andabnormal CMR with diffuse hyperenhancement gaveus clinical pause to suspect an alternative etiology of

cardiomyopathy and referral to a genetic specialist toidentify an unusual genetic disorder and modifytreatment.

Mitochondrial trifunctional protein deficiency(MTPD) is a rare autosomal recessive fatty acidoxidation disorder (1). The mitochondrial trifunc-tional protein is an inner mitochondrial membrane-bound protein complex encoded by HADHA andHADHB genes (2,3). The protein complex catalyzesthe last 3 steps in mitochondrial beta-oxidation oflong chain fatty acids (4). There are <100 cases ofMTPD reported in the published data, and 2 knownmutations that can lead to a broad spectrum of dis-ease manifestations (5). The most severe cases areusually fatal and present in the neonatal period withhepatic steatosis, skeletal myopathy/neuropathy, andcardiomyopathy. The mild form presents from in-fancy to adolescence with peripheral neuropathy,episodic myalgias, and rhabdomyolysis (6). There areonly case reports of first-time diagnoses made inadulthood. To our knowledge, there are no reportedcases of MTPD-related cardiomyopathy diagnosed inadulthood.

Cardiac involvement in long-chain fatty acidoxidation defects is a common manifestation andsignificantly contributes to disease mortality in in-fancy (7,8). The myocardium appears to be suscepti-ble to either direct toxicity from metabolicaccumulation of long-chain fatty acids or from thesubstrate deficiency itself (6,9,10). Most cases ofMTPD present with cardiomyopathy (dilated or hy-pertrophic) by the time a child reaches infancy. Thisearly manifestation within the first few months of lifeis likely in part due to the transition in cardiac energy

Page 89: Full Issue PDF · The JACC: Case Reports Editorial Board and ACC editorial office would like to express their sincere gratitude to the guest editors below who worked tirelessly …

J A C C : C A S E R E P O R T S , V O L . 2 , N O . 7 , 2 0 2 0 Donald et al.J U N E 1 7 , 2 0 2 0 : 1 0 6 6 – 9 A Rare Presentation of Cardiomyopathy in Pregnancy

1069

substrate from glucose (in utero) to fatty acidmetabolism.

Our patient likely presented with the mild pheno-type of MTPD in childhood. In retrospect it is unclearif her episode of myocarditis was an independentevent related to viral illness, or if it was related to herunderlying defect in fatty acid metabolism.

FOLLOW-UP

Reassuringly, our patient has recovered to NewYork Heart Association functional class I symptomswith normalization of LVEF. We plan to maintainher on GDMT, dietary restriction, and lifestylemodifications. Although there are very few reportsof cardiac transplantation in MTPD patients, dis-ease recurrence is not expected if she maintainsher diet (10).

CONCLUSIONS

To our knowledge, our patient is the first reportedcase of a diagnosis of MTPD-related cardiomyopathy

in adulthood. The combination of substrate depletionin addition to the imbalance between energy supplyand demand and intravascular volume expansionduring pregnancy may have led to her decom-pensated presentation in the third trimester. She willcontinue to be followed closely on GDMT with dietarymodifications for signs of disease progression. Shemay require advanced heart failure interventions inthe future.

ACKNOWLEDGMENTS The authors thank the sup-port staff within the Cohen Children’s MedicalCenter (CCMC) Northwell Health Division of MedicalGenetics and Human Genomics, and the adminis-tration and support staff at Columbia UniversityIrving Medical Center for Advanced Cardiac Care.

ADDRESS FOR CORRESPONDENCE: Dr. Elena Donald,New York Presbyterian Hospital, Columbia UniversityIrving Medical Center, 622 West 168th Street, VC 2ndFloor, New York, New York 10032. E-mail: [email protected]. Twitter: @elenamdonald.

RE F E RENCE S

1. Boutron A, Acquaviva C, Vianey-Saban C, et al.Comprehensive cDNA study and quantitativeanalysis of mutant HADHA and HADHB transcriptsin a French cohort of 52 patients with mitochon-drial trifunctional protein deficiency. Mol GenetMetab 2011;103:341–8.

2. Uchida Y, Izai K, Orii T, Hashimoto T. Novel fattyacid b-oxidation enzymes in rat liver mitochondria.J Biol Chem 1992;267:1034–41.

3. Kamijo T, Aoyama T, Komiyama A, Hashimoto T.Structural analysis of cDNAs for subunits of humanmitochondrial fatty acid b-oxidation trifunctionalprotein. Biochem Biophys Res Commun 1994;199:818–25.

4. Sander MH, Wanders RJ. A general introduc-tion to the biochemistry of mitochondrial fattyacid b-oxidation. J Inherit Metab Dis 2010;33:469–77.

5. den Boer ME, Dionisi-Vici C, Chakrapani A, vanThuijl AO, Wanders RJ, Wijburg FA. Mitochondrialtrifunctional protein deficiency: a severe fatty acidoxidation disorder with cardiac and neurologicinvolvement. J Pediatr 2003;142:684–9.

6. Spiekerkoetter A, Sun B, Khuchua Z,Bennett MJ, Strauss AW. Molecular and pheno-typic heterogeneity in mitochondrial trifunctionalprotein deficiency due to beta-subunit mutations.Hum Mutat 2003;21:598–607.

7. Vockley J, Marsden D, McCracken E, et al. Long-term major clinical outcomes in patients with longchain fatty acid oxidation disorders before andafter transition to triheptanoin treatment—aretrospective chart review. Mol Genet Metab2015;116:53–60.

8. Baruteau J, Sachs P, Broué P, et al. Clinical andbiological features at diagnosis in mitochondrial

fatty acid beta-oxidation defects: a French pedi-atric study from 187 patients. J Inherit Metab Dis2013;36:795–803.

9. Lehman JJ, Kelly DP. Transcriptionalactivation of energy metabolicswitches in the developing and hypertrophiedheart. Clin Exp Pharmacol Physiol 2002;29:339–45.

10. Bursle C, Weintraub R, Ward C, Justo R,Cardinal J, Coman D. Mitochondrial trifunctionalprotein deficiency: severe cardiomyopathy andcardiac transplantation. J Inherit Metab Dis 2018;40:91–5.

KEY WORDS cardiomyopathy, geneticdisorders, lipid metabolism disorders,pregnancy

Page 90: Full Issue PDF · The JACC: Case Reports Editorial Board and ACC editorial office would like to express their sincere gratitude to the guest editors below who worked tirelessly …

J A C C : C A S E R E P O R T S V O L . 2 , N O . 7 , 2 0 2 0

ª 2 0 2 0 T H E A U T H O R S . P U B L I S H E D B Y E L S E V I E R O N B E H A L F O F T H E AM E R I C A N

C O L L E G E O F C A R D I O L O G Y F O U N DA T I O N . T H I S I S A N O P E N A C C E S S A R T I C L E U N D E R

T H E C C B Y - N C - N D L I C E N S E ( h t t p : / / c r e a t i v e c o mm o n s . o r g / l i c e n s e s / b y - n c - n d / 4 . 0 / ) .

CASE REPORT

CLINICAL CASE

To Be or Not To Be

A Case of Recurrent Swelling Syndrome of Thoracic Duct

Ayesha Azmeen, MBBS,a John R. McArdle, MD,b John E. Foster, MD,c Ayesha Shaik, MBBSa

ABSTRACT

L

ISS

Fro

Ha

the

Th

ins

vis

Ma

Thoracic duct aneurysm is a rare entity presenting as a stable, asymptomatic, left supraclavicular swelling. We report an

unusual case of a thoracic duct aneurysm in a 71-year-old woman presenting as a recurrent swelling syndrome of the left

supraclavicular area associated with sporadic episodes of sharp left subcostal pain. (Level of Difficulty: Intermediate.)

(J Am Coll Cardiol Case Rep 2020;2:1070–3) © 2020 The Authors. Published by Elsevier on behalf of the

American College of Cardiology Foundation. This is an open access article under the CC BY-NC-ND license

(http://creativecommons.org/licenses/by-nc-nd/4.0/).

T he patient was a 71-year-old woman withepisodic left lower chest pain and associatedleft supraclavicular swelling for 6 years. She

had episodes of sharp pain in the left subcostal arealasting 2 to 6 h. The pain was soon after associatedwith a swelling in the left supraclavicular area. Thisoccurred over a duration of 6 years and increased infrequency. She reported no pain or tenderness inthe neck area. The subcostal pain was almost alwaysassociated with this inexplicable neck swelling,which disappears shortly after the resolution of thepain. No inciting factors were recognized. There wasno history of chest trauma or surgery prior to thisoccurring.

EARNING OBJECTIVES

To recognize the atypical and dynamic pre-sentation of a TD cyst.To understand the diagnosis and manage-ment of recurrent swelling syndrome of theTD.

N 2666-0849

m the aDepartment of Internal Medicine, University of Connecticut, Farm

rtford Hospital, Hartford, Connecticut; and the cJefferson Radiology, Ha

y have no relationships relevant to the contents of this paper to disclose

e authors attest they are in compliance with human studies committe

titutions and Food and Drug Administration guidelines, including patien

it the JACC: Case Reports author instructions page.

nuscript received February 24, 2020; revised manuscript received April 2

On physical examination, her neck was supple.There were no carotid bruits and carotid upstrokeswere normal bilaterally. There was a slight fullnessnoted in the left supraclavicular area withouttenderness (Figure 1). Her chest was clear to auscul-tation with normal heart sounds. Her abdomen wassoft without tenderness or masses. Her upper ex-tremities were warm and peripheral pulses werenormal bilaterally.

PAST MEDICAL HISTORY

She has a past surgical history of bladder prolapserepair but is otherwise healthy.

INVESTIGATIONS

The blood counts and biochemical tests did not revealany pathological findings. Ultrasonography of the softtissues of the left side of her neck performed duringthe acute phase of one of her spells demonstratedthin-walled tubular structures with no internalperfusion on color-coded Doppler (Figure 2).

https://doi.org/10.1016/j.jaccas.2020.05.003

ington, Connecticut; bPulmonology and Critical Care,

rtford, Connecticut. The authors have reported that

.

es and animal welfare regulations of the authors’

t consent where appropriate. For more information,

8, 2020, accepted May 6, 2020.

Page 91: Full Issue PDF · The JACC: Case Reports Editorial Board and ACC editorial office would like to express their sincere gratitude to the guest editors below who worked tirelessly …

FIGURE 1 Thoracic Duct Aneurysm

Left supraclavicular fullness.

AB BR E V I A T I O N S

AND ACRONYM S

FNA = fine needle aspiration

TD = thoracic duct

J A C C : C A S E R E P O R T S , V O L . 2 , N O . 7 , 2 0 2 0 Azmeen et al.J U N E 1 7 , 2 0 2 0 : 1 0 7 0 – 3 Recurrent Swelling Syndrome of Thoracic Duct

1071

Serial computed tomography scans of the chestshowed a small, smooth ovoid fluid density structureinterposed between the descending thoracic aortaand the thoracic spine at the T5 vertebra initiallymeasuring 1.1 � 0.7 cm increasing to 2.1 � 1.2 cm over1 year. Similar findings were noted on magneticresonance imaging.

MANAGEMENT

Due to suspicion of thoracic duct (TD) origin,lymphangiography was performed. A sonographicsurvey was performed of the left neck and supra-clavicular area prior to the procedure for localizationof the previously seen dilated tubular structures.Following the administration of local anesthesia, apuncture was performed of the tubular structure inthe left neck using a micropuncture needle underultrasound guidance. Serial injections of contrastwith pressure were performed into this structure,some with digital subtraction angiography. The ter-minal TD was noted to be aneurysmal and lobulatedover a 2.2-cm segment measuring up to 10 mmdiameter (Figure 3). There was a high-grade, irregularstenosis of the TD approximately 3 mm proximal to itsanastomosis with the left subclavian vein (Video 1).Reflux was seen up the left internal jugular vein. Afew lymphatic collaterals proximal to the stenosiswere noted. The TD was also dilated over its entirelength measuring approximately 3.5 mm in diameter(Figure 4). The cisterna chyli was opacified located inthe midline spanning from mid-T12 through mid-L1

vertebrae level measuring 2.9 � 1.5 cm. There wastransient filling of some outpouching and branchingstructures arising from the lower TD around the levelof the mid-T10 to mid-T11 vertebrae. Contrary to im-aging studies, there was no filling of any focal dilatedlymphatic structures at the T5 level. The inner dilatorof the micropuncture kit was then inserted usingSeldinger technique. Several microwires were madeto cross the stenosis in the TD with some success(Figure 5, Video 2). The catheter was removed and asterile dressing was placed. The patient tolerated theprocedure well. There were no post-operativecomplications.

FOLLOW-UP

Complete resolution of her symptoms was notedpost-procedure at 1-year follow-up.

DIFFERENTIAL DIAGNOSIS

Thyroid lesions are unusual in the supraclaviculararea, but metastatic papillary carcinoma of the

thyroid can present here; however, it is usu-ally heterogenous and less likely cystic (1).Thymic cyst, parathyroid cyst, lipoma, cystichygroma, and branchial cleft cyst can presentas a left supraclavicular mass. Less

commonly, TD cysts present as a stable, supra-clavicular mass.

DISCUSSION

The TD originates from the cisterna chyli in theabdomen, penetrates the diaphragm at the aorticaperture and travels through the posterior and supe-rior mediastinum serving as a channel for lymph todrain into the venous system. Cystic dilations are rareand may occur anywhere along the course of the duct(1). Furthermore, the terms “thoracic duct cyst” and“thoracic duct aneurysm” have been used inter-changeably in the published data. Suzuki et al. (2)described a TD cyst as an aneurysmal dilation of theTD. Livermore et al. (3) reported a TD aneurysm as acyst in the left supraclavicular area. Review of thepublished data reports 30 cases of TD cysts, the ma-jority presenting as an asymptomatic stable swelling.Cases of cervical TD cysts have been reported withsymptoms of pain, dyspnea, dysphagia, and hoarse-ness related to compression of neck structures (4).Wan et al. (1) reported a case of large abdominal TD

Page 92: Full Issue PDF · The JACC: Case Reports Editorial Board and ACC editorial office would like to express their sincere gratitude to the guest editors below who worked tirelessly …

FIGURE 2 Ultrasonography

Multiple cysts with no color Doppler flow.

FIGURE 3 Terminal Stenosis

Lymphangiogram showing 2.2 � 1 cm dilatation of the thoracic

duct with terminal stenosis (arrow).

Azmeen et al. J A C C : C A S E R E P O R T S , V O L . 2 , N O . 7 , 2 0 2 0

Recurrent Swelling Syndrome of Thoracic Duct J U N E 1 7 , 2 0 2 0 : 1 0 7 0 – 3

1072

cyst presenting with abdominal pain managed bysurgical resection. Veziant et al. (7) reported a case ofrecurrent swelling of the terminal TD with sponta-neous bilateral chylothorax and chyloperitoneumtriggered by a fat-rich diet. The left radicular pain inour case appeared to be related to a lymphocele thatenlarges when the patient is experiencing acutelymphatic obstruction with impingement of an adja-cent nerve.

Congenital weakness of the duct and acquireddegeneration of the TD wall due to inflammation oratherosclerosis have been indicated as etiologicalfactors. There have been reports of cervical TD cystsdeveloping after blunt or whiplash injury to the neck(4). No triggers were found in our patient. Therecurrent swelling may reflect the variable flow ofchyle and dynamic obstruction of the terminal TDrelated to stenosis and debris collection.

Both computed tomography and magnetic reso-nance imaging demonstrate the cystic nature of thelesion along with anatomic location and connections.The addition of lipiodol has been attempted in a fewcases to increase specificity by enhancing visualiza-tion of the TD (5). Ultrasonography has been sug-gested for primary diagnosis (6). Fine needleaspiration (FNA) reveals a milky fluid with a highconcentration of triglycerides. Due to the ambiguousnature of the cyst and pain syndrome, FNA was de-ferred to avoid provoking iatrogenic infection,inflammation, or thrombosis of vital neck structures.Lymphangiography helped to confirm the TD origin,delineate the communication of the supraclavicularcyst with the TD, recognize collaterals, and tracecystic areas.

Although the management of stable-sized cystsincludes FNA or surgical resection, there is no stan-dard management for recurrent swelling associatedwith terminal TD occlusion. Two cases of successfulcervical lymphovenous anastomosis have been re-ported (7). In the current case, obstruction of thelymphoid flow could be from a focal stenosis in theterminal portion of TD. This was relieved by dilationand pressure with contrast. It is unclear if this is theabsolute treatment of her condition or if her symp-toms could recur. No recurrence was noted at 1-yearfollow-up. We discussed embolization of the TDabove the level of cisterna chyli should her symptomsrecur. However, reopening the TD stenosis directlycould be tried again given the prolonged benefit.

CONCLUSIONS

TD aneurysm with terminal duct stenosis can presentas a recurrent swelling over variable areas along its

Page 93: Full Issue PDF · The JACC: Case Reports Editorial Board and ACC editorial office would like to express their sincere gratitude to the guest editors below who worked tirelessly …

FIGURE 4 Dilated Thoracic Duct

Origin at the cisterna chyli at T12-L1 level.

FIGURE 5 Management

Guidewire passing through the terminal stenosis of thoracic

duct into left subclavian vein.

J A C C : C A S E R E P O R T S , V O L . 2 , N O . 7 , 2 0 2 0 Azmeen et al.J U N E 1 7 , 2 0 2 0 : 1 0 7 0 – 3 Recurrent Swelling Syndrome of Thoracic Duct

1073

length with associated symptoms due to compressionof surrounding structures.

Dilation of the stenotic region was performed un-der fluoroscopy in this case. Successful surgical

management with TD ligation or lympho-venousanastomosis has also been reported.

Lymphangiography is the diagnostic gold standard.

ADDRESS FOR CORRESPONDENCE: Dr. AyeshaAzmeen, 2 Park Place, Apartment A26C, Hartford,Connecticut 06106. E-mail: [email protected]: @AyeshaAzmeen.

RE F E RENCE S

1. Wan X, Zhou Z. A giant thoracic duct cyst as thecause of abdomen pain: a case report and reviewof the literature. Ann Thorac Cardiovasc Surg2015;21:487–91.

2. Suzuki M, Uchida Y, Ogushi K, Otake S,Kuwano H. A thoracic duct cyst in 10-year-old boy:the youngest case report and review of the liter-ature. J Ped Surg Case Reports 2016;6:1–4.

3. Livermore GH, Kryzer TC, Patow CA. Aneurysmof the thoracic duct presenting as an asymptom-atic left supraclavicular neck mass. OtolaryngolHead Neck Surg 1993;109 3 Pt 1:530–3.

4. Gupta M, Lovelace TD, Sukumar M,Gosselin MV. Cervical thoracic duct cyst. J ThoracImaging 2005;20:107–9.

5. Lecanu JB, Gallas D, Biacabe B, Bonfils P.Lymphocele of the thoracic duct presenting as aleft supraclavicular mass: a case report and reviewof the literature. Auris Nasus Larynx 2001;28:275–7.

6. Gottwald F, Iro H, Finke C, Zenk J.Thoracic duct cysts: a rare differential diag-nosis. Otolaryngol Head Neck Surg 2005;132:330–3.

7. Veziant J, Sakka L, Galvaing G, Tardy MM,Cassagnes L, Filaire M. Lymphovenous anasto-mosis for recurrent swelling syndrome andchylous effusion due to cervical thoracic duct cyst.J Vasc Surg 2015;62:1068–70.

KEY WORDS chest pain, contrast agent,stenosis, thoracic

APPENDIX For supplemental videos,please see the online version of this paper.

Page 94: Full Issue PDF · The JACC: Case Reports Editorial Board and ACC editorial office would like to express their sincere gratitude to the guest editors below who worked tirelessly …

J A C C : C A S E R E P O R T S V O L . 2 , N O . 7 , 2 0 2 0

ª 2 0 2 0 T H E A U T H O R S . P U B L I S H E D B Y E L S E V I E R O N B E H A L F O F T H E AM E R I C A N

C O L L E G E O F C A R D I O L O G Y F O U N DA T I O N . T H I S I S A N O P E N A C C E S S A R T I C L E U N D E R

T H E C C B Y - N C - N D L I C E N S E ( h t t p : / / c r e a t i v e c o mm o n s . o r g / l i c e n s e s / b y - n c - n d / 4 . 0 / ) .

CASE REPORT

CLINICAL CASE

Multimodal Imaging for the Diagnosis ofIsolated Cardiac Sarcoidosis

David W. Louis, MD,a Alexa Papaila, MD, MPH,a Wasiq Sheikh, MD,a Michael K. Atalay, MD, PHD,b

Brian G. Abbott, MDa

ABSTRACT

L

ISS

Fro

the

tho

Th

sti

the

Ma

We report a case of isolated cardiac sarcoidosis (CS) diagnosed using a multimodality imaging approach. A patient pre-

sented after an out-of-hospital, ventricular fibrillation–mediated cardiac arrest. The use of echocardiography, cardiac

magnetic resonance, and fluorodeoxyglucose-positron emission tomography enabled the diagnosis of isolated CS.

(Level of Difficulty: Beginner.) (J Am Coll Cardiol Case Rep 2020;2:1074–8) © 2020 The Authors. Published by Elsevier

on behalf of the American College of Cardiology Foundation. This is an open access article under the CC BY-NC-ND license

(http://creativecommons.org/licenses/by-nc-nd/4.0/).

HISTORY OF PRESENTATION

A 72-year-old Caucasian woman with a history ofadvanced atrioventricular (AV) block with AV disso-ciation presented after an out-of-hospital, ventricularfibrillation–mediated cardiac arrest. Physical exami-nation was notable for elevated jugular venous pres-sure, bibasilar crackles, and cool extremities withoutedema. Limited bedside echocardiogram revealedsevere left ventricular (LV) dysfunction and regionalwall motion abnormalities (RWMAs). Emergency

EARNING OBJECTIVES

CS should be suspected in patients withnonischemic cardiomyopathy, cardiac con-duction disease, and ventricular arrhythmias.A multimodality imaging approach should beutilized when diagnosing CS.Updated guidelines now allow for a clinicaldiagnosis of CS without endomyocardialbiopsy.

N 2666-0849

m aThe Lifespan Cardiovascular Institute, Warren Alpert Medical SchoolbDepartment of Diagnostic Imaging, Warren Alpert Medical School of B

rs have reported that they have no relationships relevant to the content

e authors attest they are in compliance with human studies committees

tutions and Food and Drug Administration guidelines, including patient co

JACC: Case Reports author instructions page.

nuscript received February 14, 2020; revised manuscript received April 2

coronary angiography revealed nonobstructive coro-nary artery disease.

PAST MEDICAL HISTORY

The patient’s medical history included hypothyroid-ism and asymptomatic advanced AV block of un-known duration for which she had refused permanentpacemaker placement.

DIFFERENTIAL DIAGNOSIS

The differential diagnosis for nonischemic cardio-myopathy (NICM) includes infiltrative disease(sarcoidosis and amyloidosis), acute myocarditis(idiopathic giant cell myocarditis, necrotizing eosin-ophilic myocarditis, and lymphocytic myocarditis),stress-induced cardiomyopathy, left ventricularnoncompaction cardiomyopathy, and hypertrophiccardiomyopathy, as well as cardiotoxic drugs (che-motherapeutics), thyroid disease, and illicit sub-stances (cocaine, amphetamines).

https://doi.org/10.1016/j.jaccas.2020.05.021

of Brown University, Providence, Rhode Island; and

rown University, Providence, Rhode Island. The au-

s of this paper to disclose.

and animal welfare regulations of the authors’ in-

nsent where appropriate. For more information, visit

5, 2020, accepted May 6, 2020.

Page 95: Full Issue PDF · The JACC: Case Reports Editorial Board and ACC editorial office would like to express their sincere gratitude to the guest editors below who worked tirelessly …

AB BR E V I A T I O N S

AND ACRONYM S

CMR = cardiac magnetic

resonance

CS = cardiac sarcoidosis

FDG = fluorodeoxyglucose

PET = positron emission

tomography

J A C C : C A S E R E P O R T S , V O L . 2 , N O . 7 , 2 0 2 0 Louis et al.J U N E 1 7 , 2 0 2 0 : 1 0 7 4 – 8 Multimodal Imaging for the Diagnosis of Isolated Cardiac Sarcoidosis

1075

INVESTIGATIONS

Notable laboratory findings on arrival included a peaktroponin-I of 0.028 ng/ml, brain-type natriureticpeptide of 585 pg/ml, whole blood lactate of3.7 mmol/l, and serum creatinine 1.18 mg/dl. Com-plete blood count, basic metabolic panel, thyroidfunction tests, and liver function tests were otherwisewithin normal limits. Admission 12-lead electrocar-diogram (Figure 1) revealed varying PR conduction,trigeminy, right bundle branch block, left anteriorfascicular block, and ST-segment depressions in V2 toV6. Transthoracic echocardiogram revealed an LVejection fraction of 15% to 20% and a tricuspidannular plane systolic excursion of 10 mm. ExtensiveRWMAs were noted with diffuse akinesis of theinferoseptal, inferior, inferolateral, anterolateral, andanterior wall segments, and hypokinesis of the basaland mid-anteroseptal walls. Grade III diastolicdysfunction was noted. Strain imaging was notperformed.

Cardiac magnetic resonance (CMR) was performedrevealing global hypokinesis of the LV with severelyreduced systolic function and extensive areas of bothtransmural and subepicardial late gadoliniumenhancement (LGE) throughout most basal segments,

FIGURE 1 Initial Electrocardiogram

Electrocardiography post-cardiac arrest: varying PR conduction, trigemin

T-wave inversions in V2 to V6.

as well as patchy subepicardial LGE in themid-ventricular and apical segments(Figure 2). No hilar adenopathy was noted.Given our high index of suspicion for isolatedcardiac sarcoidosis (CS), fluorodeoxyglucose(FDG) positron emission tomography (PET)with computed tomography (CT) imaging wasperformed, which demonstrated increasedFDG uptake in the septal, distal anterior, and

mid-to-basal inferior walls of the LV and apicalanterior wall of the right ventricle (Figures 3A to 3C,Figure 4). Single-photon emission CT (SPECT) perfu-sion abnormalities were seen on the accompanyingresting myocardial single-photon emission CT perfu-sion images, most prominently involving the septum(Figures 3D to 3F). Areas of FDG uptake on PET/CTcorrelated to areas of LGE on CMR.

MANAGEMENT

A diagnosis of isolated CS was established. The pa-tient was initiated on oral prednisone along withlisinopril and spironolactone. Beta blockade was nottolerated. Amiodarone was given to decrease furtherrisk of ventricular arrhythmias. An implantablecardioverter-defibrillator was placed.

y, right bundle branch block, left anterior fascicular block, and deep

Page 96: Full Issue PDF · The JACC: Case Reports Editorial Board and ACC editorial office would like to express their sincere gratitude to the guest editors below who worked tirelessly …

FIGURE 2 Cardiac Magnetic Resonance Imaging

Late gadolinium enhancement (acquired using inversion-recovery with spoiled gradient echo readout) short-axis (A to C) and long-axis

(D to E) images showing a nonischemic hyperenhancement pattern (arrows). T2-weighted midventricular short-axis image (F) acquired using

short-tau inversion recovery with fast spin echo readout and 51 ms echo time demonstrating heterogenous high signal (arrows) consistent

with edema in the septal and anterior segments.

Louis et al. J A C C : C A S E R E P O R T S , V O L . 2 , N O . 7 , 2 0 2 0

Multimodal Imaging for the Diagnosis of Isolated Cardiac Sarcoidosis J U N E 1 7 , 2 0 2 0 : 1 0 7 4 – 8

1076

DISCUSSION

Sarcoidosis is a multisystem disease propagatedlargely by overactivation of tissue macrophagesleading to variable combinations of bilateral hilarlymphadenopathy, pulmonary, cardiac, neurologic,bone, ophthalmic, or dermatologic involvement. CSappears to be underdiagnosed by clinicians, as au-topsy records have documented cardiac involvementin upwards of 25% of patients with known sarcoid-osis. Epidemiological studies have found significantlyhigher rates of CS in Japanese women over the age of50 years and African Americans (1).

Signs and symptoms of CS are vague and mayinclude palpitations, pre-syncope, ectopy, ventriculararrhythmias, AV block, bundle branch blocks, andsudden death. Distinguishing CS from acute myocar-ditis (idiopathic giant cell myocarditis, necrotizingeosinophilic myocarditis, and lymphocytic myocar-ditis) can be challenging. Symptom onset to time ofpresentation is more rapid in idiopathic giant cellmyocarditis and eosinophilic myocarditis compared

with CS (2). Similarly, symptomatic heart failure ismore common with myocarditis (2). Elevated eosin-ophil counts may be suggestive of eosinophilicmyocarditis.

Cardiac imaging plays an integral role in the diag-nosis of CS (Table 1). Common transthoracic echo-cardiogram findings include LV systolic dysfunction,RWMA in a noncoronary distribution, interventricularseptal thinning, and abnormal strain rates (1,3). CMRis suggestive of CS when LGE is seen in a patchy andmultifocal distribution, usually involving the basalseptum and basal lateral walls, mid-myocardium, andepicardium of the myocardium (1,3). FDG-PET typi-cally shows focal FDG uptake with or without restingperfusion defects and wall motion abnormalities (1,3).FDG-PET uptake patterns may help differentiate CSfrom other nonischemic processes including acutemyocarditis. In CS, regions of fibrosis along the basalor midventricular septum may be seen, comparedwith myocarditis, which commonly follows specificvascular territories in the inferior and inferolateralmyocardial segments (4). Endomyocardial biopsy can

Page 97: Full Issue PDF · The JACC: Case Reports Editorial Board and ACC editorial office would like to express their sincere gratitude to the guest editors below who worked tirelessly …

FIGURE 3 FDG-PET and Technetium Tc-99m Tetrofosmin SPECT Imaging

Fluorodeoxyglucose (FDG) positron emission tomography (PET) images (A to C) showing increased FDG uptake in the septal, distal anterior,

and mid-to-basal inferior walls of the left ventricle and apical anterior wall of the right ventricle (arrows). Resting myocardial single photon

emission computed tomography (SPECT) perfusion images (D to F) showing abnormalities most prominently involving the interventricular

septum (arrows).

J A C C : C A S E R E P O R T S , V O L . 2 , N O . 7 , 2 0 2 0 Louis et al.J U N E 1 7 , 2 0 2 0 : 1 0 7 4 – 8 Multimodal Imaging for the Diagnosis of Isolated Cardiac Sarcoidosis

1077

be pursued if histological examination is warranted(5); however, the patchy distribution of fibrosiscommonly leads to false negative samples, with sen-sitivities as low as 20% (6).

Guidelines for the diagnosis of CS have recentlybeen updated to reflect advancements in multi-modality cardiac imaging. The 2016 Japanese Cir-culation Society Guideline on Diagnosis andTreatment of Cardiac Sarcoidosis now separatespatients into either a “clinical diagnosis group” or“histological diagnosis group” based on whether ornot endomyocardial biopsy has been performed,and provides additional guidelines on diagnosingisolated CS. These guidelines differ from other so-cietal guidelines (7–10), which all require endo-myocardial biopsy for diagnostic confirmation. Amultimodality imaging approach without endo-myocardial biopsy may allow for a more rapid

initiation of immunosuppressive therapy andimproved outcomes.

FOLLOW-UP

Repeat FDG-PET 6 months later revealed improve-ment in LV ejection fraction from 20% to 58% withoutsignificant interval changes in perfusion abnormal-ities or intense regional FDG uptake. Outpatient PPMinterrogation revealed underlying sinus rhythmwithout further ventricular arrhythmias. Overall,these findings suggest persistent myocardial inflam-mation despite clinical recovery, possibly indicativeof premature follow-up imaging.

CONCLUSIONS

Isolated CS should be suspected in patientswith an unexplained nonischemic cardiomyopathy,

Page 98: Full Issue PDF · The JACC: Case Reports Editorial Board and ACC editorial office would like to express their sincere gratitude to the guest editors below who worked tirelessly …

FIGURE 4 Fluorodeoxyglucose Positron Emission

Tomography With Computed Tomography

Fluorodeoxyglucose positron emission tomography with

computed tomography imaging of the heart showing increased

activity in the interventricular septum (arrow), distal anterior,

and mid to basal inferior walls. In the correct clinical context,

these findings are consistent with a pro-inflammatory state

such as sarcoidosis.

TABLE 1 Multimodal Imaging Findings in Cardiac Sarcoidosis

Echocardiogram � LV dysfunction� RWMA not explained by coronary artery dis-

ease or in a noncoronary artery distribution� Basal thinning of the ventricular septum� Abnormal ventricular wall anatomy (ventricu-

lar aneurysm, regional ventricular wallthickening)

� Abnormal strain imaging

CMR � 1 or more patchy regions of late gadoliniumenhancement atypical for myocardial infarction(i.e., sparing the endocardial border and not inthe distribution of prior myocardial infarction)

FDG-PET � CS most commonly associated with focal FDGuptake in isolation or on background of milddiffuse uptake with or without resting perfu-sion defects and wall motion abnormalities

� PET perfusion tracers can help excludeobstructive coronary artery disease

� Identification of ongoing active inflammation

CS ¼ cardiac sarcoidosis; FDG ¼ fluorodeoxyglucose; LV ¼ left ventricular;PET ¼ positron emission tomography; RWMA ¼ regional wall motionabnormalities.

Louis et al. J A C C : C A S E R E P O R T S , V O L . 2 , N O . 7 , 2 0 2 0

Multimodal Imaging for the Diagnosis of Isolated Cardiac Sarcoidosis J U N E 1 7 , 2 0 2 0 : 1 0 7 4 – 8

1078

conduction abnormalities, and/or life-threateningarrhythmias. A multimodality imagingapproach should be considered in cases of sus-pected CS. Recent guideline updates now allow

for the diagnosis of CS without endomyocardialbiopsy.

ADDRESS FOR CORRESPONDENCE: Dr. Brian G.Abbott, The Lifespan Cardiovascular Institute, War-ren Alpert Medical School of Brown University, 593Eddy Street, RIH APC 814, Providence, Rhode Island02903. E-mail: [email protected].

RE F E RENCE S

1. Birnie DH, Nery PB, Ha AC, Beanlands RS. Car-diac sarcoidosis. J Am Coll Cardiol 2016;68:411–21.

2. Okura Y, Dec GW, Hare JM, et al. A clinical andhistopathologic comparison of cardiac sarcoidosisand idiopathic giant cell myocarditis. J Am CollCardiol 2003;41:322–9.

3. Slart R, Glaudemans A, Lancellotti P, et al.A joint procedural position statement on imagingin cardiac sarcoidosis: from the Cardiovascular andInflammation & Infection Committees of the Eu-ropean Association of Nuclear Medicine, the Eu-ropean Association of Cardiovascular Imaging, andthe American Society of Nuclear Cardiology. J NuclCardiol 2018;25:298–319.

4. James OG, Christensen JD, Wong TZ, Borges-Neto S, Koweek LM. Utility of FDG PET/CT in in-flammatory cardiovascular disease. Radiographics2011;31:1271–86.

5. Cooper LT, Baughman KL, Feldman AM, et al. Therole of endomyocardial biopsy in the managementof cardiovascular disease: a scientific statementfrom the American Heart Association, the AmericanCollege of Cardiology, and the European Society ofCardiology. Circulation 2007;116:2216–33.

6. Uemura A, Morimoto S, Hiramitsu S, Kato Y,Ito T, Hishida H. Histologic diagnostic rate ofcardiac sarcoidosis: evaluation of endomyocardialbiopsies. Am Heart J 1999;138:299–302.

7. Judson MA, Baughman RP, Teirstein AS,Terrin ML, Yeager H Jr. for the ACCESS ResearchGroup. Defining organ involvement in sarcoidosis:the ACCESS proposed instrument. A case controletiologic study of sarcoidosis. Sarcoidosis VascDiffuse Lung Dis 1999;16:75–86.

8. Birnie DH, Sauer WH, Bogun F, et al. HRS expertconsensus statement on the diagnosis and

management of arrhythmias associated with car-diac sarcoidosis. Heart Rhythm 2014;11:1305–23.

9. Hunninghake GW, Costabel U, Ando M, et al.ATS/ERS/WASOG statement on sarcoidosis.American Thoracic Society/European RespiratorySociety/World Association of Sarcoidosis and otherGranulomatous Disorders. Sarcoidosis Vasc DiffuseLung Dis 1999;16:149–73.

10. Terasaki F, Azuma A, Anzai T, et al. JCS 2016guideline on diagnosis and treatment of cardiacsarcoidosis- digest version. Circ J 2019;83:2329–88.

KEY WORDS cardiac magnetic resonance,cardiomyopathy, heart failure,positron-emission tomography, ventricularfibrillation

Page 99: Full Issue PDF · The JACC: Case Reports Editorial Board and ACC editorial office would like to express their sincere gratitude to the guest editors below who worked tirelessly …

J A C C : C A S E R E P O R T S VO L . 2 , N O . 7 , 2 0 2 0

ª 2 0 2 0 T H E A U T H O R S . P U B L I S H E D B Y E L S E V I E R O N B E H A L F O F T H E A M E R I C A N

C O L L E G E O F C A R D I O L O G Y F OU N D A T I O N . T H I S I S A N O P E N A C C E S S A R T I C L E U N D E R

T H E C C B Y - N C - N D L I C E N S E ( h t t p : / / c r e a t i v e c o mm o n s . o r g / l i c e n s e s / b y - n c - n d / 4 . 0 / ) .

CASE REPORT

CLINICAL CASE

Resolution of Hypoxia and Ascites WithPercutaneous Intervention of MustardBaffle Obstruction and Leak

Riyad Y. Kherallah, MD,a Darren Harrison, MD,a Wilson W. Lam, MD,b Srinath Gowda, MD,b John J. Seger, MD,c

Peter R. Ermis, MDb

ABSTRACT

L

ISS

Fro

Te

rep

Th

ins

vis

Ma

A 45-year-old man with history of Mustard repair for transposition of the great arteries, cirrhosis, and chronic hypoxemic

respiratory failure presented for subacute worsening of his chronic symptoms, which were found to be secondary to a

previously unrecognized baffle stenosis and leak. Percutaneous intervention resolved his ascites and hypoxia.

(Level of Difficulty: Intermediate.) (J Am Coll Cardiol Case Rep 2020;2:1079–83) © 2020 The Authors. Published

by Elsevier on behalf of the American College of Cardiology Foundation. This is an open access article under the

CC BY-NC-ND license (http://creativecommons.org/licenses/by-nc-nd/4.0/).

A 45-year-old man with complex medical his-tory, including Mustard repair for simpletransposition of the great arteries (D-TGA)

presented for dyspnea. His D-TGA was palliated byballoon atrial septostomy and Blalock-Hanlon atrialseptectomy as a neonate, followed by a Mustardrepair at 2 years of age with a complicated post-operative course requiring revision with Dacron(Maquet, Rastatt, Germany) grafts 1 day after theinitial procedure (Figure 1). Since then, he devel-oped multiple arrhythmias including sinus nodedysfunction necessitating pacemaker implantation

EARNING OBJECTIVES

To review Mustard repair anatomy andphysiology.To understand the clinical presentation andmanagement of baffle complications.

N 2666-0849

m the aDepartment of Internal Medicine, Baylor College of Medicine, Ho

xas Children’s Hospital, Houston, Texas; and the cHouston Electrophysio

orted that they have no relationships relevant to the contents of this pap

e authors attest they are in compliance with human studies committe

titutions and Food and Drug Administration guidelines, including patien

it the JACC: Case Reports author instructions page.

nuscript received April 30, 2020; accepted May 6, 2020.

at age 9 years, ventricular tachycardia requiringablation at age 35 years, atrial tachycardia, andatrial fibrillation status post-ablation 1 year ago.Additionally, he had chronic resting hypoxia (satu-rations of 88% to 92% on room air) for which hehad been receiving 2 l of oxygen for multiple years.He had cirrhosis (Child Pugh class B complicated byhepatic encephalopathy and ascites) that was firstdiagnosed 20 years ago after he underwent explor-atory laparotomy for a ruptured colon. Since age40 years, he had received monthly therapeuticparacenteses.

The patient presented with 3 to 4 months ofworsening of his chronic exertional dyspnea, along-side increased lower extremity swelling and ascitesrequiring more frequent therapeutic paracenteses. Hewas referred to our congenital heart specialty centerby his general cardiologist because of medicalcomplexity.

https://doi.org/10.1016/j.jaccas.2020.05.004

uston Texas; bTexas Adult Congenital Heart Center,

logy Associates, Houston, Texas. The authors have

er to disclose.

es and animal welfare regulations of the authors’

t consent where appropriate. For more information,

Page 100: Full Issue PDF · The JACC: Case Reports Editorial Board and ACC editorial office would like to express their sincere gratitude to the guest editors below who worked tirelessly …

FIGUR

Diagra

and IV

segme

LV ¼ l

ABBR EV I A T I ON S

AND ACRONYMS

D-TGA = simple transposition

of the great arteries

IVC = inferior vena cava

RV = right ventricle

SVC = superior vena cava

Kherallah et al. J A C C : C A S E R E P O R T S , V O L . 2 , N O . 7 , 2 0 2 0

Resolution of Hypoxia and Ascites With Percutaneous Intervention of Mustard Baffle Obstruction and Leak J U N E 1 7 , 2 0 2 0 : 1 0 7 9 – 8 3

1080

DIFFERENTIAL DIAGNOSIS

We were concerned about a long-termcomplication of the patient’s Mustardrepair. Systemic baffle stenosis may lead toelevated systemic pressures and explain hisliver dysfunction, edema, and ascites. Abaffle leak with right-to-left shunt would

explain his hypoxia. Right ventricle (RV) failure maydevelop as a result of chronic exposure to systemicpressures with D-TGA anatomy. Tricuspid regurgita-tion may develop secondary to RV dysfunction orbecause of altered geometry post-surgery. Finally,atrial arrhythmias, including both bradyarrhythmiasand tachyarrhythmias, are especially common in pa-tients with a history of Mustard repairs.

INVESTIGATIONS

Pacemaker interrogation showed rate-controlledatrial fibrillation with minimal pacing requirements.A transthoracic echocardiography revealed moderateRV dysfunction without significant valvular regur-gitation. Cardiac computed tomography demon-strated moderate to severe narrowing of thesystemic baffle superior limb (Figure 2A) and

E 1 Mullen Diagram of Mustard Repair

m depicting our patient’s cardiac anatomy (A) before and (B) after

C-b. Meanwhile, pulmonary blood is redirected to the RA via a pu

nt in addition to combined stenosis and leak in the SVC-b segment

eft ventricle; PA ¼ pulmonary artery; RA ¼ right atrium; RV ¼ rig

moderate to severe narrowing of the inferior limbwith dilation of the inferior vena cava (IVC) hepaticsegment (Figure 3A).

MANAGEMENT

The patient was taken to the catheterization labora-tory for evaluation and possible intervention of abaffle stenosis. Angiography demonstrated superiorvena cava (SVC) and IVC baffle obstructions withmean gradients of 5 and 4 mm Hg, respectively(Figures 2B and 3B). In addition, there was a baffleleak across the systemic baffle superior limb withright-to-left shunting (Figure 2B). Cardiac index waslow (1.8 l/min/m2), pulmonary vascular resistancewas normal (<2 WU), and pulmonary blood flow tosystemic blood flow ratio was 0.9:1.

The pacemaker leads traversing the superior limbwere first extracted by using a combination of a 14-FGuidelight laser sheath and an 11-F TightRail (Spec-tranetics, Colorado Springs, Colorado). Then, a Pal-maz XL 4010 stent (Cordis, Santa Clara, California)followed by an 8-zig 3.4-cm Cheatham Platinumcovered stent (Braun Interventional Systems, Hop-kinton, New York) was deployed to relieve the SVCbaffle obstruction and leak (Figure 4). The IVC limb

Mustard repair. Systemic blood is redirected to the LA via the SVC-b

lmonary baffle (not pictured). Our patient had stenosis in the IVC-b

. AO ¼ aorta; IVC-b ¼ systemic baffle inferior limb; LA ¼ left atrium;

ht ventricle; SVC-b ¼ systemic baffle superior limb.

Page 101: Full Issue PDF · The JACC: Case Reports Editorial Board and ACC editorial office would like to express their sincere gratitude to the guest editors below who worked tirelessly …

FIGURE 2 Systemic Baffle Superior Limb Stenosis

(A) Axial view computed tomography scan showing moderate to severe narrowing of the systemic baffle superior limb (red arrow). (B) Angiogram of the systemic baffle

superior limb demonstrating severe stenosis (solid white arrow) and baffle leak with right-to-left shunting (small white arrows). Abbreviations as in Figure 1.

J A C C : C A S E R E P O R T S , V O L . 2 , N O . 7 , 2 0 2 0 Kherallah et al.J U N E 1 7 , 2 0 2 0 : 1 0 7 9 – 8 3 Resolution of Hypoxia and Ascites With Percutaneous Intervention of Mustard Baffle Obstruction and Leak

1081

stenosis was relieved by placing a 36-mm Max LargeDiameter stent (Boston Scientific, Marlborough,Massachusetts) (Figure 5). Following stent placement,pulmonary wedge angiography demonstratedpatency of the pulmonary venous baffle. As the pa-tient’s pacemaker interrogation showed a low pacingrequirement, a new pacemaker was not implanted.

FIGURE 3 Systemic Baffle Inferior Limb Stenosis

(A) Coronal view computed tomography scan demonstrating narrowing o

arrow). (B) Angiogram of the systemic baffle inferior limb showing foca

cava; RPA ¼ right pulmonary artery; other abbreviations as in Figure 1.

DISCUSSION

Baffles are surgical conduits composed of eitherbiological or synthetic material commonly utilizedfor the redirection of blood flow in congenitalheart surgery. Mustard and Senning repairs,commonly used in the 1960s to 1980s to correct

f the systemic baffle inferior limb (thin red arrow) with upstream dilation of the IVC (thick red

l severe stenosis (arrow) with aneurysmal dilation proximal to stenosis. IVC ¼ inferior vena

Page 102: Full Issue PDF · The JACC: Case Reports Editorial Board and ACC editorial office would like to express their sincere gratitude to the guest editors below who worked tirelessly …

FIGURE 4 Systemic Baffle Superior Limb Intervention

(A) Balloon angioplasty of the systemic baffle superior limb demonstrating tight waist (arrow). (B) Angiogram showing Palmaz 4010 XL stent implantation with mild

residual stenosis (larger arrow) and a baffle leak (small arrows). (C) An 8-zig 3.4-cm CP covered stent (larger arrow) was positioned proximally within the Palmaz XL

stent (small arrow). (D) Angiogram demonstrates resolution of the baffle leak after covered stent deployment (arrows).

FIGURE 5 Systemi

(A) Balloon angiopla

(Boston Scientific) ac

Kherallah et al. J A C C : C A S E R E P O R T S , V O L . 2 , N O . 7 , 2 0 2 0

Resolution of Hypoxia and Ascites With Percutaneous Intervention of Mustard Baffle Obstruction and Leak J U N E 1 7 , 2 0 2 0 : 1 0 7 9 – 8 3

1082

previously fatal TGA, utilized baffles to reroutesystemic and pulmonary blood flow such that theRV becomes the physiological systemic pump andthe left ventricle becomes the physiological pul-monary pump (Figure 1) (1). Other surgeries thatuse baffles include Fontan procedures and thedouble-switch operation for congenitally correctedTGA (1).

Unfortunately, baffles are prone to deterioration.Among patients with Mustard repair, obstructionsand leaks have a lifetime incidence of up to 30% (2–4).

c Baffle Inferior Limb Intervention

sty of the systemic baffle inferior limb demonstrating moderate waist (arrow

ross the stenotic site (arrow). (C) Angiogram of the inferior limb demonstra

Their clinical presentation can be variable, includingpulmonary hypertension (with pulmonary venousbaffle obstruction), dyspnea, SVC syndrome, volumeoverload, chylothorax, and protein-losing enteropa-thy (1,3). Our patient had chronic hypoxia related toright-to-left shunting from an SVC baffle leak andliver cirrhosis secondary to IVC baffle stenosis andhepatic congestion. Although it is difficult to pinpointwhen his baffle complications began, they likely wentunrecognized for multiple years, as inferred by hissymptom duration. Early referral to a specialized

). (B) Angiogram showing positioning of a 36 IntraStent Max

tes no residual stenosis after stent deployment (arrow).

Page 103: Full Issue PDF · The JACC: Case Reports Editorial Board and ACC editorial office would like to express their sincere gratitude to the guest editors below who worked tirelessly …

J A C C : C A S E R E P O R T S , V O L . 2 , N O . 7 , 2 0 2 0 Kherallah et al.J U N E 1 7 , 2 0 2 0 : 1 0 7 9 – 8 3 Resolution of Hypoxia and Ascites With Percutaneous Intervention of Mustard Baffle Obstruction and Leak

1083

clinical center of excellence could have preventeddelays in care.

The largest available studies for transcatheterintervention in baffle stenosis report favorable out-comes with low rates of restenosis over midtermfollow-up (5). Repairing defects may be performed byusing a variety of approaches, with plain balloon an-gioplasty, bare-metal stents, covered stents, andendovascular grafts all being reasonable choices. Weused a covered stent over the SVC baffle to exclude aconcomitant leak. Because angioplasty alone has highrestenosis rates, stents were placed to ensure longer-term patency and facilitate access for future electro-physiological procedures.

Finally, a notable complicating factor is the pres-ence of pacemaker leads traversing the systemicbaffle, which many patients with Mustard repair willhave because of the high rates of sinus nodedysfunction in this population. Joint procedures withlead extraction are often necessary. Previous reportsshow that combined transvenous lead extraction andstenting for baffle complications is safe (6).

FOLLOW-UP

The patient underwent an uncomplicated hospitalcourse with dramatic symptom improvement. Hisdyspnea resolved, and he was successfully weanedoff supplemental oxygen. At the 9-month follow-up,he required no more paracenteses. There has beenno cause for reintervention.

CONCLUSIONS

Baffle obstruction and leak are common complica-tions in patients with Mustard repair that may havesubtle and variable presentations. Referral to a clin-ical center of excellence specialized in the care ofpatients with congenital heart defects may avoid de-lays in care.

ADDRESS FOR CORRESPONDENCE: Dr. Riyad YazanKherallah, Baylor College of Medicine, 2243 South-gate Boulevard, Houston, Texas 77030. E-mail: [email protected].

RE F E RENCE S

1. Gatzoulis MA, Webb GD, Daubeney PEF. Diag-nosis and Management of Adult Congenital HeartDisease. Philadelphia, PA: Elsevier, 2018.

2. Ashraf MH, Cotroneo J, DiMarco D,Subramanian S. Fate of long-term survivors ofmustard procedure (inflow repair) for simple andcomplex transposition of the great arteries. AnnThorac Surg 1986;42:385–9.

3. Kron IL, Rheuban KS, Joob AW, et al. Baffleobstruction following the mustard operation:

cause and treatment. Ann Thorac Surg 1985;39:112–5.

4. Khairy P, Landzberg MJ, Lambert J,O’Donnell CP. Long-term outcomes after theatrial switch for surgical correction of trans-position: a meta-analysis comparing the Mustardand Senning procedures. Cardiol Young 2004;14:284–92.

5. Bradley EA, Cai A, Cheatham SL, et al. Mustardbaffle obstruction and leak—how successful are

percutaneous interventions in adults? Prog PediatrCardiol 2015;39:157–63.

6. Laredo M, Waldmann V, Chaix MA, et al. Leadextraction with baffle stenting in adults withtransposition of the great arteries. J Am Coll Car-diol EP 2019;5:671–80.

KEY WORDS baffle obstruction, baffleleak, congenital transposition of the greatarteries, Mustard procedure

Page 104: Full Issue PDF · The JACC: Case Reports Editorial Board and ACC editorial office would like to express their sincere gratitude to the guest editors below who worked tirelessly …

J A C C : C A S E R E P O R T S V O L . 2 , N O . 7 , 2 0 2 0

ª 2 0 2 0 T H E A U T H O R S . P U B L I S H E D B Y E L S E V I E R O N B E H A L F O F T H E AM E R I C A N

C O L L E G E O F C A R D I O L O G Y F O U N DA T I O N . T H I S I S A N O P E N A C C E S S A R T I C L E U N D E R

T H E C C B Y - N C - N D L I C E N S E ( h t t p : / / c r e a t i v e c o mm o n s . o r g / l i c e n s e s / b y - n c - n d / 4 . 0 / ) .

CASE REPORT

CLINICAL CASE

Red Blood Cell Fragmentation SyndromeAfter Placement of MitraClip

Christopher D. Barrett, MD, Lukasz Cerbin, MD, Aken Desai, MD, John D. Carroll, MD

ABSTRACT

L

ISS

Fro

the

Th

sti

the

Ma

An 87-year-old woman with a history of trastuzumab-induced left ventricular dysfunction underwent the MitraClip

(Abbott Vascular, Santa Clara, California) procedure for myxomatous mitral regurgitation. She presented a month later

with severe intravascular hemolytic anemia, attributed to the MitraClip. She underwent surgical mitral valve replacement

and had resolution of hemolysis. (Level of Difficulty: Advanced.) (J Am Coll Cardiol Case Rep 2020;2:1084–8)

© 2020 The Authors. Published by Elsevier on behalf of the American College of Cardiology Foundation. This is an

open access article under the CC BY-NC-ND license (http://creativecommons.org/licenses/by-nc-nd/4.0/).

HISTORY OF PRESENT ILLNESS

An 87-year-old African-American woman with a his-tory of chemotherapy-induced myocardial dysfunc-tion initially presented with symptoms of orthopnea,dyspnea with minimal exertion, and lower extremityedema. She was found to have myxomatous mitralvalve disease on surface echocardiography with a leftventricular ejection of fraction (LVEF) of 58%.Transesophageal echocardiogram showed severedegenerative mitral valve disease with 3þ mitralregurgitation, most of which resulted from poor

EARNING OBJECTIVES

To develop a differential diagnosis forrecurrent dyspnea and fatigue followingMitraClip placement.To recognize hemolytic anemia as a rarecomplication of the MitraClip procedure.To demonstrate that hemolysis secondary toMitraClip can be corrected with surgicalmitral valve replacement.

N 2666-0849

m the Division of Cardiology, University of Colorado School of Medicine

y have no relationships relevant to the contents of this paper to disclose

e authors attest they are in compliance with human studies committees

tutions and Food and Drug Administration guidelines, including patient co

JACC: Case Reports author instructions page.

nuscript received February 18, 2020; revised manuscript received May 1,

coaptation of the A2-P2 leaflets (Figures 1 and 2,Videos 1 and 2). No isolated leaflet flail or prolapsewas identified.

The patient was evaluated in a multidisciplinaryvalve clinic and determined to be a high-risk candi-date for surgical mitral valve replacement due to age,frailty, and history of breast cancer with chest radia-tion. She was instead referred for a percutaneousmitral valve repair with a MitraClip system (AbbottVascular, Santa Clara, California), which she under-went in April 2019. She had 2 NTR clips placed be-tween the A2 and P2 mitral valve leaflets. Bytransesophageal echocardiogram, residual 2þ mitralregurgitation was noted at the end of the procedure,with a mean diastolic gradient across the mitral valveof 3 mm Hg at a heart rate of 82 beats/min. The degreeof residual mitral regurgitation was acceptable andexpected in this case because it is challenging toachieve minimal residual mitral regurgitation usingtranscatheter edge to edge repair devices, particularlyin degenerative mitral valve that can be classified asBarlow’s disease rather than an isolated flail leaflet.She was discharged without incident on post-procedure day 3.

https://doi.org/10.1016/j.jaccas.2020.05.009

, Aurora, Colorado. The authors have reported that

.

and animal welfare regulations of the authors’ in-

nsent where appropriate. For more information, visit

2020, accepted May 6, 2020.

Page 105: Full Issue PDF · The JACC: Case Reports Editorial Board and ACC editorial office would like to express their sincere gratitude to the guest editors below who worked tirelessly …

AB BR E V I A T I O N S

AND ACRONYM S

LVEF = left ventricular

ejection fraction

J A C C : C A S E R E P O R T S , V O L . 2 , N O . 7 , 2 0 2 0 Barrett et al.J U N E 1 7 , 2 0 2 0 : 1 0 8 4 – 8 MitraClip Hemolysis

1085

She presented to the hospital 1 month afterdischarge with recurrent dyspnea and fatigue. Shealso noted dark brown urine without associatedflank pain or dysuria. She was afebrile with ablood pressure of 191/92 mm Hg, heart rate of123 beats/min, and oxygen saturation of 95% onambient air. Physical exam showed jugularvenous distension, inspiratory crackles, and bilat-eral lower extremity edema. On cardiac examshe was tachycardic with a regular rhythm andhad a 4/6 holosystolic murmur radiating to theapex. She had no friction rub or extra heartsounds.

FIGURE 1 Transesophageal Echocardiogram at 60� Omniplane Befor

Mitral valve visualized by transesophageal echocardiography in a mid-es

and post–MitraClip procedure after presentation with intravascular hem

MEDICAL HISTORY

The patient has a history of stage III breastcancer diagnosed in 2011 for which she un-derwent lumpectomy, lymph node dissec-

tion, and adjuvant chemotherapy with chestradiation in 2012. She subsequently developedtrastuzumab-induced cardiac dysfunction withdecline in her LVEF from normal to 25%, and globalhypokinesis of the left ventricle. Echocardiogram atthat time showed mild-to-moderate mitral valveregurgitation without prolapse of the mitral valveleaflets. Trastuzumab therapy was stopped, and she

e and After MitraClip Placement

ophageal commissural view prior to the MitraClip procedure (A, B),

olysis (C, D).

Page 106: Full Issue PDF · The JACC: Case Reports Editorial Board and ACC editorial office would like to express their sincere gratitude to the guest editors below who worked tirelessly …

FIGURE 2 Transesophageal Echocardiogram at 120� Omniplane Before and After MitraClip Placement

Mitral valve visualized by transesophageal echocardiogram in a mid-esophageal view at 120-degree omniplane before the MitraClip procedure

(A, B), and post–MitraClip procedure (C, D).

Barrett et al. J A C C : C A S E R E P O R T S , V O L . 2 , N O . 7 , 2 0 2 0

MitraClip Hemolysis J U N E 1 7 , 2 0 2 0 : 1 0 8 4 – 8

1086

was treated with guideline-directed medical therapyand had recovery of her LVEF to 65% by 2018 withonly mild mitral regurgitation.

DIFFERENTIAL DIAGNOSIS

The differential diagnosis for her symptoms includedhemolytic anemia related to her MitraClip procedure,clip failure with single leaflet detachment, progres-sion of mitral stenosis, and hypertensive crisisresulting in symptoms of worsening congestive heartfailure.

INVESTIGATIONS

Electrocardiogram (ECG) showed sinus tachycardiawith biatrial enlargement and left ventricular hyper-trophy by voltage criteria. Repeat transthoracicechocardiogram showed normal LVEF with moderatemitral regurgitation that appeared similar to her post-MitraClip echocardiogram. There was no evidence ofsingle leaflet detachment. There was no new leafletprolapse, flail, or perforation. Her transmitral dia-stolic gradient was measured at 5 mm Hg at a heart

Page 107: Full Issue PDF · The JACC: Case Reports Editorial Board and ACC editorial office would like to express their sincere gratitude to the guest editors below who worked tirelessly …

FIGURE 3 3-Dimensional Transesophageal Echocardiography After MitraClip Placement

Three-dimensional imaging of the mitral valve by transesophageal echocardiography following MitraClip procedure showing residual mitral

regurgitation via multiple (medial and lateral) regurgitant jets.

J A C C : C A S E R E P O R T S , V O L . 2 , N O . 7 , 2 0 2 0 Barrett et al.J U N E 1 7 , 2 0 2 0 : 1 0 8 4 – 8 MitraClip Hemolysis

1087

rate of 96 beats/min. Repeat transesophageal echo-cardiogram showed a high-velocity regurgitant jetlateral to the 2 MitraClips (Figures 1 to 3, Videos 3, 4,5, and 6). The peak regurgitant velocity was 6.0 m/swith a peak systolic gradient of 144 mm Hg.

Laboratory evaluation was notable for a decline inhemoglobin from 14.3 g/dl at the time of her Mitra-Clip procedure to 9.3 g/dl at the time of admission.Dipstick urinalysis showed dark urine with a largequantity of blood but few red blood cells by micro-scopy (Table 1). No leukocytosis, pulmonary in-filtrates, or urinary infection were identified.Peripheral blood smear showed 3þ schistocytes,serum haptoglobin was <30 mg/dl, serum lactatedehydrogenase was elevated at 2,153 U/l, reticulo-cyte index was 3.8, and a direct agglutination testwas negative. Given her race, she was evaluated forglucose-6-phosphate dehydrogenase (G6PD) defi-ciency as a cause of hemolysis and was found tohave normal G6PD enzyme activity level (16.3 U/l).

MANAGEMENT

The patient was initially managed with intravenousantihypertensive and diuretic therapy. She wastransfused to maintain a serum hemoglobin >8 g/dl.Hematology was consulted and agreed with thediagnosis of intravascular hemolysis associated withher MitraClip procedure. Despite symptomaticimprovement with blood pressure control and vol-ume optimization, it was considered unlikely that

she would sustain long-term improvement with anongoing intravascular hemolysis requiring bloodtransfusion. No cause of hemolysis was identifiedother than MitraClip placement with a high-velocityand turbulent regurgitant jet. She was thereforereferred to cardiac surgery for reconsideration ofmitral valve replacement. She underwent cardiacsurgery with removal of the 2 MitraClips andplacement of a 27-mm Magna Ease bioprostheticmitral valve. Postoperative echocardiogram showedmildly reduced left ventricular systolic function(LVEF 48%) and normal functioning prostheticmitral valve with a mean gradient of 2.9 mm Hg at aheart rate of 101 beats/min and trace prostheticregurgitation. She was discharged home on post-operative day 12. Follow-up laboratory testingshowed resolution of anemia and normalization oflactate dehydrogenase.

DISCUSSION

This is a patient who underwent a MitraClip proced-ure for severe degenerative mitral regurgitation. Shesubsequently developed a red blood cell fragmenta-tion syndrome, very likely related to turbulent flowacross her mitral valve. This probably resulted from asmall, high-velocity, mitral regurgitant jet in thelateral aspect of the valve (P1-P2 commissure) adja-cent to the MitraClip. It is less likely that red bloodcell shearing would occur from low-velocity ante-grade flow across the two MitraClip devices. No other

Page 108: Full Issue PDF · The JACC: Case Reports Editorial Board and ACC editorial office would like to express their sincere gratitude to the guest editors below who worked tirelessly …

TABLE 1 Laboratory Trends Suggestive of Intravascular Hemolysis Following MitraClip

Procedure With Resolution in Hemolysis and Hemoglobinuria After Surgical Clip Removal

and Mitral Valve Replacement

BeforeMitraClip

AfterMitraClip

Post -Mitral ValveReplacement

HGB, g/dl 14.4 8.2 14.2

HCT, % 42.8 26.3 42

LDH, U/l — 2,153 284

Total bilirubin, mg/dl 0.9 2.6 1.3

Hematuria on microscopy Negative 3þ Negative

RBC on urine microscopy(cells per high power field)

0-3 4-10 0-3

HCT ¼ hematocrit; HGB ¼ hemoglobin; LDH ¼ lactate dehydrogenase; RBC ¼ red blood cells.

Barrett et al. J A C C : C A S E R E P O R T S , V O L . 2 , N O . 7 , 2 0 2 0

MitraClip Hemolysis J U N E 1 7 , 2 0 2 0 : 1 0 8 4 – 8

1088

plausible causes of hemolytic anemia were identifiedin this case, and hemolysis resolved after referral forsurgical MitraClip explant and bioprosthetic mitralvalve replacement.

The safety and efficacy of percutaneous mitralvalve repair with the MitraClip system have been wellestablished in select patients (1,2), and proceduralvolume has increased worldwide for management ofpatients with severe mitral regurgitation who are athigh risk for cardiac surgery (3,4). Hemolytic anemiais a rare, but well-described complication of surgical

mitral valve repair (5). Clinically significant hemolysishas only been previously reported once after percu-taneous MitraClip placement (6). Management ofMitraClip-associated red blood cell fragmentationsyndrome with MitraClip removal has not been pre-viously reported.

FOLLOW-UP

The patient had resolution of hemolysis and hemo-globinuria as well as normalization of her hemoglobinafter surgical MitraClip explant and mitral valvereplacement. She was recently diagnosed withrecurrent biventricular systolic heart failure, which isbeing managed in the ambulatory setting.

CONCLUSIONS

This case is an example of intravascular hemolyticanemia occurring secondary to MitraClip placement, arare adverse event that was alleviated with surgicalvalve replacement.

ADDRESS FOR CORRESPONDENCE: Dr. ChristopherD. Barrett, University of Colorado Hospital, 12505 East16th Avenue, 3rd Floor, Aurora, Colorado 80045.E-mail: [email protected]. Twitter: @cdsbarrett1.

RE F E RENCE S

1. Feldman T, Foster E, Glower DD, et al. Percu-taneous repair or surgery for mitral regurgitation.N Engl J Med 2011;364:1395–406.

2. Stone GW, Lindenfeld J, Abraham WT, et al.Transcatheter mitral-valve repair in patientswith heart failure. N Engl J Med 2018;379:2307–18.

3. Maisano F, Franzen O, Baldus S, et al.Percutaneous mitral valve interventions in thereal world: early and 1-year results from theACCESS-EU, a prospective, multicenter, non-randomized post-approval study of the MitraClip

therapy in Europe. J Am Coll Cardiol 2013;62:1052–61.

4. Zhou S, Egorova N, Moskowitz G, et al. Trendsin MitraClip, mitral valve repair, and mitral valvereplacement from 2000 to 2016. J Thorac Car-diovasc Surg 2020 Jan 20 [E-pub ahead of print].

5. Abourjaili G, Torbey E, Alsaghir T, Olkovski Y,Costantino T. Hemolytic anemia following mitralvalve repair: a case presentation andliterature review. Exp Clin Cardiol 2012;1:248–50.

6. Yokoyama H, Mizuno S, Saito S. Sub-acute hemolytic anemia after transcatheter

edge-to-edge mitral valve repair: a casereport. Catheter Cardiovasc Interv 2020;95:1230–4.

KEY WORDS echocardiography, hemolysis,insufficiency, MitraClip, mitral valve, valverepair

APPENDIX For supplemental videos,please see the online version of this paper.